Jump to content

Wikipedia:Reference desk/Science: Difference between revisions

From Wikipedia, the free encyclopedia
Content deleted Content added
Nunh-huh (talk | contribs)
Line 777: Line 777:
:It's a key tenet of [[general relativity]] that accelerations are not relative ''except'' that they can't be distinguished from gravitational fields. You can use an [[accelerometer]] to measure accelerations; the most simple of which is a cup of coffee. Start to accelerate it, just a little—you'll start to spill it. But if you have it in an inertial frame, no matter how fast you are going (say, 67,000 miles an hour), it'll sit there placidly.
:It's a key tenet of [[general relativity]] that accelerations are not relative ''except'' that they can't be distinguished from gravitational fields. You can use an [[accelerometer]] to measure accelerations; the most simple of which is a cup of coffee. Start to accelerate it, just a little—you'll start to spill it. But if you have it in an inertial frame, no matter how fast you are going (say, 67,000 miles an hour), it'll sit there placidly.
:Einstein used to comment that theory of relativity was something of a misnomer: what's important is not what is relative, but the very few things which are ''not'' relative, like the speed of light. It is from finding the few things which are not relative—and the implications of that—that the genius of his theory comes. --[[Special:Contributions/98.217.8.46|98.217.8.46]] ([[User talk:98.217.8.46|talk]]) 00:59, 17 March 2008 (UTC)
:Einstein used to comment that theory of relativity was something of a misnomer: what's important is not what is relative, but the very few things which are ''not'' relative, like the speed of light. It is from finding the few things which are not relative—and the implications of that—that the genius of his theory comes. --[[Special:Contributions/98.217.8.46|98.217.8.46]] ([[User talk:98.217.8.46|talk]]) 00:59, 17 March 2008 (UTC)

:[[KE]] and [[momentum]] both depend on the [[reference frame]]


== DBasing the money supply ==
== DBasing the money supply ==

Revision as of 02:19, 17 March 2008

Welcome to the science section
of the Wikipedia reference desk.
Select a section:
Want a faster answer?

Main page: Help searching Wikipedia

   

How can I get my question answered?

  • Select the section of the desk that best fits the general topic of your question (see the navigation column to the right).
  • Post your question to only one section, providing a short header that gives the topic of your question.
  • Type '~~~~' (that is, four tilde characters) at the end – this signs and dates your contribution so we know who wrote what and when.
  • Don't post personal contact information – it will be removed. Any answers will be provided here.
  • Please be as specific as possible, and include all relevant context – the usefulness of answers may depend on the context.
  • Note:
    • We don't answer (and may remove) questions that require medical diagnosis or legal advice.
    • We don't answer requests for opinions, predictions or debate.
    • We don't do your homework for you, though we'll help you past the stuck point.
    • We don't conduct original research or provide a free source of ideas, but we'll help you find information you need.



How do I answer a question?

Main page: Wikipedia:Reference desk/Guidelines

  • The best answers address the question directly, and back up facts with wikilinks and links to sources. Do not edit others' comments and do not give any medical or legal advice.
See also:


March 10

Circular Motion

Imagine that you swing about your head a ball attached to the end of a string. The ball moves at a constant velocity in a horizontal circle. Can the string be exactly horizontal? Why? - Thanks —Preceding unsigned 00:23, 10 March 2008 (UTC)

Is this homework? Try setting up the problem with a horizontal string and working out all the forces involved. (There aren't too many—gravity, tension, centripetal/centrifugal.) -- BenRG (talk) 00:38, 10 March 2008 (UTC)[reply]

The answer is "no", and the presence of a downward gravitational force vector should make the reason why obvious. =Axlq 01:55, 10 March 2008 (UTC)[reply]

Sure it can. The swinger just has to provide enough force to counteract gravity. Oops. Clarityfiend (talk) 02:25, 10 March 2008 (UTC)[reply]
No it can't as would be obvious if you draw and diagram and add the forces involved. Theresa Knott | The otter sank 05:21, 10 March 2008 (UTC)[reply]
Nitpick alert: the ball can't be moving at constant velocity either, though it may be moving at constant speed. Constant velocity requires travel in a straight line. AlmostReadytoFly (talk) 09:09, 10 March 2008 (UTC)[reply]

Energy Drink Death

First off, I know that drinking enough of anything can kill you.

My question is though, our article on Cocaine (drink) states that it contains 350% as much caffiene as redbull. That seems like a lot to me. How much caffeine can the human body tolerate? (I am very disinterested in killing myself, this is just curiosity) —Preceding unsigned comment added by 24.128.192.184 (talk) 00:35, 10 March 2008 (UTC)[reply]

Caffeine. Quote: "The LD50 of caffeine in humans is dependent on weight and individual sensitivity and estimated to be about 150 to 200 milligrams per kilogram of body mass, roughly 80 to 100 cups of coffee for an average adult taken within a limited timeframe that is dependent on half-life" Wisdom89 (T / C) 00:51, 10 March 2008 (UTC)[reply]

Wouldn't the LDLo or LDmin be more appropriate than the LD50 in this case? —Keenan Pepper 20:15, 10 March 2008 (UTC)[reply]
The drink seems to have 280 mg of caffeine per serving. So that means you'd start getting into the dangerous range after 15 consecutive (and immediate) servings or so (when you start getting around 5 g of caffeine), depending on your weight. Which seems like a lot, but when compared to coffee, that's about an order of magnitude more toxic. --98.217.18.109 (talk) 15:21, 12 March 2008 (UTC)[reply]
Not quite an order of magnitude, more like less than twice (but yes, that's still quite a bit). Coffee contains an average of about 135mg of caffeine in 240mL, while Cocaine has 280mg per 240 mL (Red Bull has 80mg in the same volume). – ClockworkSoul 16:04, 12 March 2008 (UTC)[reply]
Note that most energy drinks in NZ recommend a 2 cans max daily intake. However the reason according to a number of sources (I believe one can said it and this source [1] also supports it) is not the caffeine level but the Vitamin B12 level. However Vitamin B12 doesn't mention anything about potentially lethal effects. But most energy drinks here have a level of caffein similar to a cup of coffee anyway (according to the can)... Nil Einne (talk) 16:49, 15 March 2008 (UTC)[reply]

Hair loss (non-balding)

Hi. This is not a request for medical advice. I think I've heard somewhere that the average adolescent loses roughly 120 hairs a day. Is this true? We really should have an article on hair loss that does not redirect to balness, then. Is that why when I drop food on the carpet it gets covered with hair even if it has been vacuumed? When does hair loss of more than one hair a day usually begin at, then? On average, when is the first white, gray, or silver hair occur in an average person (head)? Do people lose skin hairs too (not caused by accidental pulling)? This is not homework. On average, approximately how many hairs would have been lost (on a person that is not born will to be bald) and how many white/gray/silver hairs would have developed by the time an average person turns 18, for example? Do common mammalian pets (eg. cats, dogs, rabbits, hamsters, gerbils, mice, rats, etc) lose hair as rapidly as humans or more rapidly? What colour of hair loses hair most rapidly and which least rapidly? What about de-melanined hairs? Thanks. ~AH1(TCU) 00:49, 10 March 2008 (UTC)[reply]

These links should help you with your questions: [2],[3], [4]. Wisdom89 (T / C) 00:54, 10 March 2008 (UTC)[reply]
Anyone who has ever owned a cat will tell you how rapidly and in vast quantities they lose hair. As for the carpet, it's probably a mixture of hair which wasn't picked up by the vacuum, carpet fibres and other random dust and detritus. -mattbuck (Talk) 01:02, 10 March 2008 (UTC)[reply]

What is naphtha?

Is it really the same thing as petroleum ether? If so, why aren't the articles merged? If not, what is the difference? What is the main meaning of naphtha when the term is used in modern English? --Sonjaaa (talk) 00:55, 10 March 2008 (UTC)[reply]

Petroleum ether is obtained from petroleum refineries as the portion of the distillate which is intermediate between the lighter naphtha and the heavier kerosene says the PE article, while Naphtha says it is referred to as PE. My guess is that naphtha is not the same, but that there is a blurred boundary. -mattbuck (Talk) 01:00, 10 March 2008 (UTC)[reply]
All the terms used by the petro-chemical industry for the various fractions produced by oil refineries are ill-defined chemically and always include a large range of alkanes which overlap quite considerably with neighbouring fractions. It is more of a relative order of fractions up the distillation column which defines the terms. SpinningSpark 08:02, 10 March 2008 (UTC)[reply]

Question

Why do scientists use animals' Latin names? 58.168.209.250 (talk) 01:17, 10 March 2008 (UTC)[reply]

Biological_nomenclature#Value_of_binomial_nomenclature --Sonjaaa (talk) 01:20, 10 March 2008 (UTC)[reply]
That doesn't answer why Latin in particular, or the historical reasons for it. The simple answer is that Latin used to be the lingua franca of science—the "neutral" language that scientists of all countries used when they wanted to communicate with one another (though by the 19th century it was French and German; today it is English). It was expected that all educated men (and it was primarily men who were allowed to participate in the republic of letters for a good long time) would know the language, whereas whether they would know English, French, Russian, Italian, Swedish, etc. was less likely. Thus a large number of the early major scientific publications were originally written in Latin as well—De revolutionibus orbium coelestium, Philosophiæ Naturalis Principia Mathematica, etc.
The practice of using Latin specifically for binomial nomenclature (those animal names you are thinking of: genus plus species, Homo + sapiens) goes back to Linneaus, the father of modern taxonomy, and has stuck every since. Standardizing nomenclature in a neutral, non-changing language was important to making sure that a creature in one part of the world was really the same (or similar) creature in another, even though they had radically different names in the local languages. --98.217.18.109 (talk) 01:45, 10 March 2008 (UTC)[reply]
I've always enjoyed the fact that "Linnaeus" is itself a Latinized Swedish name. I suppose as "Carolus Linnaeus" it would be fully binomial. - Nunh-huh 01:42, 10 March 2008 (UTC)[reply]
Well, again, it's the language Linnaeus himself published his major works in. He wrote the whole thing in Latin, name included. He practiced what he preached! The custom of using only the Latinized last name for "major" writers is quite old (think also Copernicus). Scientists already drop the Albert from Einstein quite often; imagine how that'll progress in two hundred years. --98.217.18.109 (talk) 01:45, 10 March 2008 (UTC)[reply]
Yes, it's not that he was unusual in adopting a Latin name (Paracelsus, Helvetius, etc.) but that it was in his case an amusing coincidence. - Nunh-huh 02:05, 10 March 2008 (UTC)[reply]
There is a second reason Latin was used - it is a "dead" language. It is not changing due to modern usage. So, the definition of a Latin word 100 years ago is the same definition today and will be the same definition 100 years from now. English (or any other modern language) doesn't have that luxury. Consider "urchin". What is that? Easy, a nasty street kid. Or, perhaps it is one of those prickly things in the ocean. Not long ago, it was a prickly little animal in British gardens. Before that, it was simply a vulgarity. Who knows what it will mean in another 100 years. It is a good thing we don't use it for scientific purposes since it isn't nice enough to pick one definition and stick with it. -- kainaw 17:04, 10 March 2008 (UTC)[reply]
Latin is mostly dead. But it is the official language of the Vatican, and that state's panel of Latin experts do formulate new words and constructions for things like "helicopter" and "hard disk" that the Romans knew not of. There are (or was) also available on CD the Elvis Presley classics, "Tenere me ama" ("Love Me Tender") and "Nunc hic aut humquaum" ("It's Now or Never"). - Nunh-huh 23:47, 10 March 2008 (UTC)[reply]
One big advantage of standarized bionomials: it's great for organization without the level of ambiguity and repetition of common names. Common names are often misleading (flying fox), used for a wide variety of different species (razor clam), or there may be several different names for a singe animal (cougar/puma/mountain lion/panther). A single, agreed-upon term helps everyone know just what animal we're talking about. — Scientizzle 20:19, 12 March 2008 (UTC)[reply]

Many people claim that houseplants "cleanse the air", by removing particulates and carbon dioxide and adding oxygen. While this is certainly true to some degree, I'm of the opinion that the quantity of plants needed to do a significant amount of this would be far more than would fit in the home, based on plants having much slower metabolisms than people. So, I'd expect that if the biomass of people and pets in your home is 1000 lb, that you would need maybe 10,000 or 100,000 lb of plants to clean the air of the carbon dioxide and other waste products the people create. How can I calculate the actual ratio ? StuRat (talk) 01:24, 10 March 2008 (UTC)[reply]

Well, this would take some research - if you can come up with the amount of CO2 used per day by plants, we could do a quick and dirty calculation. On the human side: average CO2 content in inhaled air is roughly 383 ppm by volume, which is 0.0383%.
Average CO2 content in exhaled air is 4.5% by volume. The difference (4.5% minus 0.0383%) would be 4.1%. So the CO2 output from a human is 4.1% of the volume of inhaled (or exhaled) air.
The tidal volume (volume of a normal breath) of a human is about 500 mL. The usual respiratory rate varies between 12 to 20 breaths per minute. For our purposes, let's say 18 breaths per minute.
There are 1440 minutes in a day, or 25920 breaths per day. 25920 breaths of 500 mL each = 12960000 mL = 12960 liters per day. 4.1% of 12960 liters is 531 liters of CO2 produced per human per day. There are 22.4 liters/mole of gas at STP, so 531 liters is 23.7 moles. There are 44 g/mole of CO2, so 23.7 moles = 1043 grams of CO2, or roughly one kilogram, produced per human per day.
I think :) - Nunh-huh 02:02, 10 March 2008 (UTC)[reply]
4.5% minus 0.0383% is 4.46%, not 4.1%. Oh, how you curse us, Mr. Decimal Point.-RunningOnBrains 21:13, 12 March 2008 (UTC)[reply]
Good work, now we need some info on the plant side. StuRat (talk) 03:15, 10 March 2008 (UTC)[reply]
You also need to account for the efficiency of the plant at converting CO2 to oxygen. I have read that plants are extremely efficient (95%[5]) at using light energy for this process. So even if plant metabolism is slower, it may be compensated by a more efficient CO2 to O2 conversion than a human's ability to produce CO2 from O2.
The amount of oxygen produced also depends on the light available. I had an aquarium containing hydrilla plants. When the sun shone on them, streams of oxygen bubbles would spew forth from the leaves. =Axlq 02:09, 10 March 2008 (UTC)[reply]
23.7 moles of carbon (C) is 284g, so the total carbon content of plant mass in your house must increase by 284g per day per human. If we assume that plants are reprented by the formula CH2O (sugars) it is 948g per person per day, but there are also phosphates and nitrogen that the plant takes in, so 1kg per day would be minimum...Which looks very unrealistic.--Shniken1 (talk) 02:31, 10 March 2008 (UTC)[reply]
Interesting approach. Aren't plants something like 95% water ? I wonder what percentage of their weight is carbon. If we figure 4% by weight, then one kilogram of carbon increase would mean 25 kilograms of plant mass increase. StuRat (talk) 03:15, 10 March 2008 (UTC)[reply]
Yeah water would make up significant amount of the mass of the plant so yeah the mass of the plant as a whole would have to increase a lot more than its carbon mass (depending on the plant), and as plants get bigger I would suppose that water would be harder to obtain (via transirpation) so it may be the limiting growth factor.--Shniken1 (talk) 03:52, 10 March 2008 (UTC)[reply]
I'd expect most of the water to be supplied by humans watering the plant, with the roots sucking it up from there. StuRat (talk) 23:55, 10 March 2008 (UTC)[reply]

Much of this depends on what you mean by cleanse the air. I would expect the impact on oxygen and carbon dioxide to be minimal unless you live in an air tight box. Transpiration is a likely reason for cleansing. Transpiration cools the air due to evaporation. It also adds moisture to the air. Both these could be beneficial inside a house. David D. (Talk) 18:55, 10 March 2008 (UTC)[reply]


Incidentally, the following link is in the "Biosphere 1, 2, 3, and J" section of our Biosphere article:

"Biosphere 3 (aka BIOS-3) - Experiment to be conducted starting Jan 2008 in the Gary C. Comer Geochemistry building at Columbia University's Lamont Campus in Palisades, New York."

However, when I follow the link, the article says:

"BIOS-3 was a closed ecosystem at the Institute of Biophysics in Krasnoyarsk, Siberia, in what was then the Soviet Union."

Which is correct ? StuRat (talk) 03:33, 10 March 2008 (UTC)[reply]

(after edit conflict) According to http://www.newton.dep.anl.gov/newton/askasci/1993/biology/bio027.htm, it takes around 353 plants to match the oxygen input rate that humans require. However, I have two concerns with the webpage:
(1) The webpage assumes that all oxygen breathed in that does not enter the inactive part of the lung (the "dead space") is absorbed. According to Breath#Composition_of_air, 15-18% of exhaled air is composed of oxygen by volume. For the 252 L/h of inhaled air that does not enter the dead space, and the corresponding 252 L/h that are exhaled, about 42 L is exhaled. Since 53 L/h of oxygen that does not enter the dead space is inhaled, this gives an hourly oxygen usage rate of 53-42=11 L/h. Assuming the plant calculations are correct, 76 plants would be needed.
(2) At the end of the webpage, the author states that "oxygen production decreases as carbon dioxide concentration increases". However, according to the photosynthesis article, the amount of CO2 plants take in is equal to the amount of O2 they output. Humans also necessarily expel as much CO2 as they breathe in oxygen, so if 76 plants provide enough oxygen to sustain a human, they also absorb enough carbon dioxide to keep the environment's CO2 level constant.
So 76 plants per person, and if we assume plants are 500 g on average, that's 38 kg/person. The plants would have to be properly maintained, of course, and cannot be allowed to grow; growth would mean a higher demand for CO2 than the humans can supply.
About BIOS-3: Most sources state that it was an experiment in the Soviet Union: [6][7][8]. Searching for "bios 3 'Gary C. Comer'" on Google yields only the Wikipedia biosphere article and mirrors of it. --Bowlhover 04:06, 10 March 2008 (UTC)
That seems like way too much oxygen production per plant kg. After all, a person weighs around 38 kg, and I have a hard time accepting that plants, with their far slower metabolisms, use CO2 at the same rate people use oxygen. Also, if the plant doesn't grow, where exactly does all the carbon go that it sucks out of the air ? StuRat (talk) 00:03, 11 March 2008 (UTC)[reply]
While it might matter for a biosphere, for your purposes as I understand them (the plant cleans up after the humans), it doesn't matter whether or not the plants grow. - Nunh-huh 00:44, 11 March 2008 (UTC)[reply]
But where does the absorbed carbon go if not to plant growth ? I suppose a fruit tree puts it into fruit and a nut tree into nuts (although this is a type of growth), but what about plants that don't produce food ? StuRat (talk) 17:29, 11 March 2008 (UTC)[reply]
List of air-filtering soil and plants may be of interest too. —Pengo 09:30, 11 March 2008 (UTC)[reply]
According to http://www.bio.net/bionet/mm/plantbio/2000-October/024096.html, the average plant produces 15.2 mL of oxygen per dm^2 of leaf area per hour. The "leaves" mentioned in the previous link I gave would then be 10 dm^2 each! Anyways, the total leaf area needed to support one person would be 750 dm^2, about 2.7 m x 2.7 m. I wonder how much that would weigh.
Also, the plants will grow, but I think they have to be trimmed so they don't use up CO2 at an excessive rate. --Bowlhover 15:18, 12 March 2008 (UTC)
Yes, that sounds more reasonable. Perhaps the leaves alone might weigh 38 kg, but including the stalks, branches, roots, etc., would make it much more. StuRat (talk) 17:01, 13 March 2008 (UTC)[reply]

We have had a similar question before. What if the person consumed plant material each day in the form of carbohydrates to total a significant portion of his daily caloric needs? Could the carbon in the plant material (carbohydrates) equal the carbon in the exhaled carbon dioxide the person emits? If the person exhaled more carbon than he consumed, wouldn't he become carbon deficient over time? Then there would be no net increase in the amount of carbon sequestered in the plant material. Ever see a demonstration of a sealed glass globe with water, plants, little fish and snails? The manage to keep recycling for quite a while sometimes until there is too much/too litttle sunlight or something else goes out of balance. Edison (talk) 14:04, 11 March 2008 (UTC)[reply]

I would expect that most of the carbon people eat ends up in their feces or urine. I'd expect animals would be needed to get this carbon back in circulation from there. Worms or insects can eat the feces, birds and frogs can eat them, and other animals can eat them, until we get to something people eat. StuRat (talk) 17:29, 11 March 2008 (UTC)[reply]
Some people might consume carbon in the forms of pencil graphite, diamonds, or charcoal and poop it out, but that is a small portion of the carbon consumed by humans.Human feces does contain a large proportion of undigested food which would still have much of its carbon content. Urine does incorporate some carbon from the carbon dioxide resulting from metabolism of food. Most carbon consumed is probably in the form of Carbohydrates, which are transformed along with some non-carbohydrates into Glucose by digestion, so that the cells of the body can use it as fuel. Glucose is (C6H12O6). It yields energy and exits the body as water and carbon dioxide. Long term space travel will likely involve a closed system of recycling the products of human and animal metabolism into new food such as the Biosphere 2 experiment tried to test. Edison (talk) 16:47, 12 March 2008 (UTC)[reply]

How can you ever fall in a black hole?

Since time slows per the reference frame of an outside observer as an object approaches the event horizon, and 'freezes' at the horizon (thus the Russian term for black hole: 'frozen star'), how can a black hole ever increase in mass? This was once considered a flaw in the concept of black holes, but evidently was somehow resolved. However, I've never seen an account of how it was resolved. (And yes, I do understand that for an observer falling past the event horizon, supposedly nothing unusual would appear to happen - except that you'd think s/he'd witness the end of the universe.) — kwami (talk) 07:03, 10 March 2008 (UTC)[reply]

This question has been previously answered here [9] and probably many times before. However, I never felt satisfied with the answer given then, for the same reason given by Kwamikagami. If someone can elaborate, please do. SpinningSpark 08:15, 10 March 2008 (UTC)[reply]
I’m no physicist. But as the mass of the black hole increases, the event horizon will increase in radius and objects that previously were hovering on the edge will be enveloped. And objects don’t need to actually be at the center of the singularity to contribute to the black hole’s mass/gravitational effect. (This is just my speculation.) — Knowledge Seeker 08:33, 10 March 2008 (UTC)[reply]
But the BH can't increase in mass unless something crosses the event horizon in finite time in the external reference frame. — kwami (talk) 08:46, 10 March 2008 (UTC)[reply]
I’m not sure I agree with that statement. — Knowledge Seeker 08:51, 10 March 2008 (UTC)[reply]
How would it increase in mass, without mass being added to it? — kwami (talk) 08:54, 10 March 2008 (UTC)[reply]
Perhaps I should be more clear. I contend that a mass placed just outside the event horizon will increase the gravitational field/curvature of space (as it would anywhere) and that therefore a small volume of space which previously had almost, but not quite enough gravity to have an escape velocity greater than c (or equivalent formulation) now will; in effect, the combined event horizon around the two objects will be larger. (Again, speculation.) — Knowledge Seeker 08:57, 10 March 2008 (UTC)[reply]

A group of massive objects, act as if all their gravity were concentrated at thier center of mass. So from a distant observer's POV anything frozen at the edge of an event horizon would still contribute to the total mass of the black hole system. Does that resolve the problem? Theresa Knott | The otter sank 12:28, 10 March 2008 (UTC)[reply]

Does this mean that an unfortunate astronaut just outside the event horizon might be observed by a "local" observer (say, another astronaut also falling into the black hole) as not having quite reached the horizon, but that both might be "observed" by a distant observer to be within the event horizon (i.e. part of the black hole)? dbfirs 12:54, 10 March 2008 (UTC)[reply]
Hi. On a Discover magazine there's this guy named João Magueijo that has a theory called Varying Speed of Light (VSL). When he introduced this theory, one physicist said it really stood for "Very SiLly". However, according to this theory, time and the speed of light slow near a black hole and never allow anything to escape nor go in (after all, black holes are infinitely dense). Hope this helps. Thanks. ~AH1(TCU) 14:22, 10 March 2008 (UTC)[reply]
As far as I know Magueijo's VSL theory is supposed to be an alternative to cosmic inflation, so it only matters in the very early universe and doesn't have any bearing on (present-era) black holes. -- BenRG (talk) 21:33, 10 March 2008 (UTC)[reply]

Objects can and do fall in. They only appear to stop at the event horizon. 64.236.121.129 (talk) 16:43, 10 March 2008 (UTC)[reply]

The objects may see themselves falling in. An outside observer sees the objects dim and disappear as they approach the Schwartzchild radius. An alternate theory Gravastar suggests that matter falling into a compact stellar object will appear to cool and form a Bose-Einstein condensate. One consequence of general relativity is that perception is affected by perspective. Jehochman Talk 16:51, 10 March 2008 (UTC)[reply]
64 and Jehochman, it’s not that Kwami’s really disputing whether they fall in or not (the phrasing of the heading, unfortunately, is misleading). His question is (as I understand it), essentially: If, from the reference frame of an outside observer, an object never passes the event horizon [due to time dilation], how/when will he observe an increase in the black hole’s mass?Knowledge Seeker 18:11, 10 March 2008 (UTC)[reply]
If matter has fallen inside the event horizon, how can you get back any information about mass inside? If light can't get out, neither can any other sort of information. If an outside observer can never see anything cross the event horizon, then presumably from their point of view, nothing ever has. A sphere or shell of mass around a point will be indistinguishable to an outside observer from all that mass being located at the center point. There would be no observable difference on the force of gravity affecting an outside observer. There are a lot of contradictions if we assume the existence of black holes. This may mean that our black hole model is wrong. Jehochman Talk 18:16, 10 March 2008 (UTC)[reply]

I don't know that much about black hole dynamics, but I think Knowledge Seeker is correct. Certainly it's a mistake to think that infalling matter has to fall through the event horizon to add to the mass, because nothing that happens inside the event horizon can be relevant to physics outside. Whatever it means for "the black hole to gain mass", it has to involve only the physics outside the event horizon.

The event horizon is acausal; it will happily expand faster than light to engulf infalling matter. Here's a special relativistic analogy. I'll use 2+1 dimensional Minkowski space because it's easier to visualize than 3+1 dimensions and 1+1 isn't enough for this example. At time t = 0 pick some region of the xy plane and magically destroy everything there. There's then a region of spacetime (a collection of events) with t < 0 which is unobservable to anyone who survives past t = 0, because any signal that might have originated from those events was destroyed. For example, say you destroy everything in the circular region x2 + y2 < 1 light year2. Then the unobservable region is a cone in Minkowski space whose apex is x = 0, y = 0, t = −1 year and whose base is the destroyed disc. If you destroy a square (|x| < 1 ly, |y| < 1 ly) then the unobservable region is a square pyramid. The boundary of the unobservable region is the event horizon. The event horizon is always a null surface because of how it was defined, but it may have "creases" which are not null. For example, in the square pyramid case you have corners which go from x = y = 0 at time t = −1 year to x = ±1 ly, y = ±1 ly at time t = 0; they're effectively moving at c√2.

Let S be the destroyed set. Then the observable region at time −t is , where Sc is the complement of S and Br(x) is the ball of radius r centered at x. Imagine S being eroded away from all sides at the speed of light as you go backward in time. Now say S is a sort of peanut shape, or an overlapping union of two circles. As you go back in time it will become a thinner peanut; then it will break into two teardrop shapes, with the pointy bits pointing toward one another; then the teardrops will shrink and become more circular until they disappear. Now run this forward in time: two expanding teardrops appear out of nowhere, extend toward each other faster than light, and merge. That's kind of what a black hole merger is like. The event horizon is not at all like a physical object obeying dynamical laws; it's a global property of the whole spacetime and it "knows about the future". In fact it's a theorem that the event horizons of two black holes that will eventually merge have the teardrop point on them from the beginning. That's for merging black holes, i.e. tossing one black hole into another, but I think tossing ordinary matter into a black hole works similarly. Whether any of this answers the original question I'm not entirely sure. This is a tricky subject. -- BenRG (talk) 21:33, 10 March 2008 (UTC)[reply]

I wonder if Knowledge Seeker might be on the right track: Once an object approaches a black hole close enough that their center of mass lies within the EH, does the EH then expand correspondingly? But this would happen while a small mass is still quite far from the EH, so it wouldn't be engulfed. (If you were to drop a sandwich into a Solar-mass black hole, it would hardly affect the EH at all, so we're left with my original question.)
It's not true that we can't know what's within the EH, only that no signal can be sent out. Fields extend beyond the EH, but any modulations of those fields would be red-shifted to a frequency of zero. However, with those fields we can still measure the total electronic charge, mass, and momentum (linear and angular). — kwami (talk) 22:23, 10 March 2008 (UTC)[reply]
Also, pace another comment, it isn't the case that the object just appears to slow down, due to some kind of optical illusion. It isn't light that slows down, but time itself. In the external reference frame, it really doesn't reach the EH. Thus the problem. — kwami (talk) 22:50, 10 March 2008 (UTC)[reply]

Intermolecular Forces

Can a molecule have hydrogen bonding, permanent dipole - permanent dipole and instantaneous dipole - induced dipole intermolecular forces acting upon it at any one time? 88.108.198.114 (talk) 17:38, 10 March 2008 (UTC) —Preceding unsigned comment added by 88.108.198.114 (talk) 17:37, 10 March 2008 (UTC)[reply]

Yes. something like liquid cholesterol would be a easy example to consider87.102.94.48 (talk) 17:45, 10 March 2008 (UTC)[reply]
True. Remember that molecules can be very very large (Protein), or even macroscopic (DNA), so different parts of one molecule can have completely different properties. --Bmk (talk) 17:59, 10 March 2008 (UTC)[reply]

Batteries in series vs. parallel arrangement

If one battery in a set is bad (low voltage), which arrangement is affected more? —Preceding unsigned comment added by 66.120.95.52 (talk) 19:22, 10 March 2008 (UTC)[reply]

Series. Voltages in series add, so a bad one will lower the voltage. In parallel, the remaining good batteries will try to supply whatever current it takes backwards through the bad battery to drop the supply voltage across it. If they can't supply enough current to do that, the voltage will fall, but if they can it won't, much. So a series arrangement is always immediately affected, and a parallel arrangement might not be affected very much at first. --Milkbreath (talk) 19:54, 10 March 2008 (UTC)[reply]
Could be either. If the one with low voltage is that way because the internal resistance has increased (low voltage under load, closer to normal voltage with no load, a common enough condition) then the parallel arrangement would work better, although the bad cell should be disconnected to avoid premature drainige of the others and because it is not contributing much to the output. If the bad cell has an internal short (happens sometimes) then it would drain the output of the others in parallel but would have less effect in series. In either arrangement the bad cell could explode. Edison (talk) 13:56, 11 March 2008 (UTC)[reply]
He asked about the case where one battery has low voltage, not where one battery is a dead short. "Could be either" is the answer to a different question, and is not a sensible answer to the question of "which" is affected "more". If it depended on the situation, "neither" or "it depends" would make sense. --Milkbreath (talk) 19:33, 11 March 2008 (UTC)[reply]

What bodily function is not possibel in space?

what bodily function is not possibel in space? —Preceding unsigned comment added by 89.243.182.3 (talk) 20:13, 10 March 2008 (UTC) - Please don't post in all caps, it comes off as yelling -- MacAddct  1984 (talk &#149; contribs)[reply]

pushing the Caps Lock key a second time? Thomprod (talk) 20:14, 10 March 2008 (UTC)[reply]
If you mean in a spaceship, all bodily functions are possible, but can get a bit messy. That's why they have special devices for going to the bathroom. -- MacAddct  1984 (talk &#149; contribs) 20:28, 10 March 2008 (UTC)[reply]
It hasn't been demonstrated that a child can grow in the womb in space. I'm not sure what people have thought about this though.Sancho 20:39, 10 March 2008 (UTC)[reply]
Larry Niven made a significant plot point out of this in his Known Space series. Pregnant women who lived in the Belt were required to live in a hollowed-out asteroid, called Confinement Asteroid, that was spun to provide centrifugal force in place of gravity. --Trovatore (talk) 20:53, 10 March 2008 (UTC)[reply]
Did they have another hollowed-out asteroid to send women with PMS ? :-) StuRat (talk) 17:35, 11 March 2008 (UTC)[reply]
Bodily functions remain intact, however, there can be serious muscular atrophy due to zero gravity unless the astronaut exercises regularly. Wisdom89 (T / C)
I think it's been demonstrated for mice though. IIRC, they were born a bit smaller than normal but otherwise okay. DMacks (talk) 02:49, 11 March 2008 (UTC)[reply]
According to Canadian astronaut Chris Hadfield you can't burp normally in a weightless environment - see Q2 here. Gandalf61 (talk) 09:49, 11 March 2008 (UTC)[reply]
Have anyone heard about sex in space? Considering that the blood does not flow normally in space and some parts of our body will need it, perhaps it is not possible... —Preceding unsigned comment added by 217.168.3.246 (talk) 03:03, 13 March 2008 (UTC)[reply]
Blood flows due to heartbeats and one-way valves, even in zero gravity. However, it does stop flowing at high g's, such as those experienced by a fighter pilot. A special suit (I forget the name) can help in this situation, by alternately compressing and releasing on the legs to push the blood forward. I believe sex would work just fine in space, although both parties would want to be contained in an absorbent bag, so that any fluids released don't go floating off down the hall. As a practical matter, though, the lack of privacy and not wanting to use up precious water to clean up afterwards make this a no-no for short space flights. Long-term or even multigenerational ships would need to find a way to make this work, however. I suspect that artificial gravity, via a rotating spaceship, would be used in such a situation, as some gravity seems important in maintaining the human body over the long-term. StuRat (talk) 15:16, 13 March 2008 (UTC)[reply]
Wikipedia knows all - see sex in space. Gandalf61 (talk) 15:24, 13 March 2008 (UTC)[reply]

One man's meat is another man's poison

Are there any spices with an unusually small difference between the standard amount used for flavoring and the minimum poisonous amount? Or with an unusually large difference between the LDLO and the LD50? --67.185.172.158 (talk) 21:11, 10 March 2008 (UTC)[reply]

I seem to rememeber that 2 nutmegs could be fatal...87.102.94.48 (talk) 22:21, 10 March 2008 (UTC)[reply]
I had to look these up: LDLo = Lowest published lethal dose and LD50 = Median lethal dose. Mmmm, Eggnog with extra nutmeg! :-)) --hydnjo talk 01:51, 11 March 2008 (UTC)[reply]

Deadly meat - see fugu. 80.0.108.245 (talk) 19:32, 16 March 2008 (UTC)[reply]

Molecular Excitement

I know that molecular excitement (vibrations, oscillations - what is the correct term?) slows down as Temperature decreases - I'm thinking of a liquified gas in a tank - and increases as the Temperature rises, but what happens when the Pressure increases or decreases? What effect does Pressure have on Molecular Excitement?

P. Lamont —Preceding unsigned comment added by 72.39.249.249 (talk) 21:18, 10 March 2008 (UTC)[reply]

Temperature has the greatest effect on molecular rotation, pressure also influences this, through pressure broadening.--Shniken1 (talk) 22:54, 10 March 2008 (UTC)[reply]
Pressure can change the temperature, if you increase pressure, the substance is likely to contract, and work is done on it. This will add energy, and the temperature will increase. This head can be conducted out. Also if pressure increased so that the material is compressed to a white dwarf density, then the uncertanity principle will ensure a uncertain temperature and Degenerate matter exists. Graeme Bartlett (talk) 23:15, 10 March 2008 (UTC)[reply]


March 11

type 2 diabetes and hair loss

This question has been removed. Per the reference desk guidelines, the reference desk is not an appropriate place to request medical, legal or other professional advice, including any kind of medical diagnosis, prognosis, or treatment recommendations. For such advice, please see a qualified professional. If you don't believe this is such a request, please explain what you meant to ask, either here or on the Reference Desk's talk page.
This question has been removed. Per the reference desk guidelines, the reference desk is not an appropriate place to request medical, legal or other professional advice, including any kind of medical diagnosis or prognosis, or treatment recommendations. For such advice, please see a qualified professional. If you don't believe this is such a request, please explain what you meant to ask, either here or on the Reference Desk's talk page. --~~~~
--Milkbreath (talk) 01:54, 11 March 2008 (UTC)[reply]
Seriously, you need to consult a doctor about this - we can't offer medical advice of any kind... Wisdom89 (T / C) 01:47, 11 March 2008 (UTC)[reply]

Electron configuration in Lawrencium

Lawrencium states [Rn] 5f14 6d1 7s2, while Atomic orbitals#Orbitals table says that "This table shows all orbital configurations up to 7s, therefore it covers the simple electronic configuration for all elements from the periodic table up to Ununbium (element 112) with the exception of Lawrencium (element 103), which would require a 7p orbital."

One of these pages is wrong, is it the atomic orbitals page? Also the next electrons in [Rn] 5f14 6d1 7s2 go into the 6d orbital not 7p... How did the atomic orbital page come up with this statement?

PS: "Simplest electronic state" = ground state? --Shniken1 (talk) 03:17, 11 March 2008 (UTC)[reply]

re: " "Simplest electronic state" = ground state? " - you might think so - but it doesn't have to be the case - because 'simplest' doesn't have much meaning in this context.87.102.14.194 (talk) 08:54, 11 March 2008 (UTC)[reply]
It shouldn't be different here, electron configurations of elements are always given in their ground state, simplest configuration may refer to filling the d orbitals before the next s orbital but that would still leave lawrencium without an 7p electons.--Shniken1 (talk) 10:08, 11 March 2008 (UTC)[reply]

elements

I came across this in a magazine 1. element present in all organic compounds 2.alkane hydrocarbon with straight chain 3.heaviest alkaline earth metal 4.produced by combustion

it seems that 1 and 2 are not elements. but what are the answers? juz curious. not homework.

Uh huh. Anyway, this is an encyclopedia, and you can find answers to simple questions just by typing the relevant word into the search bar. But I'll give you the links right here: Organic compound makes the answer pretty obvious; there are many alkane hydrocarbons that have straight chains; alkaline earth metal, also makes it pretty obvious; combustion, the very first chemical equation in the article gives it away, and I'm guessing you're homework magazine is only concerned with the combustion of hydrocarbons, so that one is all you need. Someguy1221 (talk) 08:39, 11 March 2008 (UTC)[reply]
1. try organic compound
2. read alkane see subsection linear alkanes
3. read alkaline earth metal
4. read combustion87.102.14.194 (talk) 08:52, 11 March 2008 (UTC)[reply]
Has the Physics magazine guy gone to chemistry class now? bibliomaniac15 00:22, 12 March 2008 (UTC)[reply]

The Speed Of Dark......?

I was asked a question recently that sounds nonsense but may have some validity in it. "If light travels at the speed of light, what does dark travel at?" We're told light travels and dark is the expulsion of light, but does it travel? and, if so, at what speed? ajcree —Preceding unsigned comment added by 89.241.133.221 (talk) 08:29, 11 March 2008 (UTC)[reply]

Dark is the absence of light, and nothing more. If it takes t seconds for light from a bulb to reach the wall of your room after lighting up, then it takes those same t seconds for the wall to darken after the bulb dims. Someguy1221 (talk) 08:33, 11 March 2008 (UTC)[reply]
Exactly, so if a star suddenly shut off that it 50 light-years away from us, it would take us 50 years to see that the star had stopped shining. Hope this helps. Thanks. ~AH1(TCU) 16:03, 11 March 2008 (UTC)[reply]
One complication is that objects rarely stop giving off light instantly. The filament from an incandescent light bulb takes a portion of a second to cool to a temp that no longer produces visible light, for example. A star which has ceased to undergo nuclear fusion may take millions of years to cool to the point where it no longer gives off light. StuRat (talk) 17:40, 11 March 2008 (UTC)[reply]
Dark "travels" at the same speed as light because light is just photons, which are perceived using receptors in our eyes. When there are no more photons, we see darkness. Since photons travel at the speed of light by definition, the absence of photons behind them (i.e. dark) expands at the same speed. It's like dropping a stone in a pond: your question is equivalent to asking, "What speed does the trough between the ripples travel at?" Obviously, it travels at the same speed as the ripples themselves! « Aaron Rotenberg « Talk « 09:01, 13 March 2008 (UTC)[reply]

Why do Rabbits hop?

The question is pretty self-explanatory, but please don't answer "because that's how their legs work"! I know that, my question is why do rabbits hop instead of walk like most other mammals. 71.57.90.83 (talk) 08:47, 11 March 2008 (UTC)[reply]

I'd say it's about propulsion and desired speed, and like it or not the structure of both sets of legs being very unlike "most other mammals". Only Bugs Bunny seems to saunter, stroll, tap dance and so on as an upright. Similar to kangaroos (well, not quite), rabbits don't hop – as in having all four peds off the ground – if they are mooching about eating. And as prey animals of that kind, when they need to speed at a moment's notice, the bounce comes into it due to the configuration of the hind legs. Does that help?Julia Rossi (talk) 10:32, 11 March 2008 (UTC)[reply]
Consider a similar animal without the long hoppy back legs - ie a guinea pig if you've ever seen one run you might wonder why they weren't long ago predated out of existence... 87.102.14.194 (talk) 10:42, 11 March 2008 (UTC)[reply]
Interesting point – it probably doesn't exist in the wild any longer. Julia Rossi (talk) 11:11, 11 March 2008 (UTC)[reply]
The article sort of confirms that - seems they use safety in numbers - including random scattering when a predator approaches.. In general though - in a pack of 100 even a 100% efficient predator will only make one kill before stopping I would guess - all that remains is for the creatures to breed sufficiently fast to cover the predation... Poor little things.87.102.14.194 (talk) 12:16, 11 March 2008 (UTC)[reply]
Like when you're being shot at: think zigzag. It's a good way to confuse and loose a predator. 200.127.59.151 (talk) 14:20, 11 March 2008 (UTC)[reply]
...which is useful if the predator is tight on your tail. StuRat (talk) 17:45, 11 March 2008 (UTC)[reply]
I would think that the rabbit's hop is a result of what sort of animals prey upon it. The fox and the hawk come to mind. These are both very quick animals, so the rabbit must also be quick. Its large back legs enable the rabbit to spring forth in an instant, while a more controlled gait would see it end up as lunch more often than not. Vranak (talk) 15:29, 11 March 2008 (UTC)[reply]
Well, if we're just throwing out conjecture, it seems to me that hopping would have evolved in an open environment, grassland maybe, where you won't get snagged in the brush mid-hop or bean yourself on a branch. It is a quickness thing, isn't it? It would have the additional advantage that it would carry you above the grass with each hop, so you could get your bearings. Lots of evolutionary adaptations seem odd, and perhaps this one initially evolved especially to meet the requirements of some small niche, and when the animal spread out it took the form of locomotion with it, retaining it with refinements. It certainly is one good way to make tracks, strange as it seems. --Milkbreath (talk) 19:51, 11 March 2008 (UTC)[reply]
Animals also make do with what they've already got. The ancestor of rabbits may have had legs adapted for something else entirely that turned out to be easy to adapt to hopping and zigzagging, which proved to be a very effective escape. Some gazelle have also evolved a leaping zigzagging flight, but since they started out with a different form of locomotion, they ended up with a somewhat different result. — kwami (talk) 01:05, 12 March 2008 (UTC)[reply]
Why not some OR? Wandering in a wild field in the evening, spotting young bunnies way off, in grasses up to my hips, when a big (medium dog size) rabbit ran past me parallel to the ground (no arching bounces for reconnaissance) very fast and low. And straight. So those back legs have a strong thrust forward when needed, not just the up and down thing. I'm sure it would have zigzagged if it was being chased. Julia Rossi (talk) 03:46, 12 March 2008 (UTC)[reply]
Yes, two different strategies can be employed depending on if the predator is "out of range". If so, running straight and fast to safety makes sense. If the predator would catch the prey before they can reach safety, however, then taking evasive action is the best plan. Zigzagging side to side, hopping up and down, and suddenly reversing direction are all methods that offer some hope that the prey can outmaneuver the predator. Similar methods can be employed by ships, submarines, or planes which are being hunted. StuRat (talk) 14:51, 13 March 2008 (UTC)[reply]

Perhaps rabbits hop for the same reason as the kangaroo - their legs etc act like springs so that they use less energy to travel. Some antelope-like animals - the springbok? - hop when a pedator is near to demonstrate that they are fit and that the predator would be wasting its time and energy to chase them. 80.0.108.245 (talk) 19:38, 16 March 2008 (UTC)[reply]

Interestingly, the rabbit / hare family (Leporidae) has its closest living relative in the pika family (Ochotonidae), which are terrestrial mammals (to my knowledge) not particularly prone to hopping. Together Leporidae and Ochotonidae make up the order Lagomorpha, and it's quite uncertain to whom this order is most closely related. Some data suggest relation to rodents, but other morphological evidence points to a more recent divergence from Marsupials -- male testes being located anterior of the penis, for example. If that were true -- and it's a big if -- then one can imagine hopping being inherited by both Peter and Roo from a common progenitor. As with all things in evolution, it is desirable that a new trait confers an advantage, but it really only needs to not confer a disadvantage to be passed on. See U Michigan's excellent taxonomy and diversity site, animaldiversity.ummz.umich.edu/site/accounts/classification/Lagomorpha.html for more information. Vance.mcpherson (talk) 16:55, 17 March 2008 (UTC)[reply]

What is this equation?

Edit (hit enter before asking question!) What is this eqation meant to represent? [10] "Pe + (Pe)r" not NaCl....:D --Shniken1 (talk) 10:19, 11 March 2008 (UTC)[reply]

It says 'Pepper' - otherwise it seems meaningless87.102.14.194 (talk) 10:30, 11 March 2008 (UTC)[reply]
"Pepper, not Salt." Someone likes puns. JohnAspinall (talk) 18:27, 11 March 2008 (UTC)[reply]

Why not swing?

Reflecting on the question about rabbits in motion, it set me wondering how come, as descendants of great apes now walking upright, humans don't swing from things as a means of locomotion? Julia Rossi (talk) 10:42, 11 March 2008 (UTC)[reply]

Lack of trees.. imagine if we did though .. we'd have to build adventure playgrounds instead of footpaths.. Seriously though we never would have left the forest.87.102.14.194 (talk) 11:05, 11 March 2008 (UTC)[reply]
First, humans are not descendants of great apes - we are great apes, because we are members of the biological family Hominidae (along with chimapnzees, gorillas and orangutans). And not all of the other great apes swing from trees - gorillas are ground dwelling. As the contributor above says, it is a matter of different species adapting to different environments. The same thing happens in the squirrel family, where we see both tree squirrels and ground squirrels. What exactly caused humans to acquire bipedalism is still a matter of research and debate - our bipedalism article says "There are at least twelve distinct hypotheses as to how and why bipedalism evolved in humans". Gandalf61 (talk) 12:37, 11 March 2008 (UTC)[reply]
I'd go with the idea that as prehumans moved out onto prairie, which lacked trees, they needed to develop an efficient method of locomotion for that environment. Since many primates can stand on hind legs for short periods, it wasn't too much of a leap to develop full-time bipedal motion. While less efficient than quadruped motion, bipeds had the huge advantage of having their hands free to use tools and weapons, which the opposable thumb our primate ancestors developed for grasping tree limbs first made possible. The relatively high intelligence of primates also made it possible to put tools to good use. StuRat (talk) 17:57, 11 March 2008 (UTC)[reply]
But lots of primates have moved out onto the savanna, and they didn't go bipedal. Also, Lucy apparently wasn't much of a tool user, but she was bipedal. All these arguments about what makes us different from other apes - we lost our hair cuz it was hot, we stood on our hind legs to see further or to free our hands - remind me of speculations about the extinction of the dinosaurs back before the extra iridium was discovered in the K-T boundary layer: They all failed, because they didn't address why, for example, plankton should go extinct just because dinosaurs got constipated from flowers (yes, I think that was actual proposal!). — kwami (talk) 00:59, 12 March 2008 (UTC)[reply]
There were multiple niches for primates on the savannah, some of which were for bipedal primates. Being a quadruped would have other advantages, like being able to hide from predators and prey in tall grass and running more quickly. As for being a tool user, I expect that the earliest tools were just found objects, like sticks and rocks, so I wouldn't expect to find a collection of chipped flint tools nearby. StuRat (talk) 15:16, 12 March 2008 (UTC)[reply]
Back to the original question, humans have retained some ability to climb trees, which is why we have arches in our feet (to fit the tree trunk). StuRat (talk) 17:57, 11 March 2008 (UTC)[reply]
I have to say, I'm extremely dubious of the "arches are for climbing trees" idea. Arches of the foot are very important to humans being able to support their weight and to being able to be bipedal. --98.217.18.109 (talk) 23:48, 11 March 2008 (UTC)[reply]
In that case, the arches would go all the way across the foot, symetrically, not only on the inside edges of the feet. This form is perfect for grasping tree trunks. StuRat (talk) 03:42, 12 March 2008 (UTC)[reply]
Why don't we swing from things? Why should we? Only some of the great apes spend any time swinging from things, and even those spend a lot of time not swinging from things. When not around things to swing from, they don't swing, and get around just fine. Swinging is both difficult and dangerous; it's not an ideal form of locomotion. --98.217.18.109 (talk) 23:48, 11 March 2008 (UTC)[reply]
Given the ground dwelling great apes, how come we lost all that upper body strength belonging to the longer arm ratio that they retain? I'm wondering if maybe it had something to do with the homosapiens taking over from the neanderthals and using the hands for specialisation that swinging might otherwise have interfered with. (oops the third part was answered in the thread where it goes back on itself above - I missed that) Julia Rossi (talk) 03:34, 12 March 2008 (UTC)[reply]
I expect that it has to do with our weak backs. The spine was really not designed to be used as we use it, so can't support nearly as much weight in bipeds as in quadrupeds. This means our upper bodies had to be minimized, such as having shorter, less muscular arms. We also have thinner skulls than many of our quadruped primate friends. If I had to redesign the spine for bipeds, I'd take the nerves outside the spine into something like a notochord, and replace them with cartilage to try to hold the spine together. StuRat (talk) 15:24, 12 March 2008 (UTC)[reply]
Why would we invest in longer, stronger arms when that would interfere with walking and running? Shorter, less strong arms are more convenient for not swinging from tree to tree, so if it's more of an advantage to be walking and running on the savannah than swinging from tree to tree there is a disadvantage in investing energy in those tree-swinging arms. I don't really see what's to explain here :/ Skittle (talk) 18:29, 12 March 2008 (UTC)[reply]
  • Humans stick to the ground because we've grown too tall and heavy to swing from trees. Gorillas suffer the same problem. As you know from Newton's law of universal gravitation, there's an inverse-square relationship at play: . What that means is that the proverb "the bigger they come, the harder they fall" is literally true: a taller, heavier creature hits the earth with much, much more force if it falls. At a certain point, evolutionarily speaking, the behavior is too risky to be worth pursuing. --M@rēino 20:27, 12 March 2008 (UTC)[reply]


I'd just like to point out that some of us do swing. In fact, otherwise, it don't mean a thing. --Trovatore (talk) 20:50, 12 March 2008 (UTC)[reply]

astronomy

the stars in the sky seen from the UK are the same as from Australia?cypru÷90.193.58.40 (talk) 13:51, 11 March 2008 (UTC)alex[reply]

Nope. While there is some axial tilt to the Earth's rotation, the UK sees a great many stars well north of the ecliptic plane and Australia sees a great many well south of the same. Stars near the ecliptic will be visible to both places (though constellations will be inverted) but many regularly visible to one are never visible to the other. — Lomn 14:12, 11 March 2008 (UTC)[reply]
Two good examples would be the North Star and the Southern Cross. AlmostReadytoFly (talk) 14:34, 11 March 2008 (UTC)[reply]
Hi. A good indicator of which stars you can see is to take your latitude, positive if north and negative if south, then add 80 for northern limit and subtract 80 for southern limit. Well, that should be a good indicator of which stars you can see without too much interference from the atmosphere, light pollution, terrestial objects, haze, and all that. So, if for example the southern limit for a location in the UK is -25, then you can see Sirius easily because it is at -17 or so, oh and if you get a number like +120, subtract 120 - 90 to mean that you can see 30 degrees in declination beyond the north pole reigon near the pole star to the north, and similar methods for southern hemisphere. Hope this helps. Thanks. ~AH1(TCU) 16:10, 11 March 2008 (UTC)[reply]

Alcohol injection

Is it possible to survive when a small amount of alcohol is injected intravenously (for instance, a 40% vodka within an average syringe capacity, or less, if lethal)? IMO, one should break through because it's similar to drinking alcohol on an empty stomach. No one is experimenting here:) --85.132.14.38 (talk) 18:54, 11 March 2008 (UTC)[reply]

Injecting a small amount of ethanol and injecting a small amount of 40% vodka are two different things. If I had to choose, I'd choose the ethanol since I don't know what else might be in the vodka. ----Seans Potato Business 21:08, 11 March 2008 (UTC)[reply]
Steve-O did a whole IV bag of vodka intravenously (well, duh!) in one of his videos. It didn't kill him. That's not to say that you should try it at home (or at work, or in someone else's home, or anywhere else for that matter). --Kurt Shaped Box (talk) 00:21, 12 March 2008 (UTC)[reply]
Let's consider the math in the reverse direction. A blood alcohol level of >0.40% is usually fatal, so let's consider a BAC of 0.30% (the point where you'll generally lose consciousness). A typical adult male has about 5 litres of blood. So 0.30% of 5L is 15mL of alcohol. So, if you're using a typical 1mL, 3mL or 10mL syringe, then it looks like you'll be fine you may survive, even if it's full of pure ethanol. However, an IV bag with 40% alcohol all at once would almost certainly kill you. There was a problem a few years ago where people were experimenting with inhaling alcohol (you get drunk way faster, but it's also much easier to take a lethal dose vs. ingested alcohol). My advice: stick to the recommended one-drink-per-day (or less). (EhJJ)TALK 01:01, 12 March 2008 (UTC)[reply]
I should also point out that Steve-O was not 'well' as such after his experiment - it just didn't kill him. His body is probably far more accustomed to abuse than yours or mine too. --Kurt Shaped Box (talk) 01:15, 12 March 2008 (UTC)[reply]
I would not count on being "fine" after injecting 10 ml of absolute ethanol. Total quantity of alcohol in the bloodstream is not the only issue here. You're going to get all sorts of local damage at the injection site, probably lysis of blood cells, God knows what else. (Not to mention that it's going to hurt like anything.) Could it result in a clot that would wind up in your coronary artery? Don't know; don't see any reason to do the experiment. --Trovatore (talk) 01:17, 12 March 2008 (UTC)[reply]
Trovatore's quite right about the concentration effects—pure ethanol does really nasty things to living cells, and I agree with him about the likelihood of damage at the injection site. (Not that I would guarantee 40% ethanol won't do damage, mind you—and case reports indicate that it burns like the devil going in: [11].)
As to EhJJ's calcuation about lethal BAC's, it's worth noting that ethanol is a small molecule, and the concentration of ethanol in surrounding tissue will rapidly come to equilibrium with the ethanol concentration in the blood. This equilibration is a fairly rapid process, so you really need to be looking at a dilution of the injected alcohol into nearly all of the body's water rather than just the circulating blood volume. Even bearing that in mind, of course, a full IV bag is rather a lot of vodka to consume at one sitting, by any route of administration. TenOfAllTrades(talk) 02:22, 12 March 2008 (UTC)[reply]
Indeed, one would need to know the volume of distribution of ethanol. The volume of distribution page actually lists ethanol as an example @ 30L (no citation). Therefore EhJJ's back-of-the-envelope calculations may be off by a factor of six. But I imagine anything 40% or greater bust sting like hell as an injection... — Scientizzle 20:06, 12 March 2008 (UTC)[reply]
That article mentions snorting vodka. I've done that (a dessertspoonful). Not very much fun and not something I'm planning on doing again - it stung and made my eyes water like crazy. I was already drunk at the time, so I can't tell you if it had any significant intoxicating effect. --Kurt Shaped Box (talk) 02:42, 12 March 2008 (UTC)[reply]

Intravenous alcohol is used as a treatment for ethylene glycol poisoning -- common engine antifreeze. Ethyl alcohol (the fun kind) outcompetes antifreeze for an enzyme called ADH, which breaks booze down into vinegar but antifreeze into nasty toxins. So IV alcohol, with a little dextrose and diluted in water, is administered, in conjunction with dialysis to make sure the kidneys maintain the will to live. Obviously, this procedure can only be conducted in a hospital with adequate facilities and professionals with adequate training. See Diseases of the Kidney and Urinary Tract, pp 1201-1202, by Robert W. Schrier. Vance.mcpherson (talk) 17:10, 17 March 2008 (UTC)[reply]

Electron and Hole Concentration in Carbon

I've been trying to find electron and hole concentration data for carbon to calculate the Hall coefficient to predict the Hall p.d. across a thin carbon sheet. I can't seem to find any although I managed to find the data for silicon (http://mems.caltech.edu/courses/EE40%20Web%20Files/Supplements/02_Hall_Effect_Derivation.pdf)

Do these values depend on the sample of carbon or is there any other way of predicting the Hall p.d.? Welsh-pingpong (talk) 21:35, 11 March 2008 (UTC)[reply]

What kind of carbon are we talking about here? Graphite, diamond, or some nanomaterial like graphene (which is an atomically "thin carbon sheet")? They all have different energy band structures. Assuming we're talking about a graphite sheet of macroscopic thickness, this paper says the electron and hole concentrations are each about cm−3 at low temperatures. Which brings up another important question: what temperature is the carbon? —Keenan Pepper 23:00, 11 March 2008 (UTC)[reply]
It's a graphite sheet of thickness 0.18mm. The temperature was at room temperature with 0.1A passing through it - I didn't notice any significant temperature increase. How does temperature affect electron and hole concentrations? Also where can I find data about electron and hole mobility - I'm trying to make a prediction of the Hall voltage and need these values for the Hall coefficient. —Preceding unsigned comment added by Welsh-pingpong (talkcontribs) 08:51, 12 March 2008 (UTC)[reply]


March 12

vitamin A

wat way can i chemicaly make or extract pure Vitamin A. And if i can't get it pure, how can i get it the purest? —Preceding unsigned comment added by 76.14.124.175 (talk) 04:03, 12 March 2008 (UTC)[reply]

Probably easy to extract it from a commercial over-the-counter vitamin supplement. Depends what you want to do with it how pure is "pure". DMacks (talk) 06:35, 12 March 2008 (UTC)[reply]
Vitamin A can be make synthetically - you might need to know a lot of chemistry though - a good library should have something about it's synthesis in the organic chemistyr section.
It's possible to buy vit A (Retinol) from a chemical supplier at 99% pure.
It would also be possible to extract vitamin A from a sample containing it - though you would probably need to do chromatography to get it pure.87.102.17.32 (talk) 13:31, 12 March 2008 (UTC)[reply]

Just a reminder that does of vitamins much above the recommended make you ill or worse. I have stopped taking multivitamins after reading scientic research which showed that they do more harm than good, increasing cancer rates and reducing survival and recovery. (I do take Vitamin D though). Another example: explorers have died from eating liver from polar bears or huskies, which contain poisonous quantities of Vitamin A. 80.0.108.245 (talk) 19:51, 16 March 2008 (UTC)[reply]

CHloroacetic acid

Below is how it is made. But wat are the exacts? do i have to heat it? do i have to use pure chlorine? how do i use the catalyst?

Chloroacetic acid is synthesized by chlorinating of acetic acid in the presence of red phosphorus, sulfur, or iodine as a catalyst:

CH3CO2H + Cl2 → ClCH2CO2H + HCl —Preceding unsigned comment added by 76.14.124.175 (talk) 04:32, 12 March 2008 (UTC)[reply]

I'm gonna go with "if you don't know how to look these kinds of things up in the chemistry literature, you shouldn't be doing this reaction". DMacks (talk) 06:33, 12 March 2008 (UTC)[reply]

Chemisrty literature??????????????? —Preceding unsigned comment added by 76.14.124.175 (talk) 23:48, 12 March 2008 (UTC)[reply]

chemotherapy or radiation therapy for obesity

Could anticancer therapies be tweaked or redesigned to attack fat cells? Thanks, 05:37, 12 March 2008 (UTC)Rich (talk) 05:38, 12 March 2008 (UTC)[reply]

Have you read our article on antineoplastics and/or chemotherapy? There are a number of different techniques, but they typically rely on cancer's fast growth rate (for example, they slow/damage new DNA synthesis). This works really well on fast growing cells (such as malignant tumors and hair follicles), it doesn't typically affect slow growing cells (such as neurons and adipocytes (aka fat cells)). So, generally speaking, I'd say no. (EhJJ)TALK 11:43, 12 March 2008 (UTC)[reply]
Also, it would be bad to get rid of fat cells. You want a lot of small fat cells for smooth skin. With no fat cells you would look like a skeleton, and with a few fat cells they would tend to become large and clump together, giving you a lumpy appearance. StuRat (talk) 15:01, 12 March 2008 (UTC)[reply]
Good answers, thanks.Rich (talk) 21:09, 12 March 2008 (UTC)[reply]

steroids

Whats the difference between anabolics and steroids? —Preceding unsigned comment added by 75.4.67.159 (talk) 05:43, 12 March 2008 (UTC)[reply]

Our anabolic steroid page is a good place to start. DMacks (talk) 06:11, 12 March 2008 (UTC)[reply]

Quantum Mechanics: Entangled Wave Function

The equation 9, or (EPR9) for short here, in the original paper of EPR paradox gives a wave function of two entangled particles

(EPR9)

where is Planck's constant, and are the variables describing the two particles and is just some constant. According to reduction of the wave packet, when an observable of the first particle is measured, (EPR9) can be expanded by the eigenfunctions of in the form

(EPR8)

where are the corresponding coefficients to the eignefunctions. If is a continuous observable, the coordinate of the first particle, (EPR8) can be written as

(EPR15)

According to the paper, the eigenfunctions of is

(EPR14)

which has corresponding eigenvalue . The first question is how come the eignefunction and the eigenvalue of are (EPR14) and , respectively? It seems that

(1)

and if we let

(2)

then

(3)

Find the solution of

(4)

we have

(5)
(6)

the right-hand side of (6) is the eigenvalue of . Similarly, the eigenvalue of the observable

(EPR17)

can be found by knowing

(EPR16)

and let

(7)

The solution of

(8)

is

(9)

Again, the right-hand side of (9) is the eigenvalue of which complies with the paper. But it still doesn't explain how to figure out the eignefunction (EPR14).

To continue the unsolved discussion last time, the second question is how to denote the entangled wave function (EPR9) in bra-ket notation? If it can be done, it should help with respect to the last discussion. The bra-ket notation of (EPR9) is supposed to be in the Hilbert space which is the tensor product of the state spaces associated with the the two particles. - Justin545 (talk) 06:36, 12 March 2008 (UTC)[reply]

Hi, I'm sorry I haven't followed up to the old thread yet, but maybe a response here will serve the same purpose.
There are many ways to write (EPR9) in bra-ket notation; for example I could just write where Ψ is defined by (EPR9). In terms of tensor products of kets inhabiting the state spaces of the individual particles, I could write for example . I'm not sure those are properly normalized, to the extent that these mathematical monstrosities can be considered to be normalized to begin with. The product might also equivalently be written or or . The subscripts 1 and 2 just indicate which subspaces the kets inhabit; they could be left off since the two subspaces are isomorphic in this case.
I'm not sure I understand your first question. Finding eigenfunctions of the position operator in a single-particle space involves solving equations of the form where B(z) = z and BΨ is a pointwise function product. It should be clear enough that the only possibilities for Ψ here are functions that are zero everywhere except at a point, and the "normalized" versions of these functions are the delta functions, which form an orthonormal eigenbasis. In the two-particle space things are a bit more interesting. You're now solving where . The normalized solutions here are where g is any normalized function of x2. These do not form a basis; there are far too many of them for that. You have to choose arbitrarily some orthonormal basis for the functions g. This happens because there are degenerate eigenvalues; the discrete analogy is that there's only one orthonormal eigenbasis for diag(1,2,3) but many for diag(1,1,2). -- BenRG (talk) 12:55, 12 March 2008 (UTC)[reply]
It's reasonable making the ket be the function of the corresponding eigenvalue since each eigenvalue identifies an unique basis or eigenfunction. But, I am a bit confused with the bra-ket notation since I expect the bra-ket notation should be in the form
(10)
rather than in the form
(11)
It seems the integral surrounding the ket can not be removed. But, will the integral of the ket yield another "ket" in the same space? Another confusion is about the momentum part of the bra-ket example . I am not able to figure out in it.
Apologies for obscuring my first question. My first question is just to understand why the eigenfunction of is a "delta function". Just wonder how the delta function (EPR14) is mathmatically derived. As you said "Finding eigenfunctions of the position operator in a single-particle space involves solving equations of the form where B(z) = z and BΨ is a pointwise function product." But I can not understand why it's pointwise. Excuse my poor quantum mechanics, I left so many question marks here :-) Justin545 (talk) 08:43, 13 March 2008 (UTC)[reply]
The integral is the sum, it just happens to be a sum with uncountably many terms. You need an uncountable sum here because the wave function is a superposition of uncountably many tensor-product states—the particles could be at x and x+x0 for any real x. I picked somewhat arbitrarily the position basis vectors and the momentum basis vectors . They're somewhat arbitrary because they're only unique up to scalar multiplication, but they're eigenvectors of the appropriate operators with the appropriate eigenvalues (unless I got the sign convention backwards). Then and . So in particular , which is where my momentum integral form came from. The position integral one is odder. When , EPR9 gives , which to a mathematician is undefined but to a physicist is zero. When , EPR9 gives , which to a physicist is the peak of a delta function. So EPR9 describes a "function" that's zero everywhere except on the line where it's infinity, and my position integral expressed that more directly.
Let me explain the operators in a finite-dimensional case. Let's say we have a four-state system with position states and momentum states (the Fourier basis). We can arbitrarily assign a distinct real number to each position and to each momentum. Say the positions are 1,2,3,4 and the momenta are 0,1,2,−1. Then there exists a matrix which scales each position/momentum axis by the corresponding real number. For the position basis it's just diag(1,2,3,4), while for the momentum basis it's U diag(0,1,2,−1) U−1, where U is the unitary matrix whose columns are the aforementioned Fourier basis vectors. This matrix will always be Hermitian (it's a theorem that a matrix is Hermitian if and only if it can be written in the form UAU−1 where U is unitary and A is real diagonal). In this case the scaling factors were all distinct, so by solving the eigenvalue equation we can recover the original basis from the Hermitian matrix. If some scaling factors are equal then all you can tell is that a particular (hyper)plane was scaled by that factor; you can't uniquely recover the basis vectors lying in that hyperplane. That's the case for a four-state system that's the product of two single-particle two-state systems, where the position of the two particles might be represented by the matrices diag(1,2,1,2) and diag(3,3,4,4) respectively. In the continuous case you can't write down matrices any more, but the differential operators serve the same purpose. In order to get the right eigenbasis and eigenvalues, the position operator has to multiply the wave function by a real number corresponding to the position, which is why I described it as a pointwise function product. It might have been better to say that B is an operator defined by (B f)(x) = x f(x). -- BenRG (talk) 14:32, 14 March 2008 (UTC)[reply]

Cancer

Why do swelling often occurs around the tumor? there doesn't seem to be an answer in wikipedia.


There's no single answer; tumor can block lymphatic drainage and cause fluid accumulation; cytokines secreted by tumor cells can cause inflammation; some tumors upregulate angiotensin II type 1 receptors, triggering chronic inflammatory response; some tumors cause the formation of new blood vessels (angioneogenesis) causing hyperemia and swelling. - Nunh-huh 06:59, 12 March 2008 (UTC)[reply]
And then there's the simple method of the tumor growth itself pushing on the surronding tissue, which then expands outward, similar to how a growing weed can push asphalt upward. StuRat (talk) 14:57, 12 March 2008 (UTC)[reply]
Tumors often elicit immune responses from cytotoxic T lymphocytes, but also bear in mind that tumors promote angiogenesis as well. Wisdom89 (T / C) 18:49, 12 March 2008 (UTC)[reply]

Hepatitis C

How dangerous and contagious is hepatitis C, can it be transmitted through a cats claws? —Preceding unsigned comment added by DSTiamat (talkcontribs) 08:31, 12 March 2008 (UTC)[reply]

If you'll read our article on Hepatitis C, you'll find that it is spread by blood-to-blood contact, and over 90% of cases are spread through blood products or recreational drug use. A less common form of spread is via sexual contact. Though one can make up scenarios in which an infected person is scratched by a cat, who then immediately scratches and infects someone else, this is not something that's ever been reported as actually happening. Hepatitis C infections can range from asymptomatic to fatal, so obviously its dangerousness varies greatly, depending on a number of other factors, including luck. - Nunh-huh 09:10, 12 March 2008 (UTC)[reply]
If you want a disease associated with cat scratches, look up cat scratch fever. StuRat (talk) 14:52, 12 March 2008 (UTC)[reply]

Who makes the 120mm fans used on the space shuttle?

I have no Idea on where to go to start looking for such information, how many different manufacturers are used and what they are used for...just think it'd be interesting. —Preceding unsigned comment added by 71.237.205.31 (talk) 09:11, 12 March 2008 (UTC)[reply]

You need to start with a specific module so you can narrow down to the country that built the module. Then, you can hope some of the contracts were made public within that country. -- kainaw 15:16, 12 March 2008 (UTC)[reply]
The shuttle has at least four different types of fans in the environmental control system. There are two cabin fans, six avionics bay cooling fans, three IMU fans, and at least one airlock booster fan; there may be more I'm forgetting. I'm not sure if they are all identical (I would assume not). I don't know any details (or even where to start looking) about make and model of the fans. anonymous6494 18:32, 12 March 2008 (UTC)[reply]
Whether you are looking for low noise, high efficiency, or

the security of cooling fans designed and built for the long haul, Howden Cooling Fans has lead the market since 1955. With over 150 years of experience, Howden is the world’s largest and longest established manufacturer of air and gas handling equipment. Today, no one equals Howden’s accumulated knowledge of fans, with applications ranging from cooling fans on the space shuttle to 15,000 hp axial fans for power station boilers.

Biochemistry

The process in purin purification by Molecular exclusion62.24.99.237 (talk) 10:51, 12 March 2008 (UTC)[reply]

You probably meen purine and "molecular exclusion" can be searched for on the web, or may be described at Size exclusion chromatography87.102.17.32 (talk) 18:40, 12 March 2008 (UTC)[reply]
Could you be more specific87.102.17.32 (talk) 18:41, 12 March 2008 (UTC)[reply]

Panic Attack while teaching my kid

This question has been removed. Per the reference desk guidelines, the reference desk is not an appropriate place to request medical, legal or other professional advice, including any kind of medical diagnosis or prognosis, or treatment recommendations. Please consult a physician, psychiatrist or psychologist--(EhJJ)TALK 11:33, 12 March 2008 (UTC)[reply]

(EhJJ)TALK 11:33, 12 March 2008 (UTC)[reply]

heat conductor

Why does the a plain cloth catches fire when put near a flame at about 10 cm compared to the cloth wraping a coin in it? —Preceding unsigned comment added by 165.21.155.69 (talk) 12:43, 12 March 2008 (UTC)[reply]

Maybe it gets hotter quicker without the coin?87.102.17.32 (talk) 13:33, 12 March 2008 (UTC)[reply]
Yes, the coin is a good thermal conductor, so pulls heat out of the cloth, acting as a heat sink, until it gets up to temp, then it will no longer draw off heat. StuRat (talk) 14:49, 12 March 2008 (UTC)[reply]

we are looking for info regarding mining

What is fools gold ( piriet) and why do they call it fools gold?

The impact that the mining of gold has on SA's environment: The use of sinade and uranium as by-products in the mining process The influence of mining on water and the effect of water on buildings Mine heaps amounts of rock removed from the earth surface. Air Pollution. —Preceding unsigned comment added by 196.211.98.154 (talk) 13:03, 12 March 2008 (UTC)[reply]

Fools gold is pyrites (spelling) it's actually Iron Sulphide.
sinade = cyanide which is used to extract the gold.87.102.17.32 (talk) 13:13, 12 March 2008 (UTC)[reply]
Gold mining should be of interest. there's some info on the enviromental hazards of cynaide seeCyanide_process#Effects_on_the_environment.87.102.17.32 (talk) 13:16, 12 March 2008 (UTC)[reply]
Also try http://www.google.co.uk/search?hl=en&sa=X&oi=spell&resnum=0&ct=result&cd=1&q=gold+mining+environment&spell=1
Looks like uranium is a by product and a pollutant produced when gold in mined http://www.ingentaconnect.com/content/klu/gejo/2004/00000061/00000002/00002867;jsessionid=1rj6eslojhj0m.alexandra? —Preceding unsigned comment added by 87.102.17.32 (talk) 13:20, 12 March 2008 (UTC)[reply]

Looks like South African homework. :-P --98.217.18.109 (talk) 14:29, 12 March 2008 (UTC)[reply]

Hi. It might be. OP is from South Africa. ~AH1(TCU) 00:48, 13 March 2008 (UTC)[reply]
Fool's Gold has the chemical formula FeS2. The name is derive from it's physical appearance. Miners mining for gold occasionally stumble upon shiny flaky materials mistaking it for actual gold. Hence the term fool's gold. Hey mrs tee (talk) 09:30, 19 March 2008 (UTC)[reply]

Cuttlefish wants to cuddle

How do Cuttlefish know how to blend in with a given environment if they can't see in color? Why would their skin have such a wide range of colors possible if they can't distinguish between most of them? --98.217.18.109 (talk) 15:07, 12 March 2008 (UTC)[reply]

According to our article, cuttlefish change color to camouflage themselves as well as communicate. The camouflage function would benefit from a range of colors even if they are colors the cuttlefish can't themselves see. As for how they match colors they can't see, I don't know how they do that, and according to this article, at least one cuttlefish camouflage expert doesn't know either. --Allen (talk) 15:33, 12 March 2008 (UTC)[reply]
This article is also fascinating. —Keenan Pepper 18:37, 12 March 2008 (UTC)[reply]
See also chromatophore, specifically the section on Cephalopod chromatophores. Rockpocket 19:00, 12 March 2008 (UTC)[reply]
Let's get this straight -cuttlefish are colour blind but they do match themselves to the colour of the surroundings??? so they camoflage based on the grey scale image they see —Preceding unsigned comment added by 87.102.17.32 (talk) 20:55, 12 March 2008 (UTC)[reply]
Well, you're assuming that the color change is in response to a visual signal. Maybe it's not; maybe something in the cuttlefish's skin (if that's the right word) responds directly to the color of the light hitting it, without going through the eye at all. --Trovatore (talk) 21:04, 12 March 2008 (UTC)[reply]
I wondered about light sensors in the skin..
Though in one of the references above -(nytimes) - it describes the placement of a high contrast pebble in the fish tank, the cuttlefish then 'looking at it' and then adding a black spot to its skin pattern.. That shows (I think) at least that the eyes are at least a part of the camoflage set-up..87.102.17.32 (talk) 21:50, 12 March 2008 (UTC)[reply]

I hadn't read the Science article when I gave my answer; now I've scanned it and it looks like the NYT reporter Carl Zimmer must have made an error. He quotes Dr. Hanlon as saying "we don't know how" [in Zimmer's words:] "they see a world without color, but their skin changes rapidly to any hue in the rainbow." But the Science article clearly demonstrates that cuttlefish's skin does not change to any hue in the rainbow. So maybe there's no mystery: cuttlefish are color blind, and therefore have trouble camouflaging themselves against certain colorful backgrounds. But on the other hand, there is the last sentence of the Science paper: "However, the vexing question of how S. officinalis masters the task of camouflage in chromatically rich environments, such as those found at shallow depths of water, remains to be answered." So Mathger et al. are saying there is a mystery... but what are they referring to? The paper doesn't seem to cite any instances of S. officinalis mastering the task of camouflage in any truly chromatically rich environment. At least not that I can see. --Allen (talk) 02:21, 13 March 2008 (UTC)[reply]

urea formation

Hi all read the article on urea cycle found it a bit confusing... could anyone tell me how the NH2 group removed from an amino acid in deamination is converted to ammonia (NH3), Where does this hydrogen come from? I believe the NH2 is converted to NH3 before it reacts with CO2 to form urea, but please correct me if that is wrong. Thanks. —Preceding unsigned comment added by 172.142.47.24 (talk) 21:10, 12 March 2008 (UTC)[reply]

Are you refering to "conversion of glutamate to ammonium and α-ketoglutarate." - the 'co-factor' is NAD+ see Glutamate dehydrogenase. Was that the bit you meant. the amino acid is oxidised to a keto compound.
In the big diagram http://en.wikipedia.org/wiki/Image:Urea_cycle_2.png 'ammonia' remains bound to the molecule as an iminium cation, with loss of fumarate, water hydrolysis the iminium cation to urea..87.102.17.32 (talk) 21:42, 12 March 2008 (UTC)[reply]
Urea Cycle Metabolic Pathway
L-citrulline Carbamoyl
phosphate
L-ornithine
Pi
L-aspartate Urea
+ ATP
PPi + AMP H2O
L-argininosuccinate Fumarate L-arginine


This is probably the easier diagram - if you are still stuck can you reference your questions to this diagram. Just to make it easier (assuming it's relevent)87.102.17.32 (talk) 21:45, 12 March 2008 (UTC)[reply]

The "fish scales" in the clouds are back!

pink-teal iridescence, traces of hints of blue, violet, orange
pink-teal iridescence, orange-red halo, blue, green, and violet patches, blue-green inner halo glow, near-sun whiteness

Hi. Remember my previous question? The one a few weeks ago with the 8 deg halo and the iridescent "fish scales" in the clouds? Well, here's the story. Yesterday I saw a roughly 25 deg from the sun rainbowlike halo, and a sundog while the sun was in a cirrus-cirrostratus-altostratus cloud. Anyway, remember last time I saw those "fish scales" in the clouds and that halo as I was typing? Well, I didn't have a camera. Well, guess what? This time I was in a car. I saw it again. At first I thought we didn't bring the camera, but it turns out we did. Yay! I took dozens of pictures, but erased every one of them except seven, because the others were too indistinct. Anyway, I uploaded two of them. Click on the images if you wish. Now, remember last time there was a cirrocumulus cloud passing by as I saw the iridescent fish scales? This time, cloud after cloud passed by or near the sun. The way I saw them were much, much more colourful than in the pictures, probably because the camera was automaticly set to neutral colour or something. Anyway, the clouds were probably stratocumulus, but some of them were very fibrous, and there were some clouds slightly higher than others. Look carefully for the colours, because they're much harder to see in the images than in real life, which is why I only picked the best two. Notice in the pictures: The organge halo, the pink-teal iridescence, and a hint of rainbow colours. What do you think they are, and do we have an article on them? Thanks. ~AH1(TCU) 23:23, 12 March 2008 (UTC)[reply]

It's not exactly a glory, but that page may help. Cheers Geologyguy (talk) 01:01, 13 March 2008 (UTC)[reply]
No, it's not a glory; I guess the closest article would be Cloud iridescence. It's simply dispersion of sunlight by water droplets in the clouds. FiggyBee (talk) 03:43, 13 March 2008 (UTC)[reply]
There's a nice sample here[12] at the Cloud Appreciation Society site, but I like your "fish scales" description. Your pics wouold go well in the iridescence article to relieve the high contrast ones it has, to show some alternatives. I've seen what you describe – glad to know it's a phenomenon. Julia Rossi (talk) 07:24, 13 March 2008 (UTC)[reply]
Wow a RARE PHENOMENON!!! I saw this phenomenon twice and I had my camera one time and uploaded it!!! YAY!!! ~AH1(TCU) 17:17, 13 March 2008 (UTC)[reply]
You might check out Marcel Minnaert's book: The Nature of Light and Color in the Open Air. --Carnildo (talk) 21:40, 13 March 2008 (UTC)[reply]

March 13

Recent rise in oil prices?

Hi. I'm pretty sure most of you have noticed recent sharp increases in oil prices. In fact, just a few years ago, there were occasional times where some gas stations would give prices of less than 50¢ a litre. At that time, gas prices per litre seldom reached above 90¢ per litre. Prices above $1/l were almost unheard of. Nowadays, prices above $1 are the norm. Prices below 90¢ are now almost unheard of. The news has said that, this spring, prices could reach $1.30. Oil is also constantly above $100 a barrel, if you're American. What's going on? This all started after hurricane Katrina, it seems. Are we reaching global peak oil already? Yes, I know the US and non-OPEC countries have already peaked. Are oil companies doing this on purpose? Or is this permanent and a sign of what's to come? What's next? $2.00 a litre within the next decade? $200 a barrel? Would peak oil cause a spike in prices? Yes I know it will probably run out completely in like, 40 years, so hopefully we switch to non-fossil fuels by then. As oil dwindles but not completely, what will we see? Maybe $10 a litre, $900 a barrel? Thanks. ~AH1(TCU) 00:03, 13 March 2008 (UTC)[reply]

Your question is asking people to foretell the future. This is a reference desk, not a magic crystal you can rub and hope for a good answer. However, it isn't hard to figure out. Oil supplies are drying up. Oil demand is increasing faster than ever before. What do you think the price should do? -- kainaw 00:13, 13 March 2008 (UTC)[reply]
The present run-up (that is, above US$95 or so) is mostly related to the weak dollar - people want to hedge their investments by moving them to commodities. Daily fluctuations ($2-$3 or thereabouts) usually reflect particular news items, from weather to stocks reports to saber rattling. The basic price (say something like US$85 to $95) is a reflection of supply (near or at capacity right now) and demand (increasing despite the price). Best get used to it. Cheers Geologyguy (talk) 00:20, 13 March 2008 (UTC)[reply]
Any of your scenarios are possible, if not likely, but who can tell the future. For example I've heard serious predictions of $2 - $3 per litre within the next few years in Australia (current average price is about $1.40). And many sensible commentators also suggest that we've already passed worldwide peak oil and are now on the decline. --jjron (talk) 13:49, 13 March 2008 (UTC)[reply]

I find it entertaining seeing Americans complaining about fuel prices when we pay £1 per litre (which is about $8 per gallon) in the UK TheGreatZorko (talk) 14:11, 13 March 2008 (UTC)[reply]

Most of that is tax though. But I will say, complaining is fun, and it allows us british to feel superior knowing that we have much more right to complain than they do. -mattbuck (Talk) 14:30, 13 March 2008 (UTC)[reply]
Speaking as an American who is not complaining (mostly I'm amazed that our prices are not a lot higher), I really shudder to think what the US will be like when reality actually sets in. Cheers Geologyguy (talk) 14:34, 13 March 2008 (UTC)[reply]
I don't complain either because I don't drive an SUV the size of Kansas around. One time I pulled up to the pump after the previous person (who was an owner of a huge SUV) left and almost laughed to death when I saw he had paid almost $100 for gasoline. I haven't been to Europe in about 5 years, but from what I remember they aren't as stupid as we are, and they buy smaller cars unless they need to tow something. Do get me started on driving habits either - everyone I see with a superfluous SUV wastes gas by flooring the pedal the moment the light turns green. And yet they complain, and then they wonder why the world hates America so much. 206.252.74.48 (talk) 19:33, 13 March 2008 (UTC)[reply]

climate modelling simulation map-based continental land-influenced scenario?

Hi. I have a method you might be able to use for calculating the climate of a specific time period. Look at it and tell me what you think and what adjustments might need to be made if it plausibly works. Here are the steps:

map
  • first you need to draw on a piece of paper the shape of your Earth. Make sure you include the whole world in your map shape. You may want to get a huge piece of paper, and maybe several pieces. Now draw a grid for your map. Next, draw the continents. Take note of the elevation. If it is a map far into the past or future by millions of years, make sure you examine all the evidence. Make four maps, one for mid-winter, mid-spring, mid-summer, and mid-autumn. The seasons may not be at the months they are today because of precession, or the days might be longer or shorter, so don't write the month. If this is a period without seas, water, water vapour, sunshine, oxygen, vegetation, volcanoes, or plates, or if it is during a great bombardment, the map will be useless for that era.
  • Examine the plates at the time of the era of your map. If the fossil record shows proof of high mountainous habitats or undersea environments, include it in your map. Do not base the continents on what they look like today. Take note of the sea level. If the sea level was 300 metres below today's, then treat a piece of land that would be at sea level today as if it was 300 metres above sea level, and same if the sea level was or will be higher. If you're doing to future, include as many factors as possible in your simulation of placement of the continents. Draw in any plausible islands. If the mountain ranges that are here today did not exist back then, do not draw them. If it is likely that where there are mountains today were part of the sea floor back then, treat it as the sea floor. Make note of what type of climate it was. If there is evidense of a gulf or bay or inland sea or lake, draw it in. If there are landmasses near the poles there will likely be large glacial sheets.
  • make notice of the rivers and glaciers. If the mountains were high and cold enough, draw in glaciers at that reigon. If it is likely that there were pack ice or large sheet of ice like on Antarctica or Greenland today, draw it in. Make sure you draw these at the correct portion of the grid. If there was a seamount or undersea volcano or likely there was a coral reef or will be and the sea level is low enough to hold them above sea level, draw that in. Calculate sea level carefully, take in the factors. Look for likely positions of rivers and wetlands. If you know that there were deserts or forests in a particular location or will be, make not of that.
jet stream
  • draw 6 seasonal jet streams for both the northern and southern hemispheres, 2 polar, 2 temperate, and 2 tropical. Do this for each season. If the season in the hemisphere is summer, draw the jets closer to the poles, farther in winter, and moderately close tot he poles in spring and moderately far in autumn. Assuming that the Earth is rotation west-east during this era. Draw the polar jet streams with wind from east to west. Draw them from east to west, and make sure they meet on both ends of the map, like if it was at 80 deg at the rightmost egde of the map, make sure it is at 80 deg at the leftmost edge of the map. Cause it to dip south if there is a mountain range or large glacial sheet. If it was a warm period, draw them close to the poles. If it was a cold period, draw them away from the poles. If it was an ice age or there is an ice sheet on land kilometers thick covering the entire polar reigon and then some, draw them from west to east. Make sure that if there is a huge sheet of ice like this, make it so that the jet streams avoid the ice. On the leeward side of large tall mountain ranges that polar jet streams cross, which is the side facing away from the jet streams, cause the streams to dip towards the poles. The streams dip away from the poles if it encounters tundra or desert or ice, toward the poles if it encounters unfrozen water. It will dip towards the poles when it encounters forests in spring or summer, and away form the poles in autumn or winter. It will dip towards the poles if it encounters a coastal area after being over the sea.
  • the temperate jets are at temperate latitudes, from west to east Again draw them according to the season and warmth of the period. If it encounters mountains it will dip toward the equator. If in a warm season it encounters water it wil dip away from the poles and if in a cold season it will dip toward the poles. Rivers don't count as "water" unless it was a bay or gulf hundreds of km wide. Large areas below sea level but are not covered in water get a poleward dip and for other jets as well. Wetlands act like bodies of water if they are large enough. Deserts get poleward dips in spring and summer and equatorward dips in autumn and winter. The dips in spring and autumn are less than in summer and winter. Dip equatorward for any glaciers. For dry salt pans dip equatorward.
  • tropical jest go from east to west. Again draw them according to the global climate at the time. Oceans get a dip toward the equator, but if it just came from the sea and just met land at the coast, dip poleward. Make poleward dips if it just came from a forest. Dip poleward for deserts, and dip equatorward if it just came out of a desert. Do not cross the equator into the other hemisphere. Dip equatorward for mountains and any glaciers. Lakes and wetlands get equatorward dips if crossed.
high and low pressure
  • Label high pressure if an area generally has jet streams circling clockwise in the northern hemisphere and counter-clockwise in the southern hemisphere. For low pressure, the jets should be circling counter-clockwise in the northern hemisphere and clockwise in the southern hemisphere. Label warm and cold fronts around the warm sectors of low pressure systems. The fronts can connect with other low pressure systems but not with high pressure. They should not cross the equator. Create isobars around the high and low pressure. Make the pressure systems especially intense if they are large. Have generally higher pressure at lower areas and lower pressure on mountains and high elevation, including ice as elevation. Make sure you do these for each season, as with below.
prevailing winds
  • Draw wind direction around the pressure systems. Remember which direction the pressure systems turn in. Wind also goes away form high pressure and toward low pressure. Label upwelling and sinking of wind, it rises if it hits mountain ranges, sinks if it hits a valley or the ocean from high elevations, rises at low pressure and sinks at high pressure. Also label upper-level winds which go from upwellings towards sinkings. Wind should be allowed to cross the equator.
ocean currents
  • Draw warm and cold ocean currents. Draw cold ocean currents in cold areas and where the wind goes equatorward. Draw warm ocean currents near tropical reigons and areas where wind goes poleward. Only count surface winds. Do not draw them for lakes and inland seas disconnected from main sea unless they are enormous and deep. Make sure the currents avoid land even if it means going in the opposite direction from prevaling winds. Warm and cold currents should be allowed to converge and cold ocean currents should cross the equator unless something blocks it such as a massive release of freshwater into the oceans. Only the largest rivers should be allowed to nudge existing currrents away from its mouth. Cold currents should not enter shallow tropical gulfs, bays, and seas, and warm currents should not cross polar seas. Currents should be allowed to directly cross under a pressure system if nessecary.
precipitation
  • label wet and dry zones on the map. Areas where wind goes from the unfrozen sea or lake to land should be labelled wet. If warm currents approach land then makes a turn, colour in the areas it was approaching before it turned as wet. Leeward sides of mountains should be labelled dry. Areas directly under high pressure should be labelled dry. When cold ocean currents approach land, label these as dry. If wind from a large already wet place goes into another place, label it wet, and if wind from a large dry place goes to another place, label it wet. Label polar glaciers as dry. The windward side of a mountain, if the wind is from the ocean or a lake or a wet place, it should be labelled wet. Wind going from the land to the sea should be labelled dry. Areas where the jet stream goes from the ocean to land should be labelled wet until it turns around a large high pressure system. Areas ahead of a cold or warm front should be labelled wet, the cold front area larger in summer and warm fromt larger in winter. If the area is especially close to the equator, both spring/autumns should be considered summer. If an upwelling of air occurs near both the sea and the land, that area should be labelled wet. If upper-level winds go from the sea at an upwelling of air towards the land at moderate or high elevations, those areas should be labelled wet. Extremely large supercontinents should be labelled dry near the middle. Create lines denoting areas of equal precipitation for the particular month. Adjust the amount or precipitation of areas between wet and dry based on the global wetness at the era.
temperatures
  • If there is a large polar ice sheet on land, they are always cold, but warmer in summer than in winter. In polar areas, ocean should generally be warmer than land. Dips towards the poles should be warm, dips equatorward should be cooler. Mountains and higher elevation should generally be colder than surroundings. Temperate wet areas should be cooler than surroundings in spring and summer, and warmer in autumn and winter. Tropical wet areas should be cooler, tropical warm areas warmer. High pressure should be cooler if cold and warmer if warm. Low pressure should be warmer if cold, cooler if warm. Downwind from leeward side of mountains should be warmer. Poleward surface winds should be warmer, equatorward cooler. Consider the average global temperatures during that era and which areas are at which temperatures, create isotherms accordingly.
ecosystems and biomes
  • The ecosystems should be drawn on one map. Base it similarly to the modified Koppen system. If the total rainfall is very low for the four seasons, label it desert. Slightly wetter areas semidesert. Polar reigons, if there are glaciers, label it ice, if it’s cold and dry all year, label it tundra. If it’s sometimes warm in polar reigons and moderate precipitation, label it boreal forest. If a tropical area is warm and wet, label it rainforest. Label monsoonal reigons accorgingly and also based on their total precipitation. Label deserts hot and cold. Keep doing this according to both the climate in the areas and the likely biomes in the area.
final thoughts

Well, what do you think? I once tried this method myself (but much, much more simplicified) on the current climate and the near future, and got similar results to the real climate. Does this work, and what are some of the potential modifications tha tmay need to be made? No this is not homework. What would one discover if they tried to base it on global warming of the future? Would northern Europe cool down? Amazonia turn to grassland? Acatama and Namib deserts turn to mediterranean climate? Central North America become desert? Sahel become desert? Lower Tibet become desert? Monsoonal India become drier? North America west coast become drier? California become hotter? Increased El Ninos? SAL across the Atlantic? Little change in Atlantic hurricane activity? Is this similar to some of the climate modeling simulations? Can this be used to predict past and future climates based on continental drift? Or do we have to implement what we already know for the Holocene for it to be accurate? Thanks. ~AH1(TCU) 00:25, 13 March 2008 (UTC)[reply]

non-medical placebos?

Hi. I know when you apply a treatment and convince the patient it will work and it works, that's a placebo, what what about non-medical, ie. the power of suggestion? Like for example, if you go into a dark house, and someone convinces you it's haunted, then suddenly you start experiencing signs of "ghosts" when it's really your mind playing tricks on you based on the suggestion, what is this effect called? Or, if you convince yourself you're sick, but you're really not, and you begin to experince what you think are "symptoms" of some sickness? Thanks. ~AH1(TCU) 00:29, 13 March 2008 (UTC)[reply]

Perhaps you are thinking of psychosomatic or conversion disorder --Omnipotence407 (talk) 00:49, 13 March 2008 (UTC)[reply]
See also: nocebo effect, suggestion, hypnotism.
You might also try confirmation bias or hypochondria. - Eldereft ~(s)talk~ 00:53, 16 March 2008 (UTC)[reply]

"Random outbursts give people cancer"?!?

Hi. This is not a request for medical advice. Is this possible? Or was this a sarcastic joke or itself a random outburst? Or is it brain cancer? Is the joke about "if your hand is bigger than your head, you have cancer" solely in order to slam the victim's hand into their face? Or was this question a random outburst? Thanks. ~AH1(TCU) 00:44, 13 March 2008 (UTC)[reply]

Complete and utter hoax. Wisdom89 (T / C) 00:50, 13 March 2008 (UTC)[reply]
Maybe they meant random outbursts of gamma rays. And yes, the thing about your hand is so that you can hit someone on the nose. That's it. --98.217.18.109 (talk) 00:58, 13 March 2008 (UTC)[reply]

Goatse - how does he do that with his butt?

How did the Goatse man manage to get his rectum so insanely stretchy? Is there a likely medical explanation for it (I saw a guy who could stretch his skin about two feet from his body on tv once, maybe its the same thing), or was it just a "gift" he was born with? How do you go about discovering that you can do that anyway? --62.136.16.236 (talk) 00:58, 13 March 2008 (UTC)[reply]

Well, you can bet he probably started small and worked his way up. If you view the whole series (not recommended), you'll see that he inserts extremely large objects into his rear before doing the classic pose. That's got to help, if that's your goal. --98.217.18.109 (talk) 01:01, 13 March 2008 (UTC)[reply]
It's actually a martial art perfected by the Chinese monk Goa Tse. See here. bibliomaniac15 01:02, 13 March 2008 (UTC)[reply]
Eww!! That's uncyclopedia!!!!!!
As can be seen in slideshow form at http://www.goatsemarathon.com (obviously not worksafe). I've sometimes wondered if his internet antics (legendary they may be) have affected his continence. --Kurt Shaped Box (talk) 01:49, 13 March 2008 (UTC)[reply]
Note to self: When 98.217.18.109 says "not recommended", you don't need to check for yourself. ៛ Bielle (talk) 02:09, 13 March 2008 (UTC)[reply]
It's been years since I've seen it yet I can remember some of the scenes like it was yesterday. It's one of those visual memories you sort of resent having semi-permanently burned into your brain somewhere. Also, re: speculation about his health, I doubt that's a terribly good thing to do for your body and probably has a number of long-term side effects. That's not medical advice though I doubt any doctor would recommend doing that. --98.217.18.109 (talk) 04:38, 13 March 2008 (UTC)[reply]
Men who engage long-term in normal (well, normal for some) receptive anal intercourse (let alone grotesque activities such as fisting) often end up with continence problems because of the weakening of the muscles. What this guy has done to himself ... I'm lost for words. -- JackofOz (talk) 05:20, 13 March 2008 (UTC)[reply]
Jack, being familiar with all matters Down Under, speaks wisely. If this practice hasn't already wrecked him, I'm confident it soon will. I'd wager that traditional medicine would frown on this practice, but holistic medicine would open up to it, perhaps. StuRat (talk) 06:53, 13 March 2008 (UTC)[reply]
I dont know if the Down Under comment was accidental Stu, but its hilarious nevertheless! —Preceding unsigned comment added by 79.76.144.62 (talk) 01:44, 14 March 2008 (UTC)[reply]
Jack, would you be speaking from experience or is it just fistwishful thinking? —Preceding unsigned comment added by 79.76.144.62 (talk) 01:38, 14 March 2008 (UTC)[reply]
Might it cause a Fist ula? Edison (talk) 18:22, 13 March 2008 (UTC)[reply]
Sounds odd - but I think that the internet would probably be a poorer place without the Goatse Man. He's inspired a lot of people to do creative, humourous and culture jamming-related things over the years - just take a look at all those 'tribute to Goatse' and 'Goatse sightings in the real world' pages. Heh, Time Magazine may have even placed a deliberate nod to him on one of their covers... --Kurt Shaped Box (talk) 17:41, 13 March 2008 (UTC)[reply]
Most people believe the goatse man is Kirk Johnson. If you are sick enough to compare his vast collection of photos and videos to the official goatse photo, it is difficult to imagine that Kirk is not the goatse man. So, now that you have a good idea who he is, you search for his email address and ask him about his practices. Who knows, he may have written books or produced do-it-yourself videos. -- kainaw 18:54, 13 March 2008 (UTC)[reply]
I'm pretty sure the goatse man is dead. In fact just Google that phrase (goatse man is dead) and you'll find plenty of links, none of which I'm going to attempt to visit from my workplace computer. --LarryMac | Talk 19:11, 13 March 2008 (UTC)[reply]
If you heard a story about him dying at the age of 70-something, that was from a hoax newspaper article knocked up by the boys at Stileproject years ago. If he died recently, I haven't heard anything about it. --Kurt Shaped Box (talk) 22:50, 13 March 2008 (UTC)[reply]

psa levels

can alchohol consumption effect psa levels? —Preceding unsigned comment added by Jon julie (talkcontribs) 02:03, 13 March 2008 (UTC)[reply]

If people didn't drink so much, there wouldn't be so many public service announcements about drunk driving and so forth. I hope this answers your question.--The Fat Man Who Never Came Back (talk) 02:26, 13 March 2008 (UTC)[reply]
To translate, "psa" could refer to anything on this page. Could you please (a) work out which one you're talking about, (b) check the article about it to see if it mentions the effect of alcohol, and (c) if it doesn't, feel free to come back here and clarify what you meant. And the grammar freak in me would like to point out that the word you probably meant to use was affect, although context may prove effect correct. Confusing Manifestation(Say hi!) 02:46, 13 March 2008 (UTC)[reply]
I enjoyed my joke, but he's probably referring to Prostate specific antigen levels.--The Fat Man Who Never Came Back (talk) 02:48, 13 March 2008 (UTC)[reply]
I enjoyed it too, and I agree that that's the likely candidate, but you know what they say about people who assume. Confusing Manifestation(Say hi!) 02:50, 13 March 2008 (UTC)[reply]
Don’t assume that we know what they say. — Knowledge Seeker 09:34, 13 March 2008 (UTC)[reply]
Assume--Shniken1 (talk) 10:16, 13 March 2008 (UTC)[reply]

About Albert Einstien (E=mc^2)

On deriving E=mc², did Einstien also come to know how to travel with the speed of light by converting mass into energy (energy in the form of electromagnetic radiation) and back to mass? i.e, did he know how to travel so fast that one could almost 'dissappear' from one place and 'appear' at another place almost at the same time?59.95.77.132 (talk) 05:34, 13 March 2008 (UTC)[reply]

The short answer is no, certainly not for "everyday" objects - there is usually an irreversible increase in entropy when mass is converted to energy. But see our article on teleportation for recent developments. Gandalf61 (talk) 07:17, 13 March 2008 (UTC)[reply]
E=mc² wasn't a process for extracting energy, it was an upper bound on the energy that might be extracted by any process. Prior to this the only way you could quantify energy was by the amount converted from one form to another. You could extract work from a substance by converting it into something else, but (aside from thermodynamic limitations) it was never clear whether you'd gotten all the work you could get out of it or whether there was some yet-undiscovered way of getting more. Einstein's argument suggested that an object of (inertial) mass m had a total of mc² energy, meaning that that was the most you could ever extract—after that there would be nothing left of the original object. It said nothing about whether there was actually any physical process that could extract that much energy. However at the time people had already noticed that radioactive substances seemed to release an enormous amount of energy—though very slowly—and I think people imagined or hoped that by studying and harnessing this process they could get the full mc² energy out of ordinary matter. It's now known that this isn't correct—radioactivity is just another kind of chemical process, involving nuclear bonds instead of atomic bonds, and just rearranges the constituents of matter without creating or destroying them—but nuclear energy seems to be permanently associated with E=mc² in the public consciousness. It's also now known that you can't turn the basic constituents of matter into useful energy without violating various conservation laws. Actually all of the laws that you need to violate seem to be only approximate, but the situations in which they're violated are pretty extreme, and it's very hard for me to believe that you could ever construct a useful (and safe!) power source this way. You can get plenty of energy from nuclear chemistry alone, and for that you just need the laws of nuclear chemistry, not E=mc². So although E=mc² is of major theoretical importance, it's not relevant to any practical technology. -- BenRG (talk) 13:44, 13 March 2008 (UTC)[reply]
Nuclear fission and radioactivity are not chemical processes. Nuclear chemistry covers different aspects of a nuclear power plant to the actual energy production. AlmostReadytoFly (talk) 14:48, 13 March 2008 (UTC)[reply]
Furthermore, massive particles such as beta particles, neutrinos and antineutrinos are created in radioactive decay. AlmostReadytoFly (talk) 15:02, 13 March 2008 (UTC)[reply]
And, uh, E=mc^2 is relevant to practical technology. There is a real mass differential between U-235 and its fission products, and the energy resulting from the fission reaction (mostly in the form of the kinetic energy of the repelling fission pieces) corresponds exactly to that "missing mass" times the speed of light squared. It comes into play in fusion reactions as well. To say nothing of atomic weapons! It's not an accident or an error that the public associate E=mc^2 with nuclear energy. Fission doesn't violate any conservation laws—total energy of the system is conserved even if some of it is in the form of nuclear bonds and then it becomes kinetic energy, etc. It does violate conservation of mass, but all of SR does that, and that's OK. --98.217.18.109 (talk) 22:14, 13 March 2008 (UTC)[reply]

Tell me if it' snot a good idea....

...to make an air filter based on the design of the human nose. That is, it would consist of several long tubes filled with fibers (nose hairs) extending from the walls, coated with a thick fluid dripping from the walls onto the fibers. Air would be blown through the tubes. The nostrils would need to be occasionally cleaned, perhaps with a silicone corkscrew that could be run through them once a day. Some type of antibacterial solution would need to be used in the drip, to prevent nasties from growing in it. StuRat (talk) 06:39, 13 March 2008 (UTC)[reply]

Snot for me my friend. Who gets to work the corkscrew? Julia Rossi (talk) 07:26, 13 March 2008 (UTC)[reply]
I suppose the high-end models could have this function automated. On the other hand, if the liquid drips through quickly enough, that may clean the nostrils out sufficiently without needing a corkscrew. StuRat (talk) 14:09, 13 March 2008 (UTC)[reply]
Underemployed telephone sanitisers.
Atlant (talk) 13:10, 13 March 2008 (UTC)[reply]
The nose is not designed primarily as a filter, and it doesn't make a particularly good one. I don't think we're gonna see this on Dragons' Den in the near future.--Shantavira|feed me 08:33, 13 March 2008 (UTC)[reply]
Citation please? Seems to me that noses are excellent filters. I don't know about you but whenever I'm inside where the air quality is lower, I'm breathing through my nose to filter our some of the dust etc. And yes the nose is of course an excellent olefactory device, but if I were to posit its main functions, smell would be 1A and getting filtered oxygen to the lungs would be 1B. Vranak (talk) 14:40, 13 March 2008 (UTC)[reply]

This is actually done. Many mechanical air filters are coated with a water-soluable "stickum". They then server their purpose for a while, get dirty, are washed up, recoated, and placed back in service for another cycle. Light oil is also used but requires a detergent or solvent during the washing-up phase. My sports car has just such a filter.

Atlant (talk) 13:12, 13 March 2008 (UTC)[reply]

This wouldn't be much different than blowing air past one of those hanging fly-strips and hoping all the nasties get stuck to it. They don't. You have two things working against you. You need an adhesive that will trap everything - which will be difficult to find. The adhesive cannot dry out as air blows past - which will be difficult to find. -- kainaw 13:16, 13 March 2008 (UTC)[reply]
I don't think any filter removes 100% of the particulates from the air with each pass. As for drying out, my idea was to have a reservoir of liquid which slowly drips over the fibers. To make the fluid last longer, it should be oil-based instead of water-based. Glue traps for mice use such an adhesive, and they stay tacky for quite a long time. To do double duty, the dripping oil could have a nice scent to it, like cinnamon or mint, to provide "aromatherapy" at the same time it cleans the air. StuRat (talk) 14:00, 13 March 2008 (UTC)[reply]
This makes me wonder about bubbling air up through a reservoir of sticky oil. By the time the bubble hits the top, much of the gunk will get stuck. No need to clean anything. Just dump out the reservoir and refill as necessary. -- kainaw 15:03, 13 March 2008 (UTC)[reply]
That might work, or it might just make a thick foam that fills the entire house. :-) StuRat (talk) 15:29, 13 March 2008 (UTC)[reply]
Isn't Kainaw's idea simular to how a bong works? However, those use water and get gunky relatively quickly. Some of our inventions based off nature work quite well (velcro) while others not so well (Ornithopter). But as for this invention...who nose? 206.252.74.48 (talk) 15:40, 13 March 2008 (UTC)[reply]
The difference is that bubbles containing water quickly pop as the water evaporates, but this is not so for bubbles made of oil. StuRat (talk) 16:46, 13 March 2008 (UTC)[reply]
Congratulations you’ve made it onto my list. --S.dedalus (talk) 22:44, 13 March 2008 (UTC)[reply]
Cool. StuRat (talk) 17:05, 16 March 2008 (UTC)[reply]

volume of space

may we say that the space has afinite volume but infinite surface,like Gabriel`s horn???thank youHusseinshimaljasimdini (talk) 11:18, 13 March 2008 (UTC)[reply]

You can say anything you like. It's a free Internet. However, there are existing arguments about what constitutes the "end" of space. Is it infinite? Does it have a boundary? Does it wrap back upon itself? -- kainaw 12:10, 13 March 2008 (UTC)[reply]
If space has infinite volume I would surmise that it has infinite dimension (radius?) and so would come to the conclusion that the outer surface would have infinte area. Nevertheless the concept of an outer surface of infinite radius is a difficult one to consider.87.102.94.198 (talk) 12:51, 13 March 2008 (UTC)[reply]
The general belief is that physical space doesn't have a boundary at all. -- BenRG (talk) 12:56, 13 March 2008 (UTC)[reply]
Husseinshimaljasimdini - I am not clear what you mean by "infinite surface". You seem to be assuming that the Universe has a boundary. As BenRG says, all common models for the topology of the Universe assume that the Universe does not have a boundary (don't be misled by the use of the word "horn" in models such as the Picard horn). On the other hand, you say that "space has a finite volume". Some observations of possible periodicities in the cosmic microwave background hint at a finite universe but this is not certain, and there are still plausible models in which the Universe is infinite. Possibly when you say "space" you may be thinking of the observable universe (which is finite) rather than the whole Universe ? Our article on the shape of the Universe has more details. Gandalf61 (talk) 13:08, 13 March 2008 (UTC).I got it .thank you all.Husseinshimaljasimdini (talk) 14:58, 14 March 2008 (UTC)[reply]
In case it was not clear, the original question referenced Gabriel's Horn, a known mathematical paradox; the unique shape of this figure results in infinite area surrounding a finite volume. I don't think this shape is widely regarded as a good analogy for the universe as we know it. Nimur (talk) 08:13, 16 March 2008 (UTC)[reply]

buisness

what is the importance of buisness to the society?nipet —Preceding unsigned comment added by Nipet (talkcontribs) 12:15, 13 March 2008 (UTC)[reply]

Take a look at The Wealth of Nations. It will point you in the right direction. – ClockworkSoul 13:28, 13 March 2008 (UTC)[reply]

Gases "dissolved" in liquids; fish breathing

Can someone explain to me, in layman's terms, how exactly oxygen is dissolved in sea water and how fish breathe with their gills? Is the oxygen in sea water in the form of tiny bubbles, which the fish's gills suck into their bloodstream in the same way that land animals absorb oxygen through their lungs? Or is the oxygen somehow chemically bound to water or other molecules in sea water as it is to hemoglobin in the blood, in which case the fish's gills would presumably perform a chemical reaction? Or is there some other mechanism involved? Thanks! Marco polo (talk) 13:20, 13 March 2008 (UTC)[reply]

Refer to solubility. To put it in very simple terms, consider water as being composed of many small particles (molecules) of water. These particles nonetheless have a lot of space between them, and molecules of oxygen are able to fill some of that empty space. This is the oxygen dissolving in the water. The way the oxygen then gets transferred into a fish's bloodstream is a similar process to how lungs work. Water is passed over the gills, and the oxygen diffuses into the blood where it is at a lower concentration than in the water. It doesn't require a chemical reaction on the gills to extract the oxygen. --jjron (talk) 14:11, 13 March 2008 (UTC)[reply]
Nitpick: diffusion across a cell membrane, in this case that of the gill's cells, is called osmosis. --Bowlhover 15:04, 13 March 2008 (UTC)
Wouldn't osmosis mean the water is diffusing, not the oxygen? DMacks (talk) 06:30, 14 March 2008 (UTC)[reply]

Well water is made partially of oxygen (It's H2O) but I don't think that is how they do it. I'm sure there is oxygen that is disolved in the water (not in the form of bubbles otherwise fish could breath out of water) but liquid, if that makes sense. I'm not sure how this happens since oxygen has to be REALLY cold before it becomes a liquid on its own. I wouldn't mind hearing an answer to this also. EDIT: Oh hey an answer TheGreatZorko (talk) 14:15, 13 March 2008 (UTC)[reply]

jjron's answer covers that aspect fairly well. As for how the air is initially dissolved in water, there are several mechanisms. Wave action at the surface forces tiny bubbles into the water, which then dissolve into the water. Plants in the water also give off oxygen as a result of photosynthesis, which is then dissolved in the water. Even still water with no plant life will absorb some oxygen at the surface. Ocean currents are critical to the distribution of this dissolved oxygen, which otherwise would be at a much higher concentration near the surface than deep underwater. Note that other gases in air, like nitrogen and carbon dioxide, also dissolve in water. There is an occasional problem where an area of water is low on oxygen, creating a dead zone. Aquariums which lack wave action or sufficient plants often add oxygen to the water by bubbling air through it. StuRat (talk) 14:29, 13 March 2008 (UTC)[reply]
oxygen has to be REALLY cold before it becomes a liquid on its own
Doesn't matter. Salt has to be REALLY hot before it melts into a liquid on its own, but you have no problem with salt dissolving in water, right? —Keenan Pepper 14:49, 13 March 2008 (UTC)[reply]
Check out Lake Nyos for an interesting gas-dissolved-in-water story. --Sean 14:51, 13 March 2008 (UTC)[reply]
I've looked at the Lake Nyos article several times before, and I can't help but wonder at the thought processes and value systems I see. I'd support the "lower the level" plan a lot more cheerfully if all of the planners' children lived downstream. Lowering the pressure, without reducing the gas load, seems, well, unwise. I mean, they already have a vent pipe which works on it's own, powered solely by the rising gases coming out of solution. The vent pretty much proves that we've figured out what we're dealing with. So, given that kind of supersaturation, what happens to the immense pressure in the lower levels if the water level is lowered by the proposed 20m? You are dropping the pressure by about 5 atmospheres, which will, incidentally, allow some of the dissolved gas down there to come out on it's own, and the rising bubbles will probably cause the impending-disaster-to-be-prevented to happen right then and there. I know, take it to the discussion page. -SandyJax (talk) 18:45, 13 March 2008 (UTC)[reply]
The "lower the level" plan has nothing to do with dissolved gas. It's about preventing a landslide and resulting flood that could kill tens of thousands. --Carnildo (talk) 21:56, 13 March 2008 (UTC)[reply]
...and it will create another massive gas outpour, which will....wait for it...kill tens of thousands. It might be a good idea AFTER the pumps have lowered the supersaturation level of the deep water. -SandyJax (talk) 20:42, 14 March 2008 (UTC)[reply]
I saw a movie where a pressurised water suit had a large amount of oxygen disolved in it. Can you breath water if enough oxygen is disolved in it?--155.144.251.120 (talk) 03:32, 14 March 2008 (UTC)[reply]
See liquid breathing. DMacks (talk) 06:32, 14 March 2008 (UTC)[reply]

International Space Station

So far the international space station has always housed 3 crew members. When the ISS is finished, will it house more crew members. (I don't believe our article says; I read through it but it is possible I missed it). ike9898 (talk) 16:26, 13 March 2008 (UTC)[reply]

According to this SPACE.com article, the final crew capacity will be six or seven people. --Bowlhover 21:27, 13 March 2008 (UTC)
Sometime during Expedition 19 next year, the crew will increase from 3 to 6. anonymous6494 05:23, 14 March 2008 (UTC)[reply]

IV nurse (medicine)

A question from Germany: What is an "IV nurse"? I have heared it in Grey's Anatomy ("IV nurse to ICU") What is "LR"? - a medical acronym. ("two liter LR running wide open") (Kind of a liquid for i.v. application to substitute a volume deficit.) --84.137.46.213 (talk) 20:28, 13 March 2008 (UTC)[reply]

Isn't a IV nurse a nurse who knows how to do intravenous therapy? --Bowlhover 21:18, 13 March 2008 (UTC)
and also assigned to carry out IV therapy, as opposed to other duties. (If five nurses are on a single hospital floor, they may all know how to do IV therapy, but only one might be assigned to be the IV nurse). - Nunh-huh 02:02, 14 March 2008 (UTC)[reply]
LR is lactated Ringer's solution. TenOfAllTrades(talk) 21:28, 13 March 2008 (UTC)[reply]
Thanks for the fast answers. --84.137.46.213 (talk) 21:45, 13 March 2008 (UTC)[reply]

What are the night-blooming vegetables?

We are interested in a comprehensive list of night-blooming vegetables. —Preceding unsigned comment added by Webstergl (talkcontribs) 21:09, 13 March 2008 (UTC)[reply]

If you include herbs, you might like to look into these: Silver Thyme, 'Alba' or white Eggplant, white pumpkins, Basil, Mint, Oregano according to[13]. Silver or reflective foliage and white fruits seem to be indicators. Bats and moths the agents. Are you thinking of starting an article here? hint hintJulia Rossi (talk) 23:29, 13 March 2008 (UTC)[reply]
If you include fruit, don't forget pineapple. —Keenan Pepper 05:35, 14 March 2008 (UTC)[reply]

if cocaine is just a molecule, why dont peeple synthesize it?

so if cocaine is just a molecule (picture to the right in that article) why don't peeple synthesize it, like they do with meth or lsd, which are also molecules right? why go thru all the trouble of stuffing it up some immigrants butt excuse my french to get it past border control thank you —Preceding unsigned comment added by 79.122.103.78 (talk) 22:10, 13 March 2008 (UTC)[reply]

Because synthesizing it takes time, effort, and some technical know how. Wisdom89 (T / C) 22:18, 13 March 2008 (UTC)[reply]
Even LSD and methamphetamine have starting materials, and those happen to be quite readily available. But as mentioned in Cocain#Synthesis, creating a drug from scratch would be difficult, problematic, and also very expensive. The only answer I can give you is to guess that unlike the other two drugs, there are simply no possible starting materials that occur naturally (or in pharmacies) in sufficient quantity to make synthesis cheaper than smuggling. Someguy1221 (talk) 22:18, 13 March 2008 (UTC)[reply]
(ec) Well, sneaking it past border patrol is actually pretty cost-effective for cocaine, which has a very high street value for its relatively small volumes, unlike LSD and methamphetamines (cocaine costs roughly 10X that for the same volume of meth, if I recall). So you only have to smuggle a small amount to make up for the price of smuggling and the dangers associated. In any case, there is a section on synthesis in the article: Cocaine#Synthesis. It's not cited but it sounds plausible to me: "Synthetic cocaine would be highly desirable to the illegal drug industry, as it would eliminate the high visibility and low reliability of offshore sources and international smuggling, replacing them with clandestine domestic laboratories, as are common for illicit methamphetamine. However, natural cocaine remains the lowest cost and highest quality supply of cocaine. Actual full synthesis of cocaine is rarely done. Formation of inactive enantiomers and synthetic by-products limits the yield and purity."--98.217.18.109 (talk) 22:20, 13 March 2008 (UTC)[reply]
(Of course, I am aware that the cost of the drug is directly related to the availability. But I'm just pointing out that when the price is that high the smuggling isn't all that bad a method, even though, of course, the price is that high in part because smuggling is the method.) --98.217.18.109 (talk) 22:22, 13 March 2008 (UTC)[reply]
The answer to these types of questions is almost always "Because it's not cost effective". If people could make big money synthesizing cocaine, they would presumably already be doing it. Friday (talk) 23:38, 13 March 2008 (UTC)[reply]
An important part of the answer is that setting up a cocaine-synthesis plant would require a lot of up-front investment for an uncertain return, while paying somebody to smuggle it doesn't require much investment at all. --69.134.115.242 (talk) 00:05, 14 March 2008 (UTC)[reply]
That's true, but there is still investment on smuggling. The cocaine doesn't just make itself (or bribe officials, or hide from the Americans, or whatever). --98.217.18.109 (talk) 03:38, 14 March 2008 (UTC)[reply]
Basically same reason as there is no industrial-scale synthesis of caffeine - it can be synthesised, but it is cheaper to extract it from organic sources. Gandalf61 (talk) 09:49, 14 March 2008 (UTC)[reply]
The organic pool is always an atractive source for compounds. The other point is that synthesis requires skilled people who will get less dangerous jobs elsewhere.--Stone (talk) 09:59, 14 March 2008 (UTC)[reply]
Talk:Caffeine#Artificial_caffeine suggests that caffeine as an additive is sometimes or maybe even often/usually synthetic. But Caffeine#Production says (and cites) that it's so easy to extract (and so much is extracted anyway) that it's usually natural. DMacks (talk) 11:10, 14 March 2008 (UTC)[reply]
Cocaine suggests there are problems with current methods. It's likely that these could be solves but as people have already mentioned, existing natural sources are good enough that there is I presume no incentive. Note that the those in charge of the grower side of supply as well as anything else that would be affected by synthesis would obviously not be happy with any threat to their profit and may actively supress any attempts to develop methods to sythesise cocaine if they expect they will be significiantly cut out from the new market Nil Einne (talk) 15:55, 15 March 2008 (UTC)[reply]

March 14

physic

richard fenyman —Preceding unsigned comment added by 71.10.27.69 (talk) 00:14, 14 March 2008 (UTC)[reply]

We just happen to have an article on Richard Feynman, if that's who you're looking for.--VectorPotentialTalk 00:17, 14 March 2008 (UTC)[reply]

Quantum Physics

Why does each atom have different allowed energy levels for its electrons? I understand that each electron is actually a "particle-wave" and its orbits are only the ones allowed so the electron-wave won't cancel itself. So, why are there different orbits allowed for each compound and atom? Every electron is the same so how are there different orbits in each atom that don't make them cancel? For example, in one atom the orbits may be consecutively 2eV then 5eV, but in another, there may be an allowed orbit of 3.5eV where in the other atom this would not have been allowed. Why is this? Many thanks, Zrs 12 (talk) 00:18, 14 March 2008 (UTC)[reply]

The article on atomic orbitals will give you the full answer on this. However, my simplified answer to this is that the charge in the nucleus is different for each type of atom. Consequently, the electric potential energy is different in different atoms. The full quantum mechanical solution for atoms more complex than helium is not trivial and cannot be expressed as a neat exact mathematical formula. Numerical solution techniques must be used. For hydrogen, an exact expression is possible as derived in the Bohr model but this relies on simplifications that cannot be applied to more complex atoms. SpinningSpark 00:47, 14 March 2008 (UTC)[reply]
Mathematical side note: Roughly speaking, the calculation of energy levels involves solving the time-independent Schrödinger equation
where is the energy to be solved, is the potential energy of the electron. The depends on different kind of atom since is the integral of electrostatic force experienced by the electron and is affected by the proton number in the nucleus of different kind of atoms. Therefore, the difference in makes different set of energy levels solved. - Justin545 (talk) 02:05, 14 March 2008 (UTC)[reply]
And atoms with multiple electrons, have the electrons influencing each other. The Pauli exclusion principle applies as electrons are fermions. It forces electrons to adopt different positions in the atom. Graeme Bartlett (talk) 00:53, 14 March 2008 (UTC)[reply]
For a very simplified explanation, see the energy levels article SpinningSpark 01:15, 14 March 2008 (UTC)[reply]

What species of seal or walrus is this?

http://www.youtube.com/watch?v=DDg7kWgs5e0 --Sonjaaa (talk) 02:17, 14 March 2008 (UTC)[reply]

It is definitely an eared seal, meaning either some species of sea lion or - less likely - some species of fur seal. That leaves a choice of about 17 species. Many of the archetypal performing pinnipeds in circuses etc. are California sea lions. --mglg(talk) 16:03, 14 March 2008 (UTC)[reply]

RL circuit

I am working on a physics laboratory project dealing with an RL circuit (the resistor and inductor are in series with a battery). I am trying to find out the work done by the battery over a period of time from when the circuit is connected, and the energy dissipated from the resistor in that same time period. I realize that to find the work done I need to integrate the expression for the power (P=IV) over time, but I'm not sure how I ought to go about doing that. —Preceding unsigned comment added by 134.129.115.87 (talk) 02:19, 14 March 2008 (UTC)[reply]

The work done by the battery is equal to the energy dissipated in the resistor. Is that any help? —Preceding unsigned comment added by SpingMander (talkcontribs) 02:36, 14 March 2008 (UTC)[reply]
I don't think that's right. The battery also did work creating the magnetic field around the inductor; the magnetic field represents potential energy which can be returned to the electrical circuit at a later point in time. Integrated over infinity, your answer is correct, but it's not correct over some discrete, finite portion of time.
Atlant (talk) 12:23, 14 March 2008 (UTC)[reply]

Ozone in the interstellar medium

Molecular oxygen has been found in the interstellar medium. Why has ozone not? There is UV radiation in the ISM from nearby stars, surely this would produce ozone? Is it just that it has not yet been discovered. NB: I know that ozone is short lived but if there is O2 and UV radiation continously present, you will get ozone....right? --Shniken1 (talk) 03:38, 14 March 2008 (UTC)[reply]

You also need sufficient density of O2 to generate the ozone, which is unlikely. — Lomn 04:53, 14 March 2008 (UTC)[reply]
Especially, you need a sufficient fraction of the total gas being O2, because else the atomic oxygen produced by photodissociation will also react with other atoms or molecules, and only a small fraction with O2 to form O3. Icek (talk) 22:25, 14 March 2008 (UTC)[reply]
Additionally, the formation of ozone needs a 3-particle collision to conserve the momentum (maybe it could - but with a much lower probability - be conserved by the emission of a photon), whereas the destruction of ozone doesn't; this is another reason for the virtual non-existence of ozone in the low-density interstellar medium. Icek (talk) 06:59, 15 March 2008 (UTC)[reply]

what decides the direction of rotation of diesel engines?

section created AlmostReadytoFly (talk) 10:20, 14 March 2008 (UTC)[reply]

what decides the direction of rotaion of doesel engines--Shoby 2001 (talk) 07:09, 14 March 2008 (UTC)shoby 14 mar 08[reply]

4 stroke or 2 stroke? (just kidding) Seriously it depends on the timing of the injections of fuel into the cyclinders - which in turn must match the arrangement of the crankshaft. Also the starter motor must start in this direction.
If you are still bemused I recommend you read a piece on "how diesel engines work" search for this.. Once you know how they work it the answer should be obvious.
Need more explanation?87.102.83.204 (talk) 11:51, 14 March 2008 (UTC)[reply]
Aren't some supercharged/turbocharged diesel designs valve-less? Assuming you inject the fuel at Top Dead Center, they'll run in either direction. And aren't some marine diesel engines designed to run in both the "forward" and "reverse" directions?
Atlant (talk) 12:29, 14 March 2008 (UTC)[reply]
Your experteese not mine - all I can say is that assuming more than one cylinder I'd still have to control the timing of the fuel injections of the cylinders differently for each rotation direction. eg I'm assuming a 4 cylnder crank has the offsets at 0,90,180,270 degrees?
I'm willing to learn more though, or be wrong.87.102.83.204 (talk) 12:45, 14 March 2008 (UTC)[reply]
(correct myself) If the fuel injections are controlled by tappets on the crank then yes - I think you're right that they would work both ways..
As for 'valveless' there need to be something preventing the diesel gas escaping back out (ie a one way valve) - I don't have full understanding of the exact terminology87.102.83.204 (talk) 12:47, 14 March 2008 (UTC)[reply]
No, that's the interesting thing with a diesel: The only thing that goes in is air and the only thing that goes out is exhaust. (Unlike an Otto engine that forces in an expensive fuel-air mixture.) Because of this, you can arrange the engine so that it simply has valveless exhaust and inlet ports in the cylinder very near the bottom of the stroke. The super/turbocharger (the "one-way device" you mentioned) then rams a whole lotta air in, forcing out the previous cycle's exhaust gas. The piston then rises, closing off the ports and compression begins. It's not the most volumetrically-efficient way to arrange the engine (because 1) lots of extra volume of air is pushed through the engine and 2) because some of the stroke is "wasted" until the piston passes the ports) but it sure is simple and reliable.
Also, injection pumps used to be driven by a gear-train from the crank so that's effectively the same as if they were driven by tappets on the crank.
Atlant (talk) 16:26, 14 March 2008 (UTC)[reply]
Yes. I was thinking in terms of a four stroke 'otto like' diesel - two stroke will work in reverse as you say. So it looks like that the starter motor will determine the rotation in the case of a 2 stroke.
For the reasons you mention I always find it suprising that 2 strokes are more efficeint than 4 strokes. Maybe the 1/2 utilisation of each stroke for power in a 4 stroke tips the balance.87.102.83.204 (talk) 16:42, 14 March 2008 (UTC)[reply]
Ahh, here's one article I was thinking about: Opposed piston engine. I should have thought of it sooner seeing as how I took one of the pictures that graces that article ;-),
Atlant (talk) 23:30, 14 March 2008 (UTC)[reply]
And here's something similar (with animation!) Napier Deltic 87.102.21.171 (talk) 09:57, 15 March 2008 (UTC)[reply]

Knee problems

Does anyone know why traditional Zulu's don't have knee problems? I have searched everywhere and come up with nothing related. Leepylal (talk) 07:53, 14 March 2008 (UTC)[reply]

Where did you hear they don't? - Nunh-huh 08:32, 14 March 2008 (UTC)[reply]

I'm doing a course in exercise science. They make the statement and ask what factors contribute to this phenomenon. —Preceding unsigned comment added by Leepylal (talkcontribs) 10:40, 14 March 2008 (UTC)[reply]

Haven't a clue but people more in line for knee problems (but don't seem to have any to report) are the Maasai for jumping. Julia Rossi (talk) 11:21, 14 March 2008 (UTC)[reply]

try this http://pharyngula.org/index/weblog/print/1572/ specifically comment number 27 ? Does that help87.102.83.204 (talk) 11:47, 14 March 2008 (UTC)[reply]

The idea 'use it or lose it' comes to mind. Perhaps these traditional Zulus use their legs a great deal traversing the veldt, so they are kept in good repair. Vranak (talk) 21:02, 14 March 2008 (UTC)[reply]

Omnipresent God

Does evidence of the superposition of particles that would collapse if observed, such as elelctron difraction, not rule out an omnipresent god, as if he was all seeing and all knowing, he would see mhich gate the electron went through, and therefore destroy the wave diffraction pattern. —Preceding unsigned comment added by 81.140.151.57 (talk) 09:34, 14 March 2008 (UTC)[reply]

It's not quite true that electrons intrinsically know that they are being observed. It's simply the case that all of our detection methods alter the electron's behavior, though I'm sure someone can better describe how this works.
That's right, but please note that it is not the clumsiness of measurement that causes the alteration. It is a necessary consequence of the measurement no matter what method is used. Detection requires energy from the detected particle to be transferred to the detection device. Transfer of energy necessarily means the particle must change state or even be destroyed. If you are trying to detect a train, the change in energy is inconsequential. If you are trying to detect fundamental particles you must find them to be in one of the allowed quantum states, which are usually radically different from one another. SpinningSpark 11:53, 14 March 2008 (UTC)[reply]
When you get into God and logic, there really is no falsifiability as any number of explanations could be used to allow for exceptions. Perhaps God is incorporeal. Perhaps an all knowing God doesn't need to observe the electrons since He already knows its paths. — Ƶ§œš¹ [aɪm ˈfɻɛ̃ⁿdˡi] 09:51, 14 March 2008 (UTC)[reply]
The point the original poster was probably trying to make is that according to mainstream interpretations of various phenomena that are studied in quantum mechanics there is no fact about certain things except that they are in a superposition. And if Bell's Theorem is correct, there cannot be underlying variables which we are either ignorant of or unable to detect due to imprecise equipment.
So the OP wants to know if the universe operates like this, with certain characteristics of matter being in a superposition without clear facts about the certain characteristics other than the fact they are in a superposition, then does that invalidate the notion that a deity knows everything. This conception of the universe is at odds with what most Western religions have thought for quite some time; and it also really perplexes some philosophers of physics. Traditional theology probably defined omniscience as knowing everything that is knowable, so you can't merely object that God can't know false things. This definition seems to evade the worry the OP mentioned since, if there is no fact of the matter in respect to certain characteristics of certain pieces of matter then there is not anything that God would fail to know.--droptone (talk) 16:38, 14 March 2008 (UTC)[reply]
Theology isn't really my subject, but I don't see why omniscience would imply knowing properties of a system that, according to quantum mechanics, the system doesn't even have. What quantum mechanics says is that a system can at a given time only have some properties out of the collection of all properties a similar system might have; having certain properties precludes having certain others. (Very roughly speaking it can have half of all properties at a time.) If you know which properties it has, you can in principle measure the values of those properties without affecting the system. If you try to measure a property that it doesn't have, you're effectively forcing it into a state where it has that property at the expense of some other property it used to have, and it's in that sense that you're altering it. There's nothing that would preclude a deity knowing which properties a system has at any given time, and the values of those properties. -- BenRG (talk) 11:58, 14 March 2008 (UTC)[reply]
Indeed. Asking which slit the electron went through is like asking what colour is Thursday. If a question has no answer then even God cannot answer it. He could change the universe so that the question did have an answer, but He chooses not to. Gandalf61 (talk) 12:40, 14 March 2008 (UTC)[reply]
I see that as a particularly poor example of trying to diminish any respectability to the question, the fact is in two slits experiment if you measure which slit the electron went through, even if you do this by measuring it's "wake" as such, it did go through a slit. But if you dont measure, then the superposition occurs, and patterns of interference with itself are clear. Thursday does not have a colour, even if you try and measure it. If god genuinely saw everything (wether this is a to rigorous definition of omnipresence is another question) then he would know which slit the electron went through and no interference pattern would be made. —Preceding unsigned comment added by 81.140.151.57 (talk) 09:44, 15 March 2008 (UTC)[reply]
Well, anyway, this is the old "Does God play dice with the universe?" question, really. Which has as much to do with your ideas about God as it does what flavor of quantum mechanics you ascribe to. Omniscience is pretty poorly defined, in the sense that its actual physical properties are usually not defined, not to the degree necessary to make sense of quantum mechanical implications . Perhaps He would know the possible future outcomes for all quantum configurations? Is God an "observer" in the physical sense or in some other, supernatural sense? Who knows? It isn't exactly a rigorous set of terms. --98.217.18.109 (talk) 12:45, 14 March 2008 (UTC)[reply]
It really depends on which god you're talking about. There's this Nordic one I just heard about that might be able to do exactly what you're looking for... Imagine Reason (talk) 00:45, 16 March 2008 (UTC)[reply]

Indian pharmaceutical companies

Which pharmaceutical company is first in India Org ratings & how much turn over per year —Preceding unsigned comment added by 121.247.109.61 (talk) 10:49, 14 March 2008 (UTC)[reply]

Hope you don't mind, I turned down the volume. No caps lock please. Julia Rossi (talk) 11:23, 14 March 2008 (UTC)[reply]
maybe this http://iis-db.stanford.edu/pubs/11893/Smith.pdf will have the answer (p24+?)87.102.83.204 (talk) 11:43, 14 March 2008 (UTC)[reply]
or Category:Pharmaceutical_companies_of_India should help if you do the neccessary research.87.102.83.204 (talk) 11:44, 14 March 2008 (UTC)[reply]
PS what is "India Org"?87.102.83.204 (talk) 12:16, 14 March 2008 (UTC)[reply]

Chemical Process Pipe Labeling

I need the following question answered: When having a mixture of two or more chemicals in a vessel and the mixture is being tranfered to another vessel thru process piping what type of labeling is needed or required? Im not looking for the ansi color code standard, but the exact nomenclature to label the process pipe.

The mixture does consist of the following:

BioDiesel, Methanol, Glycerin, and Sulfuric Acid

So how do I make the correct label in correlation to the most hazardous,the most concentration, specific mixture?

Or do I need to label with all materials within the mixture?

Thank you George Leighlitner CTIBioFuels (email address removed to prevent abuse) Richard Avery (talk) 14:57, 14 March 2008 (UTC) —Preceding unsigned comment added by 66.212.144.50 (talk) 14:30, 14 March 2008 (UTC)[reply]

Right I honestly think you are taking the piss here. I apologise for my frank reply.
http://www.ctibiofuels.com/ which I assume is your company appears to be a legitimate business. We are volunteers and are not paid. I would not expect any professional company to be relying upon the advice of unpaid volunteers to give what is essentially health and safety advice - a matter which most likely is covered and governed by applicable laws in your region.
Typically such safety and safe working practice issues are taken very seriously and failure to comply with them can result in criminal prosecution. As such I would imagine that it is your responsibility to hire or employ someone who is professionally qualified and certified to give you advice on this matter.
Please do not rely on volunteers for this information.87.102.83.204 (talk) 15:33, 14 March 2008 (UTC)[reply]
You should include the words "Run away!" somewhere in your labelling scheme. --Heron (talk) 20:03, 14 March 2008 (UTC)[reply]
Maybe one pipe has Sodium Hydroxide in it instead of Sulfuric acid ? Polypipe Wrangler (talk) 05:56, 15 March 2008 (UTC)[reply]

Evolutionary biology and sexual selection in humans

Hi. I remember reading that studies are now showing that a significant percentage of women trick men into raising children who are not theirs, but I can't find any such article now. Is there a reliable source I can go to for the figure? Thanks. —Preceding unsigned comment added by Imagine Reason (talkcontribs) 15:58, 14 March 2008 (UTC)[reply]

Unfortunately, our article on cuckold has no worthwhile references. -- kainaw 16:04, 14 March 2008 (UTC)[reply]
Mmh women who trick their husbands.. I doubt it would ever be possible to get a reliable figure from people who carry out such a deceit.? Also did you specifically want women who 'trick their men' or to include all step-father type upbringings eg accident, drunk, uncontrollable fling, don't know exactly who the father is,,, etc.?87.102.83.204 (talk) 16:55, 14 March 2008 (UTC)[reply]
What the OP is asking about is called "extrapair paternity", and we have a little bit on it at Incidence_of_Monogamy#Incidence_of_genetic_monogamy. --Allen (talk) 17:23, 14 March 2008 (UTC)[reply]
To clarify, "extrapair paternity" doesn't apply to stepfathers, but only to cases where the mother's partner does not know he is not the genetic father. That's because rates of extrapair paternity are used to study how males behave differently (e.g. put less effort into raising their partner's offspring) when they cannot be sure that they are the genetic father. --Allen (talk) 17:33, 14 March 2008 (UTC)[reply]
The studies looking at this looked at DNA (or maybe just blood types, can't remember) of parents and children.. and found very high rates of cuckolding (maybe 1 in 4).. however the methodology was very flawed.. as the "fathers" were a self selecting group who wanted to know if their child was legit.. so unsurprisingly their suspicions were often founded much more often than you'd expect from the general population. The stats were publicized by an agency which specialized in this sort of test. I don't know if any "legitimate" studies have been done in this area. I also don't have any references. —Pengo 04:36, 15 March 2008 (UTC)[reply]
I think that was the study that I read about. A shame, then, that it is not so reliable. Imagine Reason (talk) 15:18, 15 March 2008 (UTC)[reply]
It boggles my mind the bad methodology that one often sees in such sorts of surveys. You'd think someone involved would have seen the very obvious problem with that approach. --98.217.18.109 (talk) 20:31, 15 March 2008 (UTC)[reply]
Maybe the less conscientious scientists do it more often. It certainly gets them into the news. Imagine Reason (talk) 00:47, 16 March 2008 (UTC)[reply]
The conventional rate of "non-paternal events" in a (human) genetic lineage is 10%, though more recent studies suggest a lower rate of about 5%. Even at the lower rate, it is more likely than not that any given chain of 14 generations has been interrupted by an (otherwise unsuspected) non-paternal event. - Nunh-huh 04:57, 15 March 2008 (UTC)[reply]
I trust your numbers, but "non-paternal event" seems to include adoption, stepfather relationships, and other situations where the mother's partner knows he is not the genetic father [14]. So they wouldn't play the same evolutionary role as unknowing extrapair paternity. --Allen (talk) 13:36, 15 March 2008 (UTC)[reply]
No, these are unexpected non-paternal events within a "known" genealogy. These are non-paternal events unsuspected by genealogists, and in most instances it's reasonable to presume also unsuspected by the (falsely) identified father. - Nunh-huh 01:56, 17 March 2008 (UTC)[reply]

3000 Farad "ultracapacitor". Is it a joke?

I've recently seen this [15] on a webpage. Could it be true? A 3000 Farad capacitor? Shouldn't it be larger than the Earth? And needing a Dyson sphere to power it up?

Oops, sorry, I see we have an article about it.. so they must exist for real. So how can it be, that it didn't solved our energy problems? As I know, the storage of power is one of our biggest problems, that's one of the causes why we are trying to develop superconductors. To store solar energy for the night. To store energy in cars and other vehicles. To create man-portable particle beam weapons and railguns. To stop wasting fuel by heating cities at nighttime and cooling them at daytime. --V. Szabolcs (talk) 19:07, 14 March 2008 (UTC)[reply]

It exists, but farads do not equal joules. You can only store a small amount of energy in one of those things, and you can only inject or extract a small amount of power. They have an amazing capacitance per unit volume, but not a very high energy per unit volume or per unit mass. If you look at our energy density article, you will see that a typical ultracapacitor has an ED of about 0.02 MJ/kg, compared to a lead-acid battery with about 0.1 MJ/kg. --Heron (talk) 19:48, 14 March 2008 (UTC)[reply]
It only operates at 2.7 volts. 3000 farads at that voltage can only store 2250 mAh of charge. About the same as a regular AA cell. SpinningSpark 20:33, 14 March 2008 (UTC)[reply]
Yes, but the nice part is that the charge/discharge cycle is substantially more efficient with an ultracapacitor and you can run it hundreds of thousands of times, unlike with rechargeable batteries that do something like 1,000 cycles. For this reason, ultracapacitors are starting to figure into the power trains of hybrid vehicles.
Atlant (talk) 23:40, 14 March 2008 (UTC)[reply]
Point taken, Atlant, but ultracaps haven't yet been economically scaled up for big applications like load-levelling of wind and solar power. (Ultracaps are used in the internal works of some wind turbines, but not for load-levelling, as far as I know.) My virtual money is on flow batteries for really big installations. --Heron (talk) 14:03, 15 March 2008 (UTC)[reply]
Actually, I saw a talk recently about a demonstration project using ultracaps for fluctuation management on a ~70 MW wind farm. Dragons flight (talk) 16:37, 15 March 2008 (UTC)[reply]

Windy metro entrances

Why is there a strong airflow at the ground-level entrances to subway stations? 91.156.148.161 (talk) 19:51, 14 March 2008 (UTC)[reply]

Two reasons come to mind. First, to ensure a constant supply of fresh, clean air at platform level, some stations may use forced ventilation to draw in clean surface air. Second, moving subway trains act as large piston heads within their tunnels, forcing air out of the tunnel ahead of them and drawing in air behind. Both of these factors can mean that a subway station will be at a higher or lower air pressure, resulting in a constant (sometimes strong) airflow. TenOfAllTrades(talk) 19:56, 14 March 2008 (UTC)[reply]
I don't actually know anything about this (other than many years commuting) but from experience the wind at the entrances seems to me to be particularly steady and therefore unlikely to be anything to do with the trains pushing the air along. This is not the case on the platforms where clearly the train is having a marked effect. Could it be that underground heat is forcing the air to rise? The air is certainly very much warmer underground. One can imagine that the air escapes through one entrance and is replaced by colder air coming in through another lower entrance (possibly even at a different station). The constriction at the entrances would magnify the air velocity so even a quite small effect of this sort could produce a strong wind at the entrance. SpinningSpark 20:46, 14 March 2008 (UTC)[reply]
Forced ventilation. There are utility buildings that house huge fans. People ruin "fresh air" pretty fast, between body heat, O2-CO2 respiration, water vapor addition, smoking, body odor, engine exhaust, you name it, we go through air FAST. And, large groups of people go through it faster. So, any restricted area (read as "small cave filled with people") will have massive air turnover systems designed in. And, rather than have three openings (people access, air in, and air out), it is cheaper to have the people opening also serve as one of the air ducts. Next, since the whole purpose is to maintain a constant supply of fresh air, and the people opening is right next to where the bus is idling on the street, belching blue smoke, it is standard to use the people opening as the air exhaust. Go to any subway. You'll note that the wind is coming FROM the subway TO the opening on the sidewalk. Now, how did I get logged out, and where did the SignBot go? -SandyJax (talk) 21:58, 14 March 2008 (UTC)[reply]
Thanks for the explanations! Yeah, the air indeed flows from inside the metro station to the opening. 91.156.148.161 (talk) 07:10, 15 March 2008 (UTC)[reply]
There is another possibility, although from the description of the air always blowing out, I doubt if this is what's happening here. In a large cave system, with only tiny openings to the outside, wind will blow in or out to equalize the pressure whenever the surface barometric pressure changes. At times this can cause strong winds at cave entrances or narrow points inside the cave. I suspect that this effect is slight in the subway, in comparison with the forced air system, due to the large number of entrances to any subway. StuRat (talk) 18:26, 16 March 2008 (UTC)[reply]

Gustatory table of elements

Do molecules, and more specifically elements, have flavours? Could you arange complex molecules in a taste map similar to the colour thing, where they fade into one another? I mean to say, do chemically similar elements taste similar? I think I'm phrasing it badly but that's the best I can do really. 81.96.160.6 (talk) 20:57, 14 March 2008 (UTC)[reply]

Yes, but I believe it would be insufferably difficult to do. However, salts taste salty, acids taste sour, bases taste bitter, most sugars taste sweet, etc..etc.. Wisdom89 (T / C) 21:03, 14 March 2008 (UTC)[reply]
There were informative answers here on a previous question regarding the taste of salts which concluded that all salts do not taste salty. There is some grouping by chemical species but the overall picture is clearly much more complex.
If you were looking for a taste equivalent of colour space it really does not exist. For one thing, human colour vision has only three receptors making the colour space possible to plot graphically. Taste has many more (given that smell forms a large part of taste). Secondly, the receptors are still being discovered and documented and a definitive "map" in terms of the fundamental sensations is not yet possible. And it would need to be multi-dimensional even if all the information were now available. SpinningSpark 22:01, 14 March 2008 (UTC)[reply]

Speed of fire ant destruction

How fast can a bunch of fire ants devour something? I guess it depends on how many there are, so let's say a door is opened and a big room full come out into another room; could a person be threatened? This is a a parody story of a soap opera I'm helping a person write, and I'm using an old idea I had years ago as a kid about an Evil Overlord threatening to take over the world using fire ants to blackmail it. Obviously, I don't need to know exactly, but just ballpark figure of what they could do.

As a side note, I recall as a kid, seeing on...maybe That's Incredible on TV, how a colony of fire ants can devour an animal, all but skeleton, in minutes. But, that's pretty vague, too, it was long ago I saw it. Thanks.63.3.19.1 (talk) 21:18, 14 March 2008 (UTC)[reply]

This isn't a science reference but I guess the author researched it – there's an informative scene of fire ant destruction in the Congo in Barbara Kingsolver's novel The Poisonwood Bible though our article is brief and doesn't mention it. Fire ant mentions them in relation to small animals. Julia Rossi (talk) 23:10, 14 March 2008 (UTC)[reply]

Can you look up this physical property for me? I've exhaused my available references

I need to know the surface tension of a saturated solution of magnesium nitrate. Can you help? If so, I also like to know where you found it... Thanks! ike9898 (talk) 21:57, 14 March 2008 (UTC)[reply]

It's possible that someone has measured this - if you can find chemical Abstracts you could look it up there. It's equally possible I think that this has never been measured before.87.102.21.171 (talk) 09:52, 15 March 2008 (UTC)[reply]

Thunderstorms

What's the capacitance of a thunderstorm? --Carnildo (talk) 21:58, 14 March 2008 (UTC)[reply]

Not at all sure that anyone has ever analysed thunderstorms in terms of capacitance. However, to have a go at this question, the thunderstorm article states that an average thunderstorm contains 3.6 x 1013 joules of energy. The lightning article tells me that 109 volts are required to break down air in a 1000 ft lightning strike. Using the equation (see capacitance article);

yields,

Which is a piddling 72 μF. Even a big storm with 100 times the energy only comes out to a surprisingly small 7200 μF. It is turning out this way because the square of a very large voltage is an even more enormous number. Whether anyone has any "real" figures for thunderstorm capacitance I don't know. SpinningSpark 23:51, 14 March 2008 (UTC)[reply]

Um, let me correct that, I was misreading the energy figure from the thunderstorm article which is the energy of the storm, not the electrical energy. The lightning article gives 5x108 joule for the energy of an average lighting strike. So translating that to capacitance yields;
Even smaller. SpinningSpark 00:08, 15 March 2008 (UTC)[reply]
Another possible approach is to calculate the capacitance directly from the geometry of the storm. Probably, the closest model is a spherical storm above a ground plane. But since some pretty wild assumptions are being made, we might as well use the slightly simpler parallel plate model. For a 500m radius storm, 300m above the ground,
putting in results in
I make the result for the spherical/ground plane model;
just over half the parallel plate answer. SpinningSpark 11:26, 15 March 2008 (UTC)[reply]
Being creatively lazy, I just Googled for the answer. The top hit for "thunder cloud capacitance" is an interesting PDF [16] on the subject. It arrives at a figure of 60 nF, so you guys are in the right rounders field.
Digression: That PDF also has an calculation of the power in all the world's thunderstorms, which turns out to be just enough for the electrical needs of one large city. I'll try to remember that fact for the next time someone asks how to harness the power of a thunderstorm. --Heron (talk) 11:55, 15 March 2008 (UTC)[reply]
Actually, the 60 nF calculated in that reference is not the same capacitance. They have calculated 'self-capacitance'. That is, the notional capacitance between the storm and an inifinitely large sphere centred on it. On the other hand, my calcualtion above is the capacitance between the storm and ground. The next paragraph in that paper points out that the important capacitance as far as the discharge of lightning is concerned is the capacitance between the storm and the ionised channel once this has been struck. This capacitance they calculate at 9 nF for the same size storm. SpinningSpark 12:55, 15 March 2008 (UTC)[reply]

March 15

Quantum Physics II

  • How does a wave have a charge (like an electron)?
  • How does light (a wave-particle) and an electron (also a wave-particle) have differing masses if they are both particle-waves?
  • How does a wave have mass at all?

Zrs 12 (talk) 03:39, 15 March 2008 (UTC)[reply]

Mass and energy are equivalent. Anything with energy has mass. An electromagnetic wave does have charge, but positive and negative mixed together so that the result is neutral. Light is a traveling wave, confined to moving at light speed, whereas an electron is a wave packet, something like a standing wave, and therefore can travel at other speeds.
But the 'why' questions dig at the limits of what we know of the universe. — kwami (talk) 03:54, 15 March 2008 (UTC)[reply]
Yes I know , so why does the electron have a mass and photons not have a mass? They both contain energy, no? Zrs 12 (talk) 04:00, 15 March 2008 (UTC)[reply]
A photon has zero rest mass, not zero mass. 'Zero rest mass' is a fiction, another way of saying that it can't be at rest. The mass of a photon is its energy. — kwami (talk) 04:07, 15 March 2008 (UTC)[reply]
The concept of relativistic mass have gone out of use, kwami, rest mass is just an obsolete term for mass. Narayanese (talk) 09:41, 15 March 2008 (UTC)[reply]
Electrons aren't waves. They are electrons. They have wave-like properties, they exhibit wave-like behavior (in some circumstances). But they are not "waves" per se. (When you lose track of the metaphor and the thing-itself then you're heading into the territory of confusion.) --98.217.18.109 (talk) 14:59, 15 March 2008 (UTC)[reply]
According to this resource electrons are particle waves. Is UC Berkeley wrong or am I missing something? Furthermore photons are also wave-particles (photons). What am I missing? Zrs 12 (talk) 20:01, 15 March 2008 (UTC)[reply]
That's a theoretical claim which has not been demonstrated. — kwami (talk) 19:56, 15 March 2008 (UTC)[reply]
Well, don't confuse one professor's lecture at UC Berkeley with UC Berkeley the institution! I haven't the time to figure out where in that hour-long lecture the claim comes up but if I were his teaching fellow (and I have been a teaching fellow in similar classes), I would probably be explaining to my students the basics of the Copenhagen interpretation in response. Electrons are electrons. When we understand them with macroscopic metaphors our understandings will be in part imperfect. It is important to remember this. We can model their behavior like a wave in some situations, and we can model their behavior like a particle in others. (Wave-particle duality). But they are neither waves nor particles. They are electrons. They act like both waves and particles. See Complementarity (physics). There are, of course, other interpretations of QM, but frankly I find Copenhagen to be the most intuitively straightforward once you grasp it, as a non-scientist. It makes your questions unnecessary and removes the confusion of worrying too much about what it would mean for a wave to have a charge—the wave doesn't have the charge, the wave is just another function of what an electron is, just as the charge is part of that. An electric charge is just a fundamental property of the subatomic entity.
If you wanted to point me to the specific time in the lecture I'd be happy to take a look at it. My bet is that the professor is just trying to convey to the non-scientist students that electrons are like both waves and particles. Again, I think a more thorough explanation—if I were doing it, anyway—would be to separate out the epistemology of the electron (what we can know about it, how we can know it) from the ontology of the electron (what the electron really is), which is more in the style of Copenhagen, and in my non-scientist opinion, easier for non-scientists to grasp (personally I think scientists have very little clue what's easier for a non-scientist to grasp or how non-scientists learn, but that's just my own observation, being someone who has long been on the borders between science and the humanities). Regarding it as a "particle wave" is fine shorthand for remembering it has both properties but again, it's a metaphor, it's in a human language, it will have limitations. --98.217.18.109 (talk) 20:19, 15 March 2008 (UTC)[reply]
From 10:42 to about 16:30 he talks about electron waves orbiting the nucleus. This part is what I was talking about. Zrs 12 (talk) 22:21, 15 March 2008 (UTC)[reply]

can we produce 50,000 units of electricity from solar energy

Hi, I want to know wheather i can produce near to 50,000 units of electricity from concentrated type solar panels. I live in a tropical country with ample sun light . Please give me the precice amount of stream needed and the time it needs to be converted into stream.I would also like to know about the suitable dynamo needed with price margine.please help me this is my last resort and this is not an homework question.(e-mail removed to avoid abuse)--Manoj man1990 (talk) 06:21, 15 March 2008 (UTC)[reply]

Find out the amount of units of electricity you can get from a single solar panel in your country and work out how many you would need to get 50,000 units. Better still contact a solar panel supplier and ask them.
What do you mean by stream? is this about a water generation method?
87.102.21.171 (talk) 10:54, 15 March 2008 (UTC)[reply]
What unit? Saying "50000 units of electricity" is like saying there is 1000 units between New York and London. Doesn't say anything at all about the actual distance - the unit could be nanometers or parsecs for all we know! -- Aeluwas (talk) 11:27, 15 March 2008 (UTC)[reply]
I think the OP is referring to a solar thermal concentrator using steam as the heat transfer fluid. There are some links in solar thermal energy that may be of use. If by 'units' you mean watts, then the answer is yes, systems delivering tens of MW have been built. If by 'units' you mean kWh per year, then the answer is still yes, since 50,000 kWh per year is not that much. --Heron (talk) 11:38, 15 March 2008 (UTC)[reply]
Stream is a common (but usually incorrect, depending on language) synonym for "electricity" in many languages. For example, my own native language Dutch has "stroom". User:Krator (t c) 15:12, 15 March 2008 (UTC)[reply]
While anon could be more clear, when it comes to household/consumer electricity, Units of electricity nearly always means kWh. This is in most English speaking countries that I'm aware of including the UK, NZ, Malaysia and apparently the US as well. Household bills and electricity providers will often talk about units rather then kWh although I presume it is defined somewhere in the small print for legal purposes Nil Einne (talk) 15:38, 15 March 2008 (UTC)[reply]
But OP can't mean kWh, because "produce near to 50,000 kWh of electricity from concentrated type solar panels" would be meaningless. 50,000 kWh in how much time? --Heron (talk) 18:23, 15 March 2008 (UTC)[reply]
Unless they say otherwise, I'd assume they mean to produce 50,000 kW per hour. Of course, since solar power only works when the Sun's out, the next question would be how much beyond 50,000 kW must be generated at peak production to allow for it to be stored so 50,000 kW can be drawn continuously. This assumes that a continuous power supply is needed. For some tasks, like pumping water into a water tower, I suppose it's fine to only operate the machinery when the Sun is shining. StuRat (talk) 18:13, 16 March 2008 (UTC)[reply]

Observes & speed of light

I may have asked this question before, but if I did, I probably didn't understand the answer as it has been bothering me still.

An observer is in a train travelling at 10 m/s on a double track in space. Another train passes on the second track, travelling at 10 m/s as well. Through the side window, the observer sees that train as if it were moving at 20 m/s, causing a certain amount of motion blur etc.

Then, the train starts travelling at 160,000,000 m/s (or: slightly more than half the speed of light), and another train passes at 160,000,000 m/s. What does the observer see? User:Krator (t c) 15:23, 15 March 2008 (UTC)[reply]

I'm a bit confused as to how many trains there are and what you are describing. If one train is going 10 m/s then it can't be passed by another train going the same speed, unless you mean 10 m/s relative to the other train. Also, are talking about trains accelerating (from 20 m/s to 160,000,00 m/s) or moving at constant velocities? Because that changes how it is handled with relativity and etc.; acceleration is much more complicated than constant speeds. In other words, I think you need to clarify a bit of what you are asking. In any case, if what you are asking is will at anytime one observer see a train passing him at the speed of light, then no, he won't—he'll see the speed as being slower, one way or another (the amount of space passed in a given amount of time will never mean their velocity is c or greater—either the time will be measured differently or the amount of space will be measured differently, depending on how you look at it). --98.217.18.109 (talk) 16:16, 15 March 2008 (UTC)[reply]
If an outside observer would see the two trains as moving in opposite directions, both at 1.6×108m/s, then an observer in one of the trains would observe the other as moving at about 2.49×108m/s. See velocity-addition formula. Algebraist 17:58, 15 March 2008 (UTC)[reply]
The OP means that the other train is travelling in the opposite direction. His problem is that if both trains are travelling at more than half C then surely the velocity of one train observed form the other will be more than C? The answer is that no it won't. Velocities do not add linearly, so in this equation where u and v are the velocities of the trains and w is the velocity of one observed from the other;
is only approximately true when u and v are small compared to C. The correct formula for adding velocities under special relativity is;
From which it can be seen that w can never be more than C. The best it can do is equaal C if either u or v are C also. SpinningSpark 17:58, 15 March 2008 (UTC)[reply]
Two trains passing each other at 160 Mm/s will look the same, from the perspective of someone riding one train, as a train at rest being passed by a train going at about 249 Mm/s (all speeds measured with respect to the track). But neither case will look at all like a sped up version of the 10 m/s case. It will look more like this. High-speed physics isn't at all like a faster version of low-speed physics. Fundamentally the reason you can't "add velocities" is that the space of velocities is hyperbolic. When you're dealing with small relative speeds, you're dealing with a small part of the space that's approximately flat, so you can pretend it's exactly flat and use vectors. When you're dealing with large relative speeds this doesn't work any more. Hyperbolic geometry is unfamiliar, but there's a close analogy with spherical geometries like the surface of the Earth. When you're only mapping a small part of the Earth you can pretend it's flat, but you can't make a flat map of a large part of the Earth without distortion. It's the same with velocities. The usual meters-per-second velocities measured with respect to a "reference frame" are actually the Klein model of hyperbolic space, which is a kind of map projection. That article unfortunately has no pictures, but the Poincaré disk model is similar and has some pretty pictures. See also Escher's Circle Limit woodcuts. All of the like-colored shapes in those pictures are the same size; they just look different because of the projection. If you scale the Klein disk so that it has a radius of c, and say that O is the center of the disk, P and Q are points 160 Mm/s from the center in opposite directions, and R is a point 249 Mm/s from the center, then the real distance between P and Q (in the hyperbolic space) is the same as the distance between O and R. That's why those two cases of the passing-train problem are the same. They look different on the map, but that's because it's distorted near the edges. There's nothing deep about the "velocity addition formula"; it's just a side effect of the traditional use of the Klein projection, which is just a historical accident. -- BenRG (talk) 19:30, 16 March 2008 (UTC)[reply]
I'll munch on this answer for the coming month. Thanks! :) I've only done so much physics, but I think I'll be able to understand the above with the help of some physicist friends. User:Krator (t c) 22:42, 16 March 2008 (UTC)[reply]

Hi. This is not homework. So, an apple with a mass of 100 grams dropped from the height of one metre, negligizing air resistance, releases approximately one joule of energy? However, at what downward speed is the apple travelling at by the time it hits the ground? What about from a height of 10 metres? I know acceleration of gravity is just under 10 m/s, but how to calculate the velocity the object is travelling at by the time it hits the ground? If an object is thrown upwards, and say is is travelling at 40 m/s, will it be travelling at 30 m/s upwards after one second, and 20m/s after 2 seconds? Will it start to fall down after 4s, or is acceleration more complicated than that? Say is starts falling down, it will be travelling at 10 m/s after one second, but what about after 2 seconds, will it be travelling at 20m/s? How can we calculate based on height? Also, how does one joule compare to the equivalent of one kg of force? Does the density of the impacted surface usually affect the amount of energy released, such as in asteroid impact simulation given by LPL Arizona? If an object weiging 1 kg is travelling forward without downward interference from gravity, such as pendulum or rolling, and air resistance and friction is negligible, and travelling at 10 m/s, how many joules of kenetic energy? What about 1 kg at 20m/s? Is this calculation linear or exponential? If a (hypopthetical) bullet (average) was shot directly upwards, how high would it reach? If it was shot downwards from the altitude of 1 km from the ground, would it speed up by then because of acceleration or slow down because of air resistance? If you and a ball were falling out of the sky at the same altitude and the same velocity, say at 100m/s, can you safely catch the ball? If an object were to explode upon impact, caused by the impact alone and not some inner explosive force such as a grenade, so it's not a bomb, would the released energy be the same as if they didn't explode? If two bullets hit each other exactly head-on, with exactly the same mass and exactly the same speed, and were travelling exactly horizontal, and they have exactly the same volume and exactly the same density, and air currents and resistance were negligiable, what would happen? If a gamma-ray photon and a radio-wave photon hit each other head on, and didn't miss, would the resulting annilated matter, if any, travel more in the direction the gamma ray was travelling because higher energy, or is it impossible because different wavelengths? Can black holes implode? Have I asked this before? Thanks. ~AH1(TCU) 15:32, 15 March 2008 (UTC)[reply]

  • An apple with a mass of 100 grams dropped from the height of one metre, negligizing air resistance, releases approximately one joule of energy?
Correct.
  • However, at what downward speed is the apple travelling at by the time it hits the ground? What about from a height of 10 metres? I know acceleration of gravity is just under 10 m/s, but how to calculate the velocity the object is travelling at by the time it hits the ground?
It's 10 m/(s^2). velocity=acceleration*time, distance=time^2*acceleration/2. You know acceleration and distance. Solve for time, then velocity.
  • If an object is thrown upwards, and say is is travelling at 40 m/s, will it be travelling at 30 m/s upwards after one second, and 20m/s after 2 seconds? Will it start to fall down after 4s, or is acceleration more complicated than that? Say is starts falling down, it will be travelling at 10 m/s after one second, but what about after 2 seconds, will it be travelling at 20m/s?
That is all correct.
  • How can we calculate based on height?
If you're talking about calculating speed from hight, you already asked that.
  • Also, how does one joule compare to the equivalent of one kg of force?
I assume you mean the weight of a kg, which is about 10N. It's one-tenth of a meter times as much. That is, pushing with a force of 10N over a tenth of a meter uses one J. This is what happens when you mix units.
  • Does the density of the impacted surface usually affect the amount of energy released, such as in asteroid impact simulation given by LPL Arizona?
No, assuming mass, rather than volume, is constant, but it does affect the way in which it is released.
  • If an object weiging 1 kg is travelling forward without downward interference from gravity, such as pendulum or rolling, and air resistance and friction is negligible, and travelling at 10 m/s, how many joules of kenetic energy? What about 1 kg at 20m/s? Is this calculation linear or exponential?
Kinetic energy=mass*velocity^2. It is neither linear nor exponential.
  • If a (hypopthetical) bullet (average) was shot directly upwards, how high would it reach?
I don't know the speed of the average bullet, or the effects of air-resistance, but if you know the former and ignore the latter, distance=time^2*acceleration/2. Air resistance would be far from negligible.
  • If it was shot downwards from the altitude of 1 km from the ground, would it speed up by then because of acceleration or slow down because of air resistance?
Terminal velocity for a bullet is much slower than what speed it's fired at. It would slow down, and approach terminal velocity.
  • If you and a ball were falling out of the sky at the same altitude and the same velocity, say at 100m/s, can you safely catch the ball?
Air resistance would make a much larger difference for the ball, then for you, but if you ignore that, you'd both be in free fall and the ball would seem to hover, making it both safe (insomuch as falling out of the sky is safe), and extremely easy.
  • If an object were to explode upon impact, caused by the impact alone and not some inner explosive force such as a grenade, so it's not a bomb, would the released energy be the same as if they didn't explode?
The energy remains constant. Unless there's some way it's storing the energy, the amount released will be the same. Whether or not it explodes will make a difference in how the energy is released, and thus its destructive power.
  • If two bullets hit each other exactly head-on, with exactly the same mass and exactly the same speed, and were travelling exactly horizontal, and they have exactly the same volume and exactly the same density, and air currents and resistance were negligiable, what would happen?
They'd either bounce off of each other or fuse. I've seen a picture of two bullets that collided in midair and fused, so I know that one's possible. The other is probably also possible. I'd say it depends on the bullet.
  • If a gamma-ray photon and a radio-wave photon hit each other head on, and didn't miss, would the resulting annilated matter, if any, travel more in the direction the gamma ray was travelling because higher energy, or is it impossible because different wavelengths?
From some point of reference, they're the same wavelength. I think they can annihilate each other, but I don't know if they have to. If they do, momentum will be conserved, so the resulting matter/antimatter or whatever gets released will be moving more in the direction of the gamma ray.
  • Can black holes implode?
They're as imploded as it gets. If our current physics are wrong, and they can implode more, there'd be no way to tell, as no information about the internal structure can leave the event horizon. — DanielLC 18:02, 15 March 2008 (UTC)[reply]
AH1, are you taking the piss? SpinningSpark 18:09, 15 March 2008 (UTC)[reply]
What?!? This? *confusion*. ~AH1(TCU) 18:48, 15 March 2008 (UTC)[reply]
Never new that was Hiberno-English. Amazing what you learn on Wikipedia! SpinningSpark 19:08, 15 March 2008 (UTC)[reply]
It may be more common in Ireland, but it's plenty common in south England. Algebraist 20:56, 15 March 2008 (UTC)[reply]
And in New Zealand, too. Not an expression to use when your among Americans, who are likely to interpret it as a golden shower. StuRat (talk) 18:04, 16 March 2008 (UTC)[reply]
You never know your luck, though (if that's the sort of thing you like). You might have randomised yourself into the path of a fellow ... er, urinee, and if you don't put it out there, you might miss a "golden" opportunity. -- JackofOz (talk) 18:35, 16 March 2008 (UTC)[reply]
Hi. By the way, great answer to my question. However, what did you mean by asking if I am taking the piss?! I still don't get it, so before I list some ridiculous possibilities, can you explain what you meant originally by this so I can solve this mystery that has been plauging me for the past 90,000 seconds? Thanks. ~AH1(TCU) 19:32, 16 March 2008 (UTC)[reply]
It means something like "Are you teasing us ?". In the context of this post I would guess they think you may be asking so many question just to annoy us. You might want to ask fewer questions at a time, to avoid this type of reaction. StuRat (talk) 22:47, 16 March 2008 (UTC)[reply]

Effect of carbon dioxide on photosynthesis

Hi all have a question I couldn't find the answer to. I know that the carbon dioxide concentration in air is usually about 0.035% but plants can benefit from much higher concentrations, up to 0.5% I believe. If the concentration is increased further I have heard it has a detrimental effect on photosynthesis causing the stomata to close. My question is why does a large concentration of carbon dioxide in the air cause the stomata to close? Surely they would benefit from the carbon dioxide. Any help would be much appreiciated. —Preceding unsigned comment added by 172.201.120.93 (talk) 15:45, 15 March 2008 (UTC)[reply]

This is a very complicated problem; see, for example, abstract of this paper. I am afraid there is no simple answer to this one. Sorry. --Dr Dima (talk) 16:08, 15 March 2008 (UTC)[reply]

Wikimania - is it an academic conference?

Is Wikimania considered an academic conference? Are publications presented there published in peer reviewed outlets? --Piotr Konieczny aka Prokonsul Piotrus| talk 16:22, 15 March 2008 (UTC)[reply]

Not routinely, though some presenters will be academics who also choose to publish their work in academic journals. Dragons flight (talk) 16:26, 15 March 2008 (UTC)[reply]

Metabolism and conservation of energy

For people or creatures that have a fast metabolism, where is the additional energy they consume going? (Mostly I care about people, but one would think that general principles would be the same across species.) I know certain studies have found that people who fidget generally are skinnier, so maybe some of it is mechanical energy that is used in gross motion. Besides that it seems like you would either need to be passing improperly digested food or translating that energy into heat somewhere. But do people with high metabolisms really have higher body temperatures? (Or have dilated surface blood vessels so that they give off heat more quickly, or something like that?) Any insight into this would be welcome. Thanks! Mangostar (talk) 17:51, 15 March 2008 (UTC) (I just noticed I wasn't that precise. I understand that technically someone who fidgets more and is skinny because of it doesn't really have a faster "metabolism". Still, my general question is the same. I guess there are three categories that I'm seeing initially--dissipation of energy through motion, poor digestion (seems implausible), and various internal processes/metabolism that would generate heat... Mangostar (talk) 17:57, 15 March 2008 (UTC))[reply]

I could be thinking about this the wrong way myself, but I don't think there's such a thing as dissipation of energy through fidgeting per se... I think that has to be heat dissipation as well. Say you're moving your foot from side to side... when you start a leftward stroke, you're turning your body's chemical energy into kinetic energy (and heat, because of the inefficiency of muscles). But then you have to decelerate your foot (if you want to keep it), so you're turning that kinetic energy into... heat, as far as I can figure. It could be a bit of air motion, but it doesn't seem like that would account for much. So unless the air motion is more important than I'm guessing, then it does seem like people with faster metabolisms must have higher body temperature. But I'm not a physicist; I'm probably mixed up somewhere here. --Allen (talk) 20:48, 15 March 2008 (UTC)[reply]
As for the dissipation thing I just meant that there immediately you are creating mechanical energy. Obviously that is somehow transformed into mechanical energy in the air around you as well as heat in the air and in your muscles etc. I just meant to distinguish that from heat that is generated from chemical reactions in your body. Mangostar (talk) 22:45, 15 March 2008 (UTC)[reply]
Mangostar's original thought is on the right track - conservation of energy. Basically, the energy expended plus the energy stored by an organism must be equal to the energy ingested. In practice, estimating those quantities is difficult; for instance, the energy expended will include both mechanical energy and heat energy (as Amcbride noted, life here on earth has not yet developed muscles that can work in reverse by converting kinetic energy into chemical energy). As a general principle, if an organism uses more energy, it must consume more energy, so the short answer to your question is that people who have faster "metabolisms" must eat more food, but it's really more complicated than that. And I don't know about the body temperature thing, but it looks like the article Warm-blooded has some good info on that topic. --Bmk (talk) 20:58, 15 March 2008 (UTC)[reply]
I guess my big question here is, are people who are naturally skinny normally just hotter than people who are naturally fat (assuming they eat the same amount of food and move the same amount)? To the extent it's true that some people are "naturally" thin in that their bodies are just extremely inefficient at energy (i.e. fat) storage, where is all that energy going? Mangostar (talk) 22:45, 15 March 2008 (UTC)[reply]
I'm not sure if that has anything to do with your question but when I eat cheese or fatty meat I sweat. So does my brother. People sometimes ask me if there is something wrong so maybe it is not that common. I don't know. 200.127.59.151 (talk) 01:19, 16 March 2008 (UTC)[reply]
I don't know the answer, but I would like to add that this is a really good question.
It's well known that different people have higher and lower metabolic rates. People with lower metabolic rates are in a sense "more efficient" at utilizing their food, in that they waste less of the energy derived from their food and therefore either need less of it or put on weight faster. There's also the terribly unfair result that if you're overweight, and go on a diet and eat less food, your body decides that you're "starving" it, and decreases its metabolic rate further, nullifying your efforts.
But Mangostar is absolutely right: that energy has to go somewhere. And it pretty much has to be released as waste heat -- that's generally the end result of any energy-transfer process.
I'm not aware that people with higher metabolisms have higher temperatures, but if we think about it, that couldn't be the answer anyway; if the excess energy just went into heating you up, you'd get hotter and hotter until you incinerated. So the heat has to be lost to the environment. (Now, it's true, given a constant ambient temperature, hotter objects lose heat to the environment faster, but I can't see a difference of a degree or two in human body temperature making much of a difference here.)
Perhaps people with a higher metabolic rate breathe faster, and thus lose more heat in the warmed air they exhale. —Steve Summit (talk) 15:09, 16 March 2008 (UTC)[reply]
This is just ignorant speculation, but maybe we are looking at this from the wrong way around. Thinner people have less fat to provide insulation, and hence it seems likely that they will dissipate internal heat more rapidly. Maybe skinny people need to burn energy more rapidly than the rest of us simply to maintain the same body temperature. Dragons flight (talk) 17:19, 16 March 2008 (UTC)[reply]
I had a grandmother who must have had a migh metabolic rate, because she was a "ball of energy". She would vaccuum every day, go to the grocery store every day, etc. I imagine she burnt the energy as heat which was dissipated as she rapidly moved around the home, to the store, etc. A person in motion should transfer heat to the air more quickly than one who stays still. If they sweat then evaporative cooling will lower the temp, so there the energy is also going into vaporized water, just like a flame under a pot of boiling water. StuRat (talk) 17:56, 16 March 2008 (UTC)[reply]

Skeleton

File:Skelotor.JPG
What could this be
File:2008 0301klklk0129.JPG
Could it be one of theses

Hi All.
I came across this skeleton at the Middle Head Fortifications in Mosman, New South Wales. Does anyone know what this could be. Cheers. --User:Adam.J.W.C. (talk) (talk) 23:36, 15 March 2008 (UTC)[reply]

It might be a bat, though that little "hand" would have to be a foot of that were the case. Has more teeth than most species of bat, I reckon. --98.217.18.109 (talk) 00:17, 16 March 2008 (UTC)[reply]
Apart from the head, I though the rest of it looked quite human, I suspected that it could be a bat due to the fact that there is some kind of black colored skin or something wrapped around the skeleton. The only animal that I have come across in abundance in that area are hairs and rabbits. Cheers. --User:Adam.J.W.C. (talk) (talk) 00:31, 16 March 2008 (UTC)[reply]
The forelimb is absolutely not bat-like. It would be easier if there was a better view of the head. Check that. Rather, the front teeth on the bottom jaw (which is pointing straight down) look rather rodent-like. But that's a bit of a guess. -- Flyguy649 talk 01:31, 16 March 2008 (UTC)[reply]
It might not be a forelimb. Imagine it is a hind limb. It's not unlikely that the skeleton is jumbled up. I thought it looked rather rodent like, but I've never seen rodents with long arms like that. However bat legs can be quite long. (I am no mammal expert, though.) --98.217.18.109 (talk) 04:10, 16 March 2008 (UTC)[reply]

I guess one should not discount any one of a small number of Australian animals that have the general appearance of rodents - bandicoots, possums, gliders, potoroos etc., and with which we are not familiar in the skeletal state. Definitely not a bat with a 'hand' like that. Richard Avery (talk) 08:31, 16 March 2008 (UTC)[reply]

Possum is a real possibility. Bandicoot, potoroo, no way. Glider, can't tell, not enough to compare with. You can google things like "Possum skull" and see if it looks anything like the skull show there, it's an easy way for ruling things out (doesn't always rule things in, though). The more I look at it, the more I think the top jaw is broken—it is lacking symmetry with the bottom one and would have a tremendous underbite as is. --98.217.8.46 (talk) 15:44, 16 March 2008 (UTC)[reply]

The Great Gazoo ? :-) StuRat (talk) 17:36, 16 March 2008 (UTC)[reply]

The lower incisors tempt one to say it's a rodent, but there is no diastema (dentistry). This question was posted in various places, and I think the most clueful reply is at Talk:Rat#Who_is_skelotor. William Avery (talk) 19:21, 16 March 2008 (UTC)[reply]

I'm pretty sure its a sugar glider Petaurus breviceps, might be another species. I tell by the broad hands (all four much the same) with parallel fingers bunched into fists. I recognise the anatomy from removing them from water tanks through the bottom 25mm tap hole. This is why you must always cover the overflow pipe with netting. Polypipe Wrangler (talk) 23:27, 16 March 2008 (UTC)[reply]

March 16

Toxicity of non-stick pan lining?

I know it's not the recommended methd of cleaning them, but I often use rough scrubbing pads on my cheap no stick pans. The coating does flake off slowly. If I am ingesting this material in small quantities, should I be worried?--68.237.2.101 (talk) 03:15, 16 March 2008 (UTC)[reply]

See Polytetrafluoroethylene. Basically if it's Teflon and hasn't been overheated it is chemically inert and should pass straight through your body. --antilivedT | C | G 03:42, 16 March 2008 (UTC)[reply]

I personally develop a bad skin condition whenever I eat something cooked in Teflon, so it can't be entirely harmless. And no overheating is not the issue. Dupont has a vested interest in having you believe their stuff is safe if used properly, so what they tell you cannot be trusted.

If I were you I'd err on the side of caution and get some regular cookware -- stainless steel, cast iron, copper if you're wealthy. But it really depends on how concerned you are about your health and whether doing a little extra scrubbing to clean normal cookware is something you'd be willing to do. If you aren't a big-time health nut it's probably not something you need to worry about. Vranak (talk) 15:20, 16 March 2008 (UTC)[reply]

This is a contentious issue, and as far as I know there isn't a definitive answer yet. As Vranek notes, the manufacturer has a vested interest, so don't believe everything you hear.
Here are the facts as I understand and observe them:
  • Teflon and related compounds are quite biologically inert; they'll pass through you like a rock.
  • "Burnt" Teflon is reasonably nasty, and is to be avoided.
  • The non-stick coating on non-stick pans does flake off. More so with cheap pans.
  • The non-stick coating is much more likely to flake off it it's been overheated, i.e. burned. Danger, Will Robinson.
  • The non-stick coating is easily damaged (and made much more prone to both burning and flaking) by the use of metal implements, either while cooking or cleaning.
My own conclusion is that the potential toxicity of burnt-and-flaked-off Teflon is suitably worrisome that it's worth avoiding.
If you want to clean your cookware so vigorously that the non-stick coating flakes off, pretty soon it isn't non-stick any more, and you might as well use non-non-stick cookware, instead. Then you don't have to worry.
Personally, I don't like non-stick cookware, because I can't be bothered to be careful of it. (Plastic spatulas are teh pitz; they melt if you leave them in.) I find that with proper cooking techniques, my ordinary (non-non-stick) cookware doesn't stick badly enough to worry about. YMMV. —Steve Summit (talk) 15:36, 16 March 2008 (UTC)[reply]
Also, steel wool is absolutely indispensible for regular cookware. Vranak (talk) 17:00, 16 March 2008 (UTC)[reply]
And why would you use steel wool on a non-stick pan ? It's guaranteed to destroy it. You might as well polish your car with steel wool. StuRat (talk) 17:31, 16 March 2008 (UTC)[reply]
I said it's good for regular cookware. i.e. non-non-stick cookware. Vranak (talk) 19:33, 16 March 2008 (UTC)[reply]
Right. I wasn't replying to you; please note the indentation. Apparently the OP is also using steel wool, or something just as bad, on non-stick pans. StuRat (talk) 22:39, 16 March 2008 (UTC)[reply]
(edit conflict)Hi. I've watched infomercials on TV way too often, and on one of them, there is a solution to all your problems: SmartWare. Try Googling it or something. Remember, this offer is not avalible in stores. Don't delay, order your SmartWare today! Call within the next 30 seconds and recieve my special bundt cake pan, as well as our cake stencil: Turn a ho-hum cake into a yum-yum cake! You should have at least two to three bowel movements a day. Less than one bowel movement a day is irregular, and can lead to constipation. (Dr. Ho) Hope this helps. Thanks. ~AH1(TCU) 19:39, 16 March 2008 (UTC)[reply]
Yes, it seems as if infomercials are full of Ho's selling crap. StuRat (talk) 22:41, 16 March 2008 (UTC)[reply]

Horizontal search functions (in visual perception)

Does anybody have a clue what could be meant by the expression horizontal search functions in the following sentence: "Targets that yield horizontal search functions are assumed to reflect visual primitives, the basic building blocks of perception." (PS this is a sentence in my textbook, not a sentence found in Wikipedia.) Lova Falk (talk) 15:11, 16 March 2008 (UTC)[reply]

It's just a guess, but they may refer to a heirarchy of objects, like the following example:
  Humans
  |    |
Men  Women
     |   |
   Barb  Sue
So, a horizontal search would be a search at the same hierarchy level. For example, "Is that Barb ?", "No", "Then is that Sue ?", etc. StuRat (talk) 17:26, 16 March 2008 (UTC)[reply]
This sounds very close to a classification project I was working on. In my project the term "horizontal" was used literally to suggest the conjunctive sense of "and" when a glyph was used as a modifier of another glyph. "Vertical" on the other hand was used to suggest the conjunctive sense of "or". Thus a word is considered a "horizontal" set of glyphs and a sentence is then a horizontal set of words, whereas alternate glyphs, words, phrases, and sentences are deemed "vertical".

It would help if you gave us a little more background on the textbook and the section in question. One isolated sentence from any textbook is going to be hard to make sense of without context. Give us the whole paragraph and the title of the book, at least. Otherwise people are going to be just guessing blindly. --98.217.8.46 (talk) 18:12, 16 March 2008 (UTC)[reply]

Is Energy relative?

This is perhaps a very silly question, but nevertheless, it has troubled my from time to time. What's wrong with the following line of reasoning:

  • Velocity is relative (depends on the reference frame)
  • Therefore, its time derivative, acceleration is relative
  • Force, which is acceleration times mass, is relative (or, we can say, momentum and its rate of change are relative)
  • Thus, quantities like kinetic energy (m*v^2/2) and work (force x displacement) are also dependent on reference frame.

Is this last proposition correct? deeptrivia (talk) 16:02, 16 March 2008 (UTC)[reply]

Yes, but the second and third aren't. Acceleration isn't inherently a frame-dependent thing. A speedometer can only tell you the relative speed between two objects (for example, your car and the road), but an accelerometer can tell you its own intrinsic acceleration without reference to anything else. Inertial navigation is based on this principle. -- BenRG (talk) 18:37, 16 March 2008 (UTC)[reply]
So, there is such a thing as an absolute acceleration? If so, how can we ever find it? For example, the earth itself accelerates as it revolves around the sun, and the sun might be accelerating around the center of the galaxy. Is it that these effects are small, and in principle measurable? Do the sensors we use include all these accelerations? deeptrivia (talk) 21:16, 16 March 2008 (UTC)[reply]
Yes these small accelerations are absolute and measurable. The accelerations ,that are a direct consequence of the gravitational forces in the universe, are making the changes in velocity that make astronomical bodies orbit each other, rather than move in a straight line as they would do if no force operated on them. GameKeeper (talk) 23:16, 16 March 2008 (UTC)[reply]
It's a key tenet of general relativity that accelerations are not relative except that they can't be distinguished from gravitational fields. You can use an accelerometer to measure accelerations; the most simple of which is a cup of coffee. Start to accelerate it, just a little—you'll start to spill it. But if you have it in an inertial frame, no matter how fast you are going (say, 67,000 miles an hour), it'll sit there placidly.
Einstein used to comment that theory of relativity was something of a misnomer: what's important is not what is relative, but the very few things which are not relative, like the speed of light. It is from finding the few things which are not relative—and the implications of that—that the genius of his theory comes. --98.217.8.46 (talk) 00:59, 17 March 2008 (UTC)[reply]
KE and momentum both depend on the reference frame

DBasing the money supply

Wouldn't it be possible to enter the serial numbers that are already on US currency into a database, record the serial numbers of any currency that is stolen in the DB, scan all bills whenever they are tendered at a business and run a database check, and thus instantly know when any stolen money is tendered ? The weak point in the chain seems to be getting businesses to pay for the devices, but I imagine some tax incentive could be offered to get them to agree. StuRat (talk) 17:21, 16 March 2008 (UTC)[reply]

Such a system could easily be built based not on a piece of new equipment but merely on the upload of a digital picture of a bill using a regular scanner. This would also help everyone including the Secret Service to determining which bills were counterfeit. If yours turned out to be counterfeit then of course you would be stuck with the loss unless you could spend it quickly somewhere a scanner was not in use and the system did not require your name and to turn in the bill if it was found not to be legitimate. —Preceding unsigned comment added by 71.100.174.10 (talk) 17:43, 16 March 2008 (UTC)[reply]
I don't quite understand what you mean, StuRat. What is preventing the counterfeiter from copying legitimate serial numbers onto fake bank notes? --Bowlhover 18:01, 16 March 2008 (UTC)
Serial numbers are unique and associated with other components of a bill which can not be duplicated exactly. Pictures of every bill issued already exist. Of two bills with the same serial number figuring out which one is real and which one is not is a relatively insignificant problem. —Preceding unsigned comment added by 71.100.174.10 (talk) 18:11, 16 March 2008 (UTC)[reply]
I was talking about the theft of valid money, not countefeit money. However, the system could be somewhat useful in apprehending counterfeiters, too. If, for example, somebody tries to spend a large amount of currency and several of the serial numbers are listed as being in the possession of other banks and businesses, it might look suspicious and security might want to detain the big spender until the police arrive. StuRat (talk) 18:38, 16 March 2008 (UTC)[reply]
The potential cost of maintaining and using a database and a system that would hold information for all of the billions of US bills in circulation would be prohibitive and probably outweigh whatever is lost in counterfeiting. The labor required to enter in every bill one received and, presumably, all of the bills that one gave out (how else would you know which ones were stolen?) would be unreasonably large. Is it possible? Yes, sure. But wildly impractical. There's no way the gains would be worth the costs. --98.217.8.46 (talk) 18:02, 16 March 2008 (UTC)[reply]
And just to illustrate: say I run a business, even one that deals with a relatively small amount of money, like a convenience store. Let's pretend I have an automated machine that'll scan all of the money I put into the till, and registers it in some sort of giganormous Treasury database which always works (despite store millions of new transaction entries a day from around the country) and is miraculously fast enough to not make every bill check take as long as a credit card takes to process (about 10 seconds a card swipe or so, so if you paid for a $5 item in ones then you've got to sit around for a minute). (And note: yes, the business WOULD need to check with the centralized database every time they received a bill for change, because they too are also in the process of looking for "stolen" money from other businesses, right?) OK—best case scenario so far?
Now a customer comes, and I have to take the money they give me, scan it, and then take the money I give them, and scan it. Because I don't keep track of what I've taken out of the till, when my store gets robbed, I won't be able to distinguish between "valid" money that I gave out as a change and "stolen" money. Let's even pretend that this was somehow practical, and the system says that a mile away, someone put one of my five dollar bills into a till at a grocery store. The police are called, the suspect kindly waits for them to arrive, and everybody has a nice conversation about where that five dollar bill came from. Maybe, in an ideal world, the suspect matches a perfect description from the robbed cashier, they put him in a line-up, they identify him, they convict him. Bravo. But what if he doesn't match? What if, say, he got the five dollar bill from someone else? OK, now we've got a complicated situation here—maybe we try to shake him down and find out where he got that bill from. And so on and so on. The question is: how many times do you have to "catch" the wrong person for 1. people to think this is a waste of time, 2. businesses to think this is a waste of resources, 3. law enforcement to think this is a waste of resources? Not many. --98.217.8.46 (talk) 18:07, 16 March 2008 (UTC)[reply]
Okay you want special equipment. NCR can build a cash register with a bill scanner inside above the drawer. You put the bill in the drawer and the cash register does the rest. As for a waste of time... such a system has the potential to waste a crook's time rather than the police by simply going the extra step of requiring validation of the bill before it can be spent. Bill gets stolen the system is notified by pressing the "theft" key and poof the bill is no longer valid until re-certified by the police. —Preceding unsigned comment added by 71.100.174.10 (talk) 18:16, 16 March 2008 (UTC)[reply]
There is the voluntary project, http://wheresgeorge.com -- kainaw 18:17, 16 March 2008 (UTC)[reply]
Too many holes to be worth the effort. —Preceding unsigned comment added by 71.100.174.10 (talk) 18:23, 16 March 2008 (UTC)[reply]
Still a ridiculous suggestion. Do you think the police want to spend their time becoming "bill re-certifiers"? No law enforcement agency in the world is going to want to take on that task. The best way to catch criminals involves talking to other people, looking at specific forensic, not trying to track down each and every physical dollar. --98.217.8.46 (talk) 20:28, 16 March 2008 (UTC)[reply]
Thanks for confirming everyone's concept of the police as worthless lazy donut eating taxpayer supported freeloaders and do nothings until the opportunity arises to write a parking ticket or a ticket for driving near enough to the speed limit that in their minds it might as well be speeding. Not to mention being self-righteous oppressors of Berkley student freedoms. —Preceding unsigned comment added by 71.100.174.10 (talk) 20:57, 16 March 2008 (UTC)[reply]
I tend to think of police that way myself, as the only interaction I seem to ever have with them is when they extort money from me via tickets. Since I am neither a pretty woman good at flirting nor another policeman or government official immune to traffic laws, I never get out of a ticket, either. When anyone is driving like a fool and actually endangering lives, no police are ever to be found. I even saw one instance where police apparently put up a barracade and stopped traffic with the sole purpose of ticketing those who drove around it. StuRat (talk) 22:30, 16 March 2008 (UTC)[reply]
Many of the limitations listed are only current limitations, which can hopefully be solved soon. I've never understood why credit card approval takes so long, and increasing bandwidth, etc., should make this process quicker soon. Also, rather than "dialing in" for each bill, something like a DSL system can be always on and ready for a check. Looking up a unique serial number in a database is one of the quickest DB operations there is. Initially, perhaps only banks would record the location and distibution of currency, with businesses merely running a check, not updating the database. A few years later on, businesses could update the records, too. Note that the full system could also be used as an anti-terrorism tool, as terrorists often need to move large sums of money around. If they go to a bank and withdraw large sums of money, the serial numbers could be tracked and we would know, say, if they used it to buy weapons and explosives. It would also be interesting to know if large sums of money given to Islamic charities are being used to purchase weapons. StuRat (talk) 18:50, 16 March 2008 (UTC)[reply]
The whole thing is totally impractical with paper money. And at least in the United States, no arms dealers are going to start using such a system—a lot of the transactions would be "off the grid", especially for those involving potentially scandalous or illegal activities. There are better ways to check how businesses and charities spend their money if one is inclined to do so.
It doesn't matter whether reading the index of a bill is a quick operation—even a relatively quick operation needs to be able to scale. And every complex system is going to have major downtimes, bugs, etc. in it.
My money (haw haw) would be on using electronic money and getting rid of paper money long before any of this would ever become practical, and I still think that even if you did have a system of tracking paper money it would still be wildly impractical. Certainly more impractical than just making everyone switch to purely electronic currency and skipping the paper step altogether. If countries make universal IDs mandatory (which I suspect they will in the next decade or so), that would make it all the easier. --98.217.8.46 (talk) 20:24, 16 March 2008 (UTC)[reply]
Certain "high-risk" businesses, like gun shops, pawn shops, etc., could be forced to comply by law. It would be easy enough to verify, just have an undercover cop go in with a "bad bill" occasionally, and, if they don't report it and refuse to sell the gun, give them a whopping huge fine. This method would allow the police to make a nice profit, ensuring that they would actually enforce the law. StuRat (talk) 22:21, 16 March 2008 (UTC)[reply]
(And if you're worried about people buying guns, why not make them register the guns? There are far less guns than dollars, and they're much larger, much more worth the effort. If you think people don't want you to track their gun-buying habits, or that they can find ways around that check, why would you assume money would be any easier?) --98.217.8.46 (talk) 20:30, 16 March 2008 (UTC)[reply]
People don't usually leave guns around after a crime. And, if they want to get around the tracking, it's a lot easier to remove a serial number from a gun than from every bill you ever have. StuRat (talk) 22:12, 16 March 2008 (UTC)[reply]
What has been learned from previous experiments to track money is that criminals create their own from of money and find other ways to thwart tracking like through the construction, purchase and sale of real estate. A "company" buys a lot for twice what it is worth, builds on it and sells the lot and the building for eight times the going price of any adjacent property. People are so open to a free market under capitalism they do not even bat an eye. Money gets laundered like this around here all of the time and no even cares.
This might help track down the laundering operations, too. If an armored car is robbed, then, a year later, somebody buys a $100,000 home for $200,000 in cash, and many of the bills are those stolen from the armored car, the buyer would have some explaining to do. StuRat (talk) 22:15, 16 March 2008 (UTC)[reply]
Why would you assume that they'd be using that money anyway? If a ridiculous tracking system went into effect, what's to prevent someone going outside of the country to launder it? You severely overestimate the technical abilities needed to do such a thing while at the same time you underestimate how easy it would be to get outside of the grid. Again, there's nothing your system would offer that a generalized electronic money system would not, except that yours would cost billions more to produce and maintain in both infrastructure costs and operation time, and because you still were trying to rely on physical money you make it exceptionally easy to evade your proposed system. In any case, armored cars are not robbed all that often and the companies that run them are insured anyway (you're using a very clunky hammer to try and hit a pin of a problem). --98.217.8.46 (talk) 00:55, 17 March 2008 (UTC)[reply]
Moving currency offshore in large quantities has it's risks, too, as anyone repeatedly smuggling large sums of cash is likely to be caught. The armored car was just an example, it doesn't really matter where the currency was obtained illegally. And, whether they are insured or not is quite irrelevant, it only means the insurance company now wants the cash recovered instead of the original target. StuRat (talk) 01:07, 17 March 2008 (UTC)[reply]

artificially macerated food

As an informal experiment I have been macerating my meals using a food processor. The consistency of maceration is about the same as bean dip used for tacos but includes meats, vegetables and most components of a regular meal including condiments. I consume a beverage with the macerated meal. Not only do I become full with only half a meal but have been able to reduce the number of meals to one per day with occasional in between unmacerated light snacks. Not only do I seem to have more stamina for excercise but the pounds are beginning to come off. Where can I find scientific or medical studies which discuss artificially macerated food? —Preceding unsigned comment added by 71.100.174.10 (talk) 17:34, 16 March 2008 (UTC)[reply]

The only way I can see that helping is if the food is less palatble in that form, so you eat less of it. StuRat (talk) 18:54, 16 March 2008 (UTC)[reply]
It is somewhat less palatable but not to a degree that it is something one can not get use to in light of other benefits. What I need is a list of studies which have explored the question in depth. —Preceding unsigned comment added by 71.100.174.10 (talk) 21:14, 16 March 2008 (UTC)[reply]
I'm not aware of any studies. But what exactly would be the mechanism by which you would desire to eat less, if not that the food is unpalatable ? I suppose that might still be better than diet/weight loss methods like Alli, which apparently causes weight loss because people who take it crap their pants unless they stop eating fat. Talk about unpalatable ! StuRat (talk) 22:03, 16 March 2008 (UTC)[reply]
Check with aged-care studies, where macerated food (often reset in more attractive moulds) is used.Polypipe Wrangler (talk) —Preceding comment was added at 23:34, 16 March 2008 (UTC)[reply]
This question sounds strikingly familiar. --The Fat Man Who Never Came Back (talk) 23:58, 16 March 2008 (UTC):-)[reply]

How much human evolution is yet to come?

Is there any evidence regarding how close human evolution is to reaching an end state where it will stay? How much improvement is left to be made over the next few million years, assuming the planet remains habitable that long? NeonMerlin 18:22, 16 March 2008 (UTC)[reply]

That's easy. Whatever will make the human physically and mentally more efficient. An adaptation to the ever increasing abundance of pornography, for instance, might be a third hand growing out of one's thigh. ;D —Preceding unsigned comment added by 71.100.174.10 (talk) 18:25, 16 March 2008 (UTC)[reply]
As long as the environment changes, evolution will cause change in the species that survive. Even if the environment on Earth becomes static (which is not likely), humans are trying to go to other planets, which will lead to evolution to match the foreign environment. -- kainaw 18:41, 16 March 2008 (UTC)[reply]
Unfortunately, human evolution is likely to take a nasty turn, where those who irresponsibly produce the most children and abandon them are likely to pass on the most genes. Historically, the abandoned children would die, but in modern society, they are taken care of by the state. In the long run, however, the state will no longer be able to care for the expanding pool of abandoned children, and they will no longer survive. Or, perhaps before we get to that point, some fairly radical solutions, like forced sterilization, will become acceptable. StuRat (talk) 19:00, 16 March 2008 (UTC)[reply]
By chance, Stu, have you seen Idiocracy? Someguy1221 (talk) 20:36, 16 March 2008 (UTC)[reply]
No, I haven't, since doing so won't tend to increase the number of times I reproduce. :-) StuRat (talk) 21:53, 16 March 2008 (UTC)[reply]
And spending time on Wikipedia does? o_O Someguy1221 (talk) 23:38, 16 March 2008 (UTC)[reply]
Irresponsibility is not a genetic trait. There's no reason to assume those particular children are going to be any stupider than others. The complaint about "abandoned children" (or the mentally disabled, or the physically disabled) overburdening the state has been a nice scare fantasy of eugenicists for over 100 years now; the state seems to be doing just fine in that respect. It is pretty unlikely that compulsory sterilization will ever become in vogue again in Western countries. --98.217.8.46 (talk) 20:20, 16 March 2008 (UTC)[reply]
I don't agree that irresponsibility isn't a genetic trait (the Kennedy family may make my point for me). But, even if we say it isn't, children who are abandoned are also less likely to grow up to be responsible parents due to the lack of a good role model. As for abandoned children being cared for "just fine", this certainly isn't the case. In Romania, for example, they have a high fatality rate in underfunded and often abusive state-run institutions. The kids are often misdiagnosed as having severe mental and physical problems, which, while not true initially, becomes a self-fulfilling prophecy the longer they remain there. For example, since they are judged incapable of learning, they aren't given access to education, which, in turn, leaves them illiterate. The story isn't much better in the US, where children are often shuffled from foster home to home, some of which are abusive. Also, 100 years is an insignificant amount of time on the scale of human evolution, so I'm not surprised that the negative aspects haven't yet become overwhelmingly apparent. StuRat (talk) 21:47, 16 March 2008 (UTC)[reply]
I say "just fine" from the standpoint of the species and the standpoint of the survival of the state. This is a conversation about evolution, right? Abandoned children make up an insignificant part of the human gene pool, as does all of Romania. Even the United States doesn't stand for a whole lot on a biological level. If you want to talk about biology, talk about biology. If you want to talk about society, okay, talk about society. But don't pretend you are talking about one thing when you're talking about the other. For all of your talk about the effects of modern society on the long-term prospects of the human species, your views are tailored to very small issues and completely absent are any considerations that would take into account, say, the most populous societies on the planet. Don't wrap your ideology up in scientific terms and pretend it is science—that's another bad legacy of the eugenicists for you to avoid. --98.217.8.46 (talk) 00:51, 17 March 2008 (UTC)[reply]
When did I say Romania and the US were the only countries having trouble caring for abandoned children ? It's a problem in many countries. If you want to wait until we get to the point where nearly all abandoned children die to see it as a problem, then it will be too late to avoid the worst consequences by then. StuRat (talk) 01:22, 17 March 2008 (UTC)[reply]
You might be interested in the higher evolution.--Shantavira|feed me 19:23, 16 March 2008 (UTC)[reply]
I won't argue StuRat on intellectual grounds but I may say that his view strikes me as a touch pessimistic.
As to the original question, a quote from Hagakure:
A certain swordsman in his declining years said the following:
In one's life, there are levels in the pursuit of study. In the lowest level, a person studies but nothing comes of it, and he feels that both he and others are unskillful. At this point he is worthless. In the middle level he is still useless but is aware of his own insufficiencies and can also see the insufficiencies of others. In a higher level he has pride concerning his own ability, rejoices in praise from others, and laments the lack of ability in his fellows. This man has worth. In the highest level a man has the look of knowing nothing.
These are the levels in general. But there is one transcending level, and this is the most excellent of all. This person is aware of the endlessness of entering deeply into a certain Way arid never thinks of himself as having finished. He truly knows his own insufficiencies and never in his whole life thinks that he has succeeded. He has no thoughts of pride but with self-abasement knows the Way to the end. It is said that Master Yagyu once remarked, "I do not know the way to defeat others, but the way to defeat myself.
Throughout your life advance daily, becoming more skillful than yesterday, more skillful than today. This is never-ending.
I believe this to be true at the level of the human race as well as the individual. So, to answer the question with this in mind: there is no end-point in human evolution. Vranak (talk) 19:31, 16 March 2008 (UTC)[reply]
The question as to whether humans will keep evolving in a meaningful way (that is, having more than just incidental circulation of gene frequencies, etc.) and whether that will be for the better or the worst has been asked since the 19th-century. There isn't really a single right answer for it. Human selective pressures have been relatively weak on average for a long time now—we don't tend to let people die off who might otherwise, and no individuals tend to have trouble reproducing if that's what they want to do. Human genetics and human society are both sufficiently complex that the Social Darwinist beliefs that those with money/power/affluence must necessarily be genetically "superior" to those without them have long been shown to be very flawed. There is also the big question of human genetic engineering, which could lead into a variety of different possible directions. Lee Silver's book Remaking Eden suggests a number of interesting possibilities in it without making terribly large assumptions about the scientific advances needed. --98.217.8.46 (talk) 20:20, 16 March 2008 (UTC)[reply]
Assuming machines keep on tending to make life easier and humans do not suddenly come under some sort of drastic survival pressure, then any changes that occur will not be considered improvements by most of us. For instance, lack of toes will not instantly result in death or the inability to reproduce. It might have been so once upon a time, but not now. What is going to stop evolution getting rid of toes if they have no survival benefit? Or take our precious brains, very expensive in energy to maintain, but no survival value now that we have machines to do all our thinking for us. Probably first thing to go if evolution needs to save on energy for any reason. We will only be saved from this terrible future if survival becomes more difficult, or cosmetic genetic engineering becomes legal and cheap. SpinningSpark 20:33, 16 March 2008 (UTC)[reply]
If there is no selection for a trait then it's not going to propagate. And I don't know about you but my machines haven't learned to do any thinking for us; so far, all predictions of human intelligence being useless have proved pretty baseless. At the moment, for all of your fears of brains being cosmetic, most of our top paid professions require big brains, ergo the difficulty of getting into and succeeding in law school, business school, medical school, etc. If anything has characterized human society in the last hundred years it has been the increased importance of expertise and the slow shuffling towards something of a meritocracy (as opposed to the centuries of hard and fast rule by aristocracy). I wouldn't be all that worried.
As for toes, if there is no strong selective pressure for it, it won't catch on. You might think there is not selective pressure against it, but I'm betting that most toe-less individuals would probably disagree to some extent. And in any case, sometimes there is a benefit to missing a few. ;-)
Evolution and genetics is much more complicated than the "toes" example, in any case. Evolution in the face of heavy selective pressures can be quick in small populations, but in a very large population with a lack of selective pressures I think you're going to find mostly a regression towards the mean on the whole, e.g. not much change in either direction over the course of the population as a whole. (Ironically, that is what the founder of eugenics, Francis Galton, thought too. He thought this was stagnation. I consider it just to be stability.) --98.217.8.46 (talk) 20:41, 16 March 2008 (UTC)[reply]
Humans and the demands of their environments are quite mobile. Adaptation often takes the form of simply moving to a place where your deficiencies become meaningless and your talents are irreplaceable. Hence, Lovette goes to jail, Hilary and Barack hit the campaign trail and McCain heads for Iraq. —Preceding unsigned comment added by 71.100.174.10 (talk) 21:27, 16 March 2008 (UTC)[reply]
Yes it's true that brains are still valued in law, science and medicine etc. but my point is that this makes no difference to an individuals abililty to procreate. Hence, it is irrelevant to evolution. The unemployed make just as many children as brain surgeons, possibly more. SpinningSpark 21:45, 16 March 2008 (UTC)[reply]
Money appears to be the deciding factor when it comes to procreation even for the lowest class rather than brains unless brains translate into money. —Preceding unsigned comment added by 71.100.174.10 (talk) 21:56, 16 March 2008 (UTC)[reply]

Biological evolution is an ever ongoing incessant process. It occurred in the past, is occurring at the present, and will continue to do so in the future as long as life exists on the planet. Wisdom89 (T / C) 22:22, 16 March 2008 (UTC)[reply]

The only end state for evolution of humanity will be extinction or immortality. If you agree with the Doomsday argument the former may be quite soon. GameKeeper (talk) 23:28, 16 March 2008 (UTC)[reply]

Extinction is the only eventual possibility, the question is when and why. Personally I'm of the "we're never going to get off of this hunk of rock" school of thought, and would be highly surprised if we made it through the next century, judging by how the last one went. --Fastfission (talk) 23:38, 16 March 2008 (UTC)[reply]

One recent study found that human evolution had recently accelerated (though "recently here" is from 50,000 BC to 10,000 BC or so). 140.247.254.4 (talk) 01:03, 17 March 2008 (UTC)[reply]

Winter Solstice

In the year 1990, did the winter solstice fall on December 22nd? Or the 21st? 71.174.25.42 (talk)Winter

The winter solstice falls on the 21st during leap years and the 22nd during regular years. 1990 was not a leap year. Therefore, December 22 was the winter solstice. --Bowlhover 21:40, 16 March 2008 (UTC)
Unfortunately, it's not quite that simple: "Depending on the shift of the calendar, the event of the Winter solstice occurs some time between December 20 and December 23 each year in the Northern hemisphere." See Winter Solstice#Date, especially the graph on the left. You also need to keep in mind that the solstice proper is a single moment, which happens simultaneously in all time zones. Depending on where on the planet you are, it may be late in the evening on one day, or early in the morning on the next. BTW, [17] (found through Google) gives the 1990 winter solstice happening on Dec 22 at 03:10 UTC. Note that this is actually 9:10 pm on Dec 21 in Chicago. (Also note that the Winter Solstice happens in June for those people south of the equator, due to the inversion of the seasons:) -- 128.104.112.85 (talk) 22:49, 16 March 2008 (UTC)[reply]

Radiography in Thailand?

Does anyone have any information on working as a radiographer in Thailand? I am studying it at the moment in Scotland and would quite like to do an elective placement there. 172.142.123.129 (talk) 23:02, 16 March 2008 (UTC)[reply]

what is the most effective toothpaste?

which brand of toothpaste is the most effective? —Preceding unsigned comment added by 79.122.42.52 (talk) 23:40, 16 March 2008 (UTC)[reply]

The brand you actually use. —Preceding unsigned comment added by 71.100.174.10 (talk) 23:44, 16 March 2008 (UTC)[reply]
That would depend on what you want it to do. There is cavity prevention, tartar control, tooth whitening, fresh breath, etc. Depending on what particular problem(s) you have, a different formulation may help. StuRat (talk) 23:53, 16 March 2008 (UTC)[reply]
The one that claimed to be able to split water molecules during brushing in order to release 'activated oxygen' into the mouth for a deeper clean sounded pretty interesting. Shame they stopped advertising it... --Kurt Shaped Box (talk) 23:58, 16 March 2008 (UTC)[reply]
All you need to do that is brush with hydrogen peroxide. —Preceding unsigned comment added by 71.100.174.10 (talk) 00:02, 17 March 2008 (UTC)[reply]

I'm not sure what brands are available in Hungary, but you might want to check out an independent product reviewer like ConsumerReports.org, or even a website like Epinions.--The Fat Man Who Never Came Back (talk) 23:54, 16 March 2008 (UTC)[reply]

All of the same major ones are available here as the US. Which is the best? (Did you read independent reviews?)

As you haven't said exactly what you mean by "most effective", you might do well to read the Consumer Reports' article on various aspects of toothpastes and the claims made about them here [18]. ៛ Bielle (talk) 01:35, 17 March 2008 (UTC)[reply]

Speed in space(not the drug)

I was discussing with a friend about the age of the universe. A subject way over my head. I brought up that a quasar had been found that was 28 billion light years away from us and gaining. Yet the universe is only 13.73 billion years old. So that would mean that the quasar is traveling faster than the speed of light. It has moved farther than light could in that time. Disance in time = speed.(I know that we are also moving, so some math would have to be done) That is how I and the traffic cops understand speed. But he said that it realy was not moving that fast. The space in between us was actualy growing. My response was Duh, you don't say. That is what distance is. Anyway, he was serrious. He said I would have to understand general reletivity to understand that objects in space are not moving at speed but that space was growing. Can you tell me if he is an idiot or not? And if he is not can you explain this in common language? thanks.

cris —Preceding unsigned comment added by 68.209.69.103 (talk) 23:59, 16 March 2008 (UTC)[reply]

Metric expansion of space, yes, space is in fact getting bigger. A common analogy used in popular science it to imagine a universe that is confined to the surface of a balloon. The balloon obviously has a volume to it, but everything, including light, is permanently stuck to travelling along the surface. If you draw two dots on the balloon to represent, say, you and a quasar, and then you blow up the balloon, you two will get farther and farther apart. Each of you can look around and accurately say, "Everything around me is moving away from me, and I seem to be standing still." So who's right, and who's moving? Or is it merely space that's changing? I'm not a general relativity expert, so that's as close as I'm going to get to explaining this to you. Someguy1221 (talk) 00:08, 17 March 2008 (UTC)[reply]
 OK. I understand the baloon analogy. But the two objects are still gaining distance from one another. Wouldn't that still asign speed to them? Are all objects in space spreading apart? Are they doing this equaly or are some moving faster away than others?  —Preceding unsigned comment added by Loach (talkcontribs) 01:18, 17 March 2008 (UTC)[reply] 

Brushing teeth with bleach?

My teeth aren't super filthy, I've just always wondered if you could bleach your teeth white at home. How dangerous would it be to brush one's teeth with household sodium hypochlorite bleach? It would have to be super-dilute of course, but even then would it be safe? HYENASTE 00:54, 17 March 2008 (UTC)[reply]

Safe yes, if diltued to say the amount you put in a swimming pool or homemade cleaning solution or load of laundry...actually maybe not that last one. But effective...no. I think this is dangerous and stupid and suggest you don't do it. —Preceding unsigned comment added by W-i-k-i-l-o-v-e-r-1-7 (talkcontribs) 01:00, 17 March 2008 (UTC)[reply]

If you want a home solution, try diluted hydrogen peroxide. StuRat (talk) 01:14, 17 March 2008 (UTC)[reply]
3% H2O2 is usually used as an oral antiseptic. Could start there. I'm not sure how effective it would be in whitening. Wisdom89 (T / C) 01:28, 17 March 2008 (UTC)[reply]
Yes yes I know that hydrogen peroxide is a whitener. I'm asking about bleach. HYENASTE 01:29, 17 March 2008 (UTC)[reply]

what is the plant in this picture?

The large bushy plants after the grass in the foreground.

I was walking in the park with my boyfriend today and we could not identify the round bushy plants. They are in the background of this pic...Are they called pampa grass or somthing?W-i-k-i-l-o-v-e-r-1-7 (talk) 00:58, 17 March 2008 (UTC)[reply]

Did the Big Bang go bang

My understanding of the BB is that before it there was nothing. A split second after the BB was hydrogen and maybe some hellium. And that the "Big Bang" was an explosion. Like they said on the History chanel, "the Universe" exploded into existence. So I mentioned this to a friend and said that the Big Bang was an explosion. And that before that there was nothing. He said I was wrong on both accounts. I all but called him a morron. He is not an ID. He is saying that the BB was not an explosion. My Q: If it was not an explosion then what was it? Q: what was it that exploded? What was there before the BB? thanks cris —Preceding unsigned comment added by Loach (talkcontribs) 01:11, 17 March 2008 (UTC)[reply]

eggs

Do all eggs contain all of the amino acids, i.e., if forced to live in a survival situation with nothing but eggs how long could you survive with no other food? —Preceding unsigned comment added by 71.100.174.10 (talk) 01:55, 17 March 2008 (UTC)[reply]